125
Help | Profile | My Organizer | My Bookmarks | Logout Answers and Explanations Test Sections Section 1 Section 2 Section 3 Section 5 Section 6 Section 7 Section 8 Section 9 Section 10 Back to Score Report Essay Online - Practice Test #1 These sample essays were originally handwritten by students but are shown typed here for ease of reading. The essays are displayed exactly as students wrote them, without any corrections to spelling, punctuation, or syntax. One handwritten sample essay is provided to illustrate the need for legible and clear handwriting. Exemplars: Essay Prompt Think carefully about the issue presented in the following excerpt and the assignment below. We often hear that we can learn much about someone or something just by casual observation. We are not required to look beneath the surface or to question how something seems. In fact, we are urged to trust our impressions, often our first impressions, of how a person or a situation seems to be. Yet appearances can be misleading. What seemsisnt always what is. Is the way something seems to be not always the same as it actually is? Plan and write an essay in which you develop your point of view on this issue. Support your position with reasoning and examples taken from your reading, studies, experience, or observations. Back to Score Report Copyright © 2006 The College Board. All rights reserved. Privacy Policy Terms of Use Contact Us 页码1/1 The Official SAT Online Course 2006-11-12 file://E:\新建文件\a2.htm UnRegistered

SAT Online Course Test 1 Answer

Embed Size (px)

DESCRIPTION

answer

Citation preview

严禁用于商业用途!

Help | Profile | My Organizer | My Bookmarks | Logout

Answers and Explanations

Test Sections

Section 1

Section 2

Section 3

Section 5

Section 6

Section 7

Section 8

Section 9

Section 10

Back to Score Report  

Essay     Online - Practice Test #1

These sample essays were originally handwritten by students but are shown typed here for ease of reading. The essays are displayed exactly as students wrote them, without any corrections to spelling, punctuation, or syntax. One handwritten sample essay is provided to illustrate the need for legible and clear handwriting.

Exemplars:

Essay Prompt

Think carefully about the issue presented in the following excerpt and the assignment below.

We often hear that we can learn much about someone or something just by casual observation. We are not required to look beneath the surface or to question how something seems. In fact, we are urged to trust our impressions, often our first impressions, of how a person or a situation seems to be. Yet appearances can be misleading. What “seems” isn’t always what is.

Is the way something seems to be not always the same as it actually is? Plan and write an essay in which you develop your point of view on this issue. Support your position with reasoning and examples taken from your reading, studies, experience, or observations.

Back to Score Report  

Copyright © 2006 The College Board. All rights reserved. Privacy Policy Terms of Use Contact Us

 

页码,1/1The Official SAT Online Course

2006-11-12file://E:\新建文件夹\a2.htm

UnRe

gistered

严禁用于商业用途!

Help | Profile | My Organizer | My Bookmarks | Logout

Answers and Explanations

Test Sections

Section 1

Section 2

Section 3

Section 5

Section 6

Section 7

Section 8

Section 9

Section 10

Back to Score Report  

View Answers and Explanations     Online - Practice Test #1

1 Despite ------- on taking rare tamarins from their habitat, the illegal trade in the tiny monkeys remains -------.

ANSWERS AND EXPLANATIONS Explanation for Correct Answer E :  Choice (E) is correct. "Prohibition" refers to something banned by legal authorities; "active" in this context means in place or functioning. If one were to insert these terms into the text, the sentence would read "Despite prohibitions on taking rare tamarins from their habitat, the illegal trade in the tiny monkeys remains active." Since the word "despite" at the beginning of the sentence indicates that the second missing term will describe an outcome of the first missing term, it makes sense to say that although taking tamarins from their habitat is prohibited, the illegal activity still goes on.

Explanation for Incorrect Answer A :  Choice (A) is incorrect. "Commendations" means expressions of approval. "Obligatory" means legally or morally binding. If one were to insert these terms into the text, the sentence would read "Despite commendations on taking rare tamarins from their habitat, the illegal trade in the tiny monkeys remains obligatory." It is illogical to say that the illegal traders receive the approval of authorities or that the illegal trade is legally binding.

Explanation for Incorrect Answer B :  Choice (B) is incorrect. "Consultations" means deliberations or discussions. "Predominant" means most common. If one were to insert these terms into the text, the sentence would read "Despite consultations on taking rare tamarins from their habitat, the illegal trade in the tiny monkeys remains predominant." Although it is possible to have "consultations" regarding the trade in tamarins, it does not logically follow to say the trade remains common in spite of these discussions.

Explanation for Incorrect Answer C :  Choice (C) is incorrect. "Restrictions" means limits; "local" means confined to a certain place. If one were to insert these terms into the text, the sentence would read "Despite restrictions on taking rare tamarins from their habitat, the illegal trade in the tiny monkeys remains local." It is illogical to say that there are limitations on taking tamarins from their habitat and that the illegal trade remains confined to a particular place.

Explanation for Incorrect Answer D :  Choice (D) is incorrect. "Penalties" means punishments prescribed by law; "illicit" means unlawful. If one were to insert these terms into the text, the sentence would read "Despite penalties on taking rare tamarins from their habitat, the illegal trade in the tiny monkeys remains illicit." It is not logical to say that the illegal trade in tamarins remains unlawful despite the punishments imposed on those who engage in it.

(A) commendations . . obligatory

(B) consultations . . predominant

(C) restrictions . . local

(D) penalties . . illicit

(E) prohibitions . . active

页码,1/21The Official SAT Online Course

2006-11-12file://E:\新建文件夹\a3.htm

UnRe

gistered

严禁用于商业用途!

2 Representing a round world on a flat surface is impossible without some -------: the Mercator projection map shows Greenland as over ten times larger than Mexico, a country in fact only slightly smaller than Greenland.

ANSWERS AND EXPLANATIONS Explanation for Correct Answer C :  Choice (C) is correct. "Distortion" means a lack of proportionality. If one were to insert this term into the text, the sentence would read "Representing a round world on a flat surface is impossible without some distortion: the Meractor projection map shows Greenland as over ten times larger than Mexico, a country in fact only slightly smaller than Greenland." The use of the colon in this sentence indicates that what follows it will be an explanation or definition of the missing word, and since the appearance of Greenland on the Mercator map clearly represents an instance of a lack of proportionality, choice (C) makes sense.

Explanation for Incorrect Answer A :  Choice (A) is incorrect. "Oversight" means watchful care. If one were to insert this term into the text, the sentence would read "Representing a round world on a flat surface is impossible without some oversight: the Mercator projection map shows Greenland as over ten times larger than Mexico, a country in fact only slightly smaller than Greenland." The use of the colon in this sentence indicates that what follows it will be an explanation or definition of the missing word; the appearance of Greenland on the Mercator map does not represent a case of careful watchfulness.

Explanation for Incorrect Answer B :  Choice (B) is incorrect. "Simplification" means a lessening in scope or complexity. If one were to insert this term into the text, the sentence would read "Representing a round world on a flat surface is impossible without some simplification: the Mercator projection map shows Greenland as over ten times larger than Mexico, a country in fact only slightly smaller than Greenland." The use of the colon in this sentence indicates that what follows it will be an explanation or definition of the missing word, and since the appearance of Greenland on the Mercator map represents not so much a lessening of complexity as a change in relative size, choice (B) is incorrect.

Explanation for Incorrect Answer D :  Choice (D) is incorrect. "Sophistication" would in this context most likely mean an increase in development or cultivation. If one were to insert this term into the text, the sentence would read "Representing a round world on a flat surface is impossible without some sophistication: the Mercator projection map shows Greenland as over ten times larger than Mexico, a country in fact only slightly smaller than Greenland." Since the appearance of Greenland on the Mercator map in no way represents a heightened development or level of complexity, choice (D) is incorrect.

Explanation for Incorrect Answer E :  Choice (E) is incorrect. "Superficiality" means affecting only the surface. If one were to insert this term into the text, the sentence would read "Representing a round world on a flat surface is impossible without some superficiality: the Mercator projection map shows Greenland as over ten times larger than Mexico, a country in fact only slightly smaller than Greenland." Since the appearance of Greenland on the Mercator map does not clearly represent a case of only the surface of something being affected or observed, choice (E) does not make sense.

(A) oversight

(B) simplification

(C) distortion

(D) sophistication

(E) superficiality

3 The highly publicized redesign of the car is essentially -------: the exterior has been updated, but the engine remains unchanged.

(A) intuitive

(B) cosmetic

(C) incoherent

页码,2/21The Official SAT Online Course

2006-11-12file://E:\新建文件夹\a3.htm

UnRe

gistered

严禁用于商业用途!

ANSWERS AND EXPLANATIONS Explanation for Correct Answer B :  Choice (B) is correct. "Cosmetic" means superficial, or lacking in true significance. If one were to insert this term into the text, the sentence would read "The highly publicized redesign of the car is essentially cosmetic: the exterior has been updated, but the engine remains unchanged." The colon in this sentence introduces a definition or explanation of the missing term. "Cosmetic" makes sense in this context, since the changes to the car's exterior are more superficial than fundamental.

Explanation for Incorrect Answer A :  Choice (A) is incorrect. "Intuitive" means perceived through instinct. If this term were inserted into the text, the sentence would read "The highly publicized redesign of the car is essentially intuitive: the exterior has been updated, but the engine remains unchanged." It makes little sense to say that the changes to a car's exterior design are "intuitive," or based on instinct.

Explanation for Incorrect Answer C :  Choice (C) is incorrect. "Incoherent" means lacking orderliness or relevance. If one were to insert this term into the text, the sentence would read "The highly publicized redesign of the car is essentially incoherent: the exterior has been updated, but the engine remains unchanged." It is illogical to say that the new design, which changes the exterior but not the engine, is lacking relevance.

Explanation for Incorrect Answer D :  Choice (D) is incorrect. "Consequential" means significant. If one were to insert this term into the text, the sentence would read "The highly publicized redesign of the car is essentially consequential: the exterior has been updated, but the engine remains unchanged." It does not make sense to say that the car's redesign is significant because the information after the colon suggests the opposite.

Explanation for Incorrect Answer E :  Choice (E) is incorrect. "Retroactive" means extending to conditions that existed or originated in the past. If one were to insert this term into the text, the sentence would read "The highly publicized redesign of the car is essentially retroactive: the exterior has been updated, but the engine remains unchanged." It makes little sense to say that the car's redesign reflects conditions that originated in the past.

(D) consequential

(E) retroactive

4 Many of our memories are -------, escaping our consciousness just as we strain to recall a face or a name.

ANSWERS AND EXPLANATIONS Explanation for Correct Answer A :  Choice (A) is correct. "Elusive" means hard to apprehend or find. If one were to insert this term into the text, the sentence would read "Many of our memories are elusive, escaping our consciousness just as we strain to recall a face or a name." Because the use of the comma in the sentence indicates that what follows it will be an explanation or definition of the missing word, it makes sense to say that memories—because they often escape our consciousness at the moment when we are trying to grasp them—are difficult to capture.

(A) elusive

(B) pervasive

(C) unvaried

(D) insensitive

(E) impractical

页码,3/21The Official SAT Online Course

2006-11-12file://E:\新建文件夹\a3.htm

UnRe

gistered

严禁用于商业用途!

Explanation for Incorrect Answer B :  Choice (B) is incorrect. "Pervasive" means tending to permeate, or to spread throughout. If one were to insert this term into the text, the sentence would read "Many of our memories are pervasive, escaping our consciousness just as we strain to recall a face or a name." This does not make sense, since the use of the comma in the sentence indicates that what follows will be an explanation or definition of the missing word, and "pervasive" means nearly the opposite of "escaping our consciousness."

Explanation for Incorrect Answer C :  Choice (C) is incorrect. "Unvaried" means uniform, or all of the same kind. If one were to insert this term into the text, the sentence would read "Many of our memories are unvaried, escaping our consciousness just as we strain to recall a face or a name." It would not make sense here to speak of "uniform memories," certainly not of the tendency of such memories to escape, or elude, our consciousness.

Explanation for Incorrect Answer D :  Choice (D) is incorrect. "Insensitive" means lacking in feeling or being tactless. If one were to insert this term into the text, the sentence would read "Many of our memories are insensitive, escaping our consciousness just as we strain to recall a face or a name." It does not make sense to speak of memories as unfeeling or tactless; in addition, the word "insensitive" is not explained or defined by the material after the comma.

Explanation for Incorrect Answer E :  Choice (E) is incorrect. "Impractical" means incapable of being put to use. If one were to insert this term into the text, the sentence would read "Many of our memories are impractical, escaping our consciousness just as we strain to recall a face or a name." It is not logical to speak of memories as being useful or not useful; moreover, the word "impractical" is not explained by the material after the comma.

5Although Caroline Gordon was rigorously objective in her journalistic writing, her lively and ------- private correspondence ------- a delightful capacity for biting commentary on the social scene.

ANSWERS AND EXPLANATIONS Explanation for Correct Answer E :  Choice (E) is correct. "Entertaining" means lively or amusing; "exhibited" means demonstrated. If one were to insert these terms into the text, the sentence would read "Although Caroline Gordon was rigorously objective in her journalistic writing, her lively and entertaining private correspondence exhibited a delightful capacity for biting commentary on the social scene." The word "although" indicates that the part of the sentence following the comma will contrast with the idea of "rigorous" objectivity, and the use of "and" before the first missing term indicates that this term will complement the word "lively." Therefore, it makes sense to say that Gordon's amusing private letters, in contrast to her straightforward and detached journalistic writing, demonstrated a pleasing subjective element.

Explanation for Incorrect Answer A :  Choice (A) is incorrect. "Incisive" means clear or sharp; "disguised" means hidden or concealed. If one were to insert these terms into the text, the sentence would read "Although Caroline Gordon was rigorously objective in her journalistic writing, her lively and incisive private correspondence disguised a delightful capacity for biting commentary on the social scene." Although it does make sense to say that Gordon wrote "incisive" letters, it does not make sense to say that such a writing style hid a capacity for satire. On the contrary, this type of writing would more likely reveal such a capacity.

Explanation for Incorrect Answer B : 

(A) incisive . . disguised

(B) eloquent . . derided

(C) dispassionate . . demonstrated

(D) exuberant . . minimized

(E) entertaining . . exhibited

页码,4/21The Official SAT Online Course

2006-11-12file://E:\新建文件夹\a3.htm

UnRe

gistered

严禁用于商业用途!

Choice (B) is incorrect. "Eloquent" means vividly expressive; "derided" means ridiculed or treated with contempt. If one were to insert these terms into the text, the sentence would read "Although Caroline Gordon was rigorously objective in her journalistic writing, her lively and eloquent private correspondence derided a delightful capacity for biting commentary on the social scene." While it makes sense to say that Gordon wrote expressive letters, it is illogical to say that such a writing style "derided," or was contemptuous of, a talent for satirical writing.

Explanation for Incorrect Answer C :  Choice (C) is incorrect. "Dispassionate" means unemotional; "demonstrated" means showed. If one were to insert these terms into the text, the sentence would read "Although Caroline Gordon was rigorously objective in her journalistic writing, her lively and dispassionate private correspondence demonstrated a delightful capacity for biting commentary on the social scene." Gordon's letters may have showed a capacity for satire, but letters cannot be both lively and "dispassionate."

Explanation for Incorrect Answer D :  Choice (D) is incorrect. "Exuberant" means extremely enthusiastic; "minimized" means to reduce to the least possible degree. If one were to insert these terms into the text, the sentence would read "Although Caroline Gordon was rigorously objective in her journalistic writing, her lively and exuberant private correspondence minimized a delightful capacity for biting commentary on the social scene." It is possible that Gordon's letters were both lively and enthusiastic, but it does not make sense to say that such a writing style reduced a capacity for satire.

6An effective member of a debating team must focus clearly on the ------- issue and avoid ------- arguments.

ANSWERS AND EXPLANATIONS Explanation for Correct Answer D :  Choice (D) is correct. "Principal" means first in importance; in this context, "peripheral," means of minor importance. If these terms were inserted into the text, the sentence would read "An effective member of a debating team must focus clearly on the principal issue and avoid peripheral arguments." Since the structure of the sentence indicates that the missing terms should be opposites, it makes sense to say that debaters should focus on important issues and avoid less important issues.

Explanation for Incorrect Answer A :  Choice (A) is incorrect. "Equivocal" means uncertain; "obstreperous" means unruly. If these terms were inserted into the text, the sentence would read "An effective member of a debating team must focus clearly on the equivocal issue and avoid obstreperous arguments." Since the structure of the sentence indicates that the missing terms should be opposites, it does not make sense to say that debaters should focus on uncertain issues while avoiding unruly or noisy arguments.

Explanation for Incorrect Answer B :  Choice (B) is incorrect. "Designated" means chosen; "pertinent" means relevant. If these terms were inserted into the text, the sentence would read "An effective member of a debating team must focus clearly on the designated issue and avoid pertinent arguments." Since the structure of the sentence indicates that the missing terms should be opposites, it may make sense here to say that good debates would focus on the "designated," or specified, issue, but it does not make sense to say that good debates would avoid relevant arguments.

Explanation for Incorrect Answer C :  Choice (C) is incorrect. "Comprehensive" means inclusive; in this context, "general," means applicable to the whole. If these terms were inserted into the text, the sentence would read "An effective member of a debating team must focus clearly on the comprehensive issue and avoid general arguments." Since the structure of the sentence indicates that the missing terms should be opposites, it

(A) equivocal. . obstreperous

(B) designated . . pertinent

(C) comprehensive . . general

(D) principal . . peripheral

(E) subtle. . significant

页码,5/21The Official SAT Online Course

2006-11-12file://E:\新建文件夹\a3.htm

UnRe

gistered

严禁用于商业用途!

does not make sense to say that debaters should focus on the inclusive topic while avoiding arguments that pertain to the whole. Rather than being opposites, these terms are somewhat synonymous.

Explanation for Incorrect Answer E :  Choice (E) is incorrect. "Subtle" means refined or artful; "significant" means meaningful. If one were to insert these terms into the text, the sentence would read "An effective member of a debating team must focus clearly on the subtle issue and avoid significant arguments." Since the structure of the sentence indicates that the missing terms should be opposites, it does not make sense to say that debaters should focus on artful, or crafty, issues while avoiding significant arguments. Although the missing terms are somewhat opposite in meaning, they are presented in the wrong order to work in the sentence.

7The ------- with which merchants and landowners in early-nineteenth-century Maryland and Virginia ------- Joshua Johnston’s professional services attests to his artistic skill as a portrait painter.

ANSWERS AND EXPLANATIONS Explanation for Correct Answer A :  Choice (A) is correct. "Avid" means very eager; "to seek" means to go in search of. If one were to insert these terms into the text, the sentence would read "The avidness with which merchants and landowners in early-nineteenth-century Maryland and Virginia sought Joshua Johnston's professional services attests to his artistic skill as a portrait painter." Since the structure of the sentence indicates that the first missing term modifies or describes the second missing term, it is logical to say that people looking for a skilled portrait painter went in search of Johnston with great eagerness.

Explanation for Incorrect Answer B :  Choice (B) is incorrect. "Diffidence" means the quality or state of being hesitant; "purchased" means bought. If one were to insert these terms into the text, the sentence would read "The diffidence with which merchants and landowners in early-nineteenth-century Maryland and Virginia purchased Joshua Johnston's professional services attests to his artistic skill as a portrait painter." This does not make sense, since hesitance on the part of patrons to buy Johnston's work would not be proof of his skill.

Explanation for Incorrect Answer C :  Choice (C) is incorrect. "Patience" is a willingness to wait; "to replace" means to put something new in the place of. If one were to insert these terms into the text, the sentence would read "The patience with which merchants and landowners in early-nineteenth-century Maryland and Virginia replaced Joshua Johnston's professional services attests to his artistic skill as a portrait painter." This does not make sense, since a willingness to put off or forestall replacing Johnston's work does not affirm his skill.

Explanation for Incorrect Answer D :  Choice (D) is incorrect. "Elegant" means refined or graceful; "regarded" means observed. If one were to insert these terms into the text, the sentence would read "The elegance with which merchants and landowners in early-nineteenth-century Maryland and Virginia regarded Joshua Johnston's professional services attests to his artistic skills as a portrait painter." Patrons regarding, or looking at, Johnston's services with a graceful attitude would make little sense and would also not pertain to Johnston's skill as a painter.

Explanation for Incorrect Answer E :  Choice (E) is incorrect. "Zealous" means emotionally intense; "overlooked" means failed to notice. If one were to insert these terms into the text, the sentence would read "The zealousness with which merchants and landowners in early-nineteenth-century Maryland and Virginia overlooked Joshua Johnston's professional services

(A) avidness . . sought

(B) diffidence . . purchased

(C) patience . . replaced

(D) elegance . . regarded

(E) zealousness . . overlooked

页码,6/21The Official SAT Online Course

2006-11-12file://E:\新建文件夹\a3.htm

UnRe

gistered

严禁用于商业用途!

attests to his artistic skill as a portrait painter." It makes no sense to say that patrons exercised great intensity in failing to notice Johnston's services.

8 The man’s colleagues characterized him as ------- because he had an irritable, quarrelsome disposition.

ANSWERS AND EXPLANATIONS Explanation for Correct Answer D :  Choice (D) is correct. "Cantankerous" means ill-tempered and quarrelsome, or argumentative. If one were to insert this term into the text, the sentence would read "The man's colleagues characterized him as cantankerous because he had an irritable, quarrelsome disposition." A man with "an irritable, quarrelsome disposition" could correctly be characterized as cantankerous.

Explanation for Incorrect Answer A :  Choice (A) is incorrect. "Tyrannical" means harsh in the use of power. If one were to insert this term into the text, the sentence would read "The man's colleagues characterized him as tyrannical because he had an irritable, quarrelsome disposition." While a "tyrant," or harsh authority figure, might very well have an irritable disposition, not all people with such a disposition possess enough power to be tyrannical.

Explanation for Incorrect Answer B :  Choice (B) is incorrect. "Disingenuous" means insincere or calculating. If one were to insert this term into the text, the sentence would read "The man's colleagues characterized him as disingenuous because he had an irritable, quarrelsome disposition." A person described as "disingenuous" by others would not likely have a quarrelsome disposition, but would more probably hide his dishonesty behind a superficially pleasing manner.

Explanation for Incorrect Answer C :  Choice (C) is incorrect. To be "sanctimonious" means to affect, or feign, piety or devoutness. If one were to insert this term into the text, the sentence would read "The man's colleagues characterized him as sanctimonious because he had an irritable, quarrelsome disposition." A person described as "sanctimonious" by others would almost certainly not have a quarrelsome disposition, but would instead maintain a superficially pious or virtuous manner.

Explanation for Incorrect Answer E :  Choice (E) is incorrect. To be "morose" means to have a gloomy disposition. If one were to insert this term into the text, the sentence would read "The man's colleagues characterized him as morose because he had an irritable, quarrelsome disposition." A person described as "morose" by others would more likely have a disposition that was more superficially sullen, or somber, than quarrelsome and irritable.

(A) tyrannical

(B) disingenuous

(C) sanctimonious

(D) cantankerous

(E) morose

Passage 1 is by Dorothy Sayers; Passage 2 is adapted from a work by Raymond Chandler.

Passage 1

      The detective story does not and cannot attain the

 loftiest level of literary achievement. Though it deals

页码,7/21The Official SAT Online Course

2006-11-12file://E:\新建文件夹\a3.htm

UnRe

gistered

严禁用于商业用途!

Passage 2

 with the most desperate effects of rage, jealousy, and

Linerevenge, it rarely touches the heights and depths of

5human passion. It presents us with an accomplished

 fact, and looks upon death with a dispassionate eye. It

 does not show us the inner workings of the murderer’s

 mind—it must not, for the identity of the criminal is

 hidden until the end of the book. The most successful

10writers are those who contrive to keep the story running

 from beginning to end upon the same emotional level,

 and it is better to err in the direction of too little feeling

  than too much.

      I think what was really gnawing at Dorothy Sayers in

15her critique of the detective story was the realization that

 her kind of detective story was an arid formula unable to

 satisfy its own implications. If the story started to be about

 real people, they soon had to do unreal things to conform

 to the artificial pattern required by the plot. When they did

20unreal things, they ceased to be real themselves. Sayers’

 own stories show that she was annoyed by this triteness.

 Yet she would not give her characters their heads and let

  them make their own mystery.

9

Which best describes the relationship between the two passages?

(A) Passage 1 explains the evolution of a genre, while Passage 2 challenges the notion of a distinct genre.

(B) Passage 1 discusses the constraints of a genre, while Passage 2 contends that many of these constraints are self-imposed.

(C) Passage 1 celebrates a genre, while Passage 2 points out its deficiencies.

(D) Passage 1 explains the popularity of a genre, while Passage 2 questions its commercial success.

(E) Passage 1 compares a genre unfavorably to other types of writing, while Passage 2 argues that the genre has unique features.

页码,8/21The Official SAT Online Course

2006-11-12file://E:\新建文件夹\a3.htm

UnRe

gistered

严禁用于商业用途!

ANSWERS AND EXPLANATIONS Explanation for Correct Answer B :  Choice (B) is correct. In Passage 1 Sayers identifies the constraints felt by "the most successful writers" of detective stories to "contrive to keep the story running from beginning to end upon the same emotional level." Passage 2 argues that Sayers seemed unable to free her own stories from that "arid formula."

Explanation for Incorrect Answer A :  Choice (A) is incorrect. Passage 1 describes certain aspects of writing in the detective genre but does not address how the genre changed over time, while Passage 2 further elaborates on those same aspects of the genre.

Explanation for Incorrect Answer C :  Choice (C) is incorrect. While Passage 2 does discuss certain deficiencies in the approach of some writers who work in the genre, Passage 1 focuses on the constraints inherent in writing in the detective genre, and so could not be said to be celebrating it.

Explanation for Incorrect Answer D :  Choice (D) is incorrect. While Passage 1 does mention the approach taken by "the most successful writers," Passage 2 confines its discussion to the artistic rather than commercial successes of those writers.

Explanation for Incorrect Answer E :  Choice (E) is incorrect. While Passage 1 says that the detective genre "does not and cannot attain the loftiest level of literary achievement," Passage 2 does not make the argument that the genre has features that distinguish it from other fiction.

10The author of Passage 2 would most likely respond to the statement in lines 4-5, Passage 1 (“it rarely . . . passion”), by

ANSWERS AND EXPLANATIONS Explanation for Correct Answer A :  Choice (A) is correct. The author of Passage 2 argues that authors like Sayers fail in that they do not give their "characters their heads and let them make their own mystery"— in other words, that these authors do not let their characters grow and show the full range of human emotions.

Explanation for Incorrect Answer B :  Choice (B) is incorrect. The author of Passage 2 says that Sayers— and, by implication, similar writers—does use formulas.

Explanation for Incorrect Answer C :  Choice (C) is incorrect. The author of Passage 2, rather than accepting the view expressed in Passage 1 that strong emotions are inappropriate in detective fiction, holds that such an approach is formulaic and that characters in detective stories should "be real."

Explanation for Incorrect Answer D :  Choice (D) is incorrect. The author of Passage 2 is not concerned with the commercial success or failure of detective stories, but with their artistic integrity.

Explanation for Incorrect Answer E :  Choice (E) is incorrect. Although the author of Passage 2 does mention plot, he

(A) arguing that this approach limits the characters’ development

(B) denying that most writers of detective stories rely on formulas

(C) agreeing that strong emotions are out of place in detective stories

(D) conceding that great literature is seldom commercially successful

(E) concurring that readers are primarily interested in plot

页码,9/21The Official SAT Online Course

2006-11-12file://E:\新建文件夹\a3.htm

UnRe

gistered

严禁用于商业用途!

does not discuss the attitude of readers toward plots in detective stories.

11 Which of the following characteristics of detective stories presented in Passage 1 would be LEAST likely to be attributed to the “pattern” mentioned in line 19, Passage 2 ?

ANSWERS AND EXPLANATIONS Explanation for Correct Answer B :  Choice (B) is correct. The intense emotions resulting from rage, jealousy, and revenge are very unlikely to appear in detective stories written following the "artificial pattern" mentioned in Passage 2.

Explanation for Incorrect Answer A :  Choice (A) is incorrect. It is unlikely that a story created following the "artificial pattern," with its "unreal" characters, would be able to achieve the level of high literary art.

Explanation for Incorrect Answer C :  Choice (C) is incorrect. Detective stories created according to the "artificial pattern" would be more likely to focus on facts than on emotional responses.

Explanation for Incorrect Answer D :  Choice (D) is incorrect. It is very likely that detective stories created following the "pattern" would display a "dispassionate," or unemotional, attitude toward dramatic events.

Explanation for Incorrect Answer E :  Choice (E) is incorrect. The "artificial pattern," in order to avoid emotional ups and downs in a story, would probably recommend avoiding insights into the "inner workings of the murderer's mind."

(A) “cannot attain the loftiest level of literary achievement” (lines 1-2)

(B) “deals with the most desperate effects of rage, jealousy, and revenge” (lines 2-4)

(C) “presents us with an accomplished fact” (lines 5-6)

(D) “looks upon death with a dispassionate eye” (line 6)

(E) “does not show us the inner workings of the murderer’s mind” (lines 7-8)

12 Passage 1 suggests that Sayers would most likely respond to lines 17-20, Passage 2 (“If the story started . . . themselves”), by pointing out that

ANSWERS AND EXPLANATIONS Explanation for Correct Answer C :  Choice (C) is correct. Sayers states that "the most successful writers" are those who keep their stories at an even "emotional level" until the very end, and even that "it is better to err in the direction of too little feeling than too much." These lines suggest that plot should always take precedence over character development.

Explanation for Incorrect Answer A :  Choice (A) is incorrect. Sayers argues that unlike great literary fiction, detective

(A) great writers seldom explore the range of human emotions

(B) detective stories do not address the consequences of people’s emotions

(C) detective stories are driven by the plot, not by the characters

(D) readers of detective stories prefer unrealistic situations

(E) real people often act in ways that are unexpected

页码,10/21The Official SAT Online Course

2006-11-12file://E:\新建文件夹\a3.htm

UnRe

gistered

严禁用于商业用途!

fiction "rarely touches the heights and depths of human passion."

Explanation for Incorrect Answer B :  Choice (B) is incorrect. Sayers states that detective stories deal "with the most desperate effects of rage, jealousy, and revenge."

Explanation for Incorrect Answer D :  Choice (D) is incorrect. Although Sayers might agree with the view that readers prefer the formalized and unrealistic situations found in many detective stories, she does not address this issue in the passage.

Explanation for Incorrect Answer E :  Choice (E) is incorrect. Although Sayers might agree with the notion that "real people often act in ways that are unexpected," she argues in the passage that the successful approach to writing detective stories necessarily involves the creation of unreal and constrained characters.

This passage is adapted from a series in which a college professor dramatizes the lectures of famous scientists from the past. Here he speaks as Louis Pasteur (1822-1895). In this part of the lecture, Pasteur has just described his discovery of the effect of heating certain microbes that infect bottled beverages (the process later named pasteurization).

      But these undesirable microbes! Where and how did

  they arise? By spontaneous generation,* as some believe?

 When I began to ask these questions of myself and of my

Linestudents and colleagues, my close friends said: “Oh, no,

5do not waste your time on such worthless philosophical

 problems. Many a scientist has floundered and perished in

 the quagmire of spontaneous generation.” I replied: “But

 the origin of life is a profound problem.” With few excep-

 tions, past discourses on spontaneous generation have been

10metaphysical exercises conducted with great passion, but

  without adding to our scientific knowledge.

      I could not set aside my burning desire to bring a little

 stone, God willing, to the frail edifice of our knowledge of

 the deep mysteries of life and death, where all our intellects

15have so lamentably failed. In defense of nonapplied science

 I have repeatedly told my students that without theory,

 practice is but routine. Only theory is able to cause the spirit of invention to arise and develop. It is

页码,11/21The Official SAT Online Course

2006-11-12file://E:\新建文件夹\a3.htm

UnRe

gistered

严禁用于商业用途!

  important that

 students should not share the opinion of those who disdain

20everything in science that has no immediate application. In

 science, chance favors only the mind that is prepared.

 I repeat: in science, chance favors only the mind that

  is prepared.

      I first confirmed the experiments of the Italian abbé,

25Lazzaro Spallanzani, known also for his studies in gastric

 digestion. I made a nutritious broth, put it in a flask such

 as this [Pasteur holds up a large flask containing a brown

 solution], heated it to violent boiling, and then sealed the

 neck of the flask in a flame. My results agreed with those

30of Spallanzani: the broth remained pure. But if the neck

 be broken to admit air, the broth soon became putrid. My

 critics said that the heating made the air in the flask unfit

 for spontaneous generation. Only when fresh air is admitted

 can life begin anew. I argued in vain—even before our

35Academy of Sciences—that the putrefaction was caused by

 admission of bacteria. More convincing experiments were

  needed.

      I opened flasks of sterilized broth in the cellar of the

 Paris observatory, where the air was still. Only one flask

40out of ten became putrid, whereas eleven flasks out of

 eleven opened in the courtyard quickly acquired a rich

 growth of bacteria. I journeyed to Mt. Montanvert in the

 Alps, where I opened twenty flasks of sterilized broth. Only

 one became putrid. I concluded that the air in the cellar and

the air above the glacier were freer of bacteria

页码,12/21The Official SAT Online Course

2006-11-12file://E:\新建文件夹\a3.htm

UnRe

gistered

严禁用于商业用途!

45 than the air

 in the city streets. But my adversaries performed similar

 experiments with different results. Perhaps they were not

 careful to follow my procedures. The neck of the flask must

 be heated first to kill the bacteria on the glass; then a heated

50instrument must be used to break the tip of the flask as it is

 held high above the head. Immediately thereafter the flask

 must be sealed again in a flame [Pasteur demonstrates the

 procedure]. In these difficult researches, while I sternly

 object to frivolous contradictions, I feel nothing but grat-

55itude toward those who warn me if I should be in error.

      I then devised a conclusive experiment. I boiled a nutri-

 tious infusion in a flask with a long curved neck like this

 one. The tip of the neck was not sealed but left open to the

 outside air. Thus, there was no hindrance to the entrance of

60fresh air with its “vital force” as claimed by the advocates

 of spontaneous generation. But bacteria in the entering air

 would be trapped by the walls of the long glass tube. The

 fluid remained sterile so long as the flask was maintained

 in the vertical position. If, however, I contaminated the

65broth by allowing some of it to flow into the neck and then

 back into the flask, putrefaction promptly followed. So we

 see that life does not arise spontaneously. Life comes only

  from life.

 

* The supposed origination of living matter directly from lifeless matter

13 The focus of the lecture is on how Pasteur

页码,13/21The Official SAT Online Course

2006-11-12file://E:\新建文件夹\a3.htm

UnRe

gistered

严禁用于商业用途!

ANSWERS AND EXPLANATIONS Explanation for Correct Answer A :  Choice (A) is correct. The lecture details how Pasteur’s experiments "disproved," or proved wrong, the "erroneous," or incorrect, theory of spontaneous generation.

Explanation for Incorrect Answer B :  Choice (B) is incorrect. The lecture does not discuss the documentation or publication of Pasteur’s experiments, but it does discuss what they proved.

Explanation for Incorrect Answer C :  Choice (C) is incorrect. Pasteur’s experiments were concerned not with killing "microbes," or tiny organisms, but with discovering how and why they come to grow in previously sterile, or clean, liquids.

Explanation for Incorrect Answer D :  Choice (D) is incorrect. Although Pasteur may have later applied his findings to new problems, this passage is concerned only with how Pasteur proved the theory of spontaneous generation to be inaccurate.

Explanation for Incorrect Answer E :  Choice (E) is incorrect. The lecture is concerned not with the improvement of laboratory standards but with Pasteur’s study of the growth of bacteria.

(A) disproved an erroneous theory

(B) documented and published his experiments

(C) developed a process for killing microbes

(D) applied his findings on spontaneous generation to new problems

(E) contributed to the improvement of laboratory research standards

14 In the lecture, Pasteur concludes that the answer to the question “Where and how did they arise?” (lines 1-2) is

ANSWERS AND EXPLANATIONS Explanation for Correct Answer B :  Choice (B) is correct. At the end of the lecture, Pasteur declares that “[l]ife comes only from life,” or that microbes that are observed in liquid broth have migrated there from the air.

Explanation for Incorrect Answer A :  Choice (A) is incorrect. In the lecture, Pasteur concludes just the opposite: bacteria do not arise "spontaneously," or suddenly, without outside influence.

Explanation for Incorrect Answer C :  Choice (C) is incorrect. Pasteur does not conclude that "impurities," or unclean elements, in the original broth caused the growth of bacteria. Rather, his experiment shows that existing bacteria in the air were the source of bacteria in the broth.

Explanation for Incorrect Answer D :  Choice (D) is incorrect. The curved neck of the flask helped Pasteur better conduct his experiment but was itself not responsible for the growth of bacteria in the broth.

(A) spontaneously

(B) from airborne bacteria

(C) from impurities in the original broth

(D) from the curved neck of a flask

(E) from a broken flask

页码,14/21The Official SAT Online Course

2006-11-12file://E:\新建文件夹\a3.htm

UnRe

gistered

严禁用于商业用途!

Explanation for Incorrect Answer E :  Choice (E) is incorrect. The broken flask helped Pasteur to control the way that air and bacteria passed into the broth, but it does not explain where and how the bacteria grew.

15 In the first two paragraphs (lines 1-23), Pasteur is primarily concerned with

ANSWERS AND EXPLANATIONS Explanation for Correct Answer C :  Choice (C) is correct. Pasteur uses the first two paragraphs of his lecture to explain his "motivation," or reason, for studying the microbes—that he wanted to contribute to the scientific knowledge of the origins of life.

Explanation for Incorrect Answer A :  Choice (A) is incorrect. The first two paragraphs do not explain the results of Pasteur’s experiments. Rather, they explain why he undertook the study.

Explanation for Incorrect Answer B :  Choice (B) is incorrect. The opposite is true: Pasteur criticizes those who have passion but have failed to increase scientific knowledge.

Explanation for Incorrect Answer D :  Choice (D) is incorrect. Although Pasteur mentions that he was faced with criticism, nowhere in these paragraphs does he attack his detractors.

Explanation for Incorrect Answer E :  Choice (E) is incorrect. The purpose of these paragraphs is not to counter a false impression concerning Pasteur’s interest in applied or nonapplied science but to explain why he chose to study the growth of bacteria.

(A) summarizing the results of his experiments about spontaneous generation

(B) criticizing those who have taken the passion out of science

(C) establishing his motivation for studying the origin of microbes

(D) attacking critics of his experiments

(E) correcting the impression that he is concerned only with experiments that have immediate application

16The word "quagmire" (line 7) is used primarily to emphasize the

ANSWERS AND EXPLANATIONS Explanation for Correct Answer B :  Choice (B) is correct. It makes sense to describe spontaneous generation, a frustrating area of research, as a "quagmire," or complicated situation from which it is difficult to escape.

Explanation for Incorrect Answer A :  Choice (A) is incorrect. In this context, "quagmire" refers to the problem of spontaneous generation, not to the general state of scientific ignorance, or lack of knowledge.

(A) state of scientific ignorance in the 1800’s

(B) futility of a particular line of research

(C) moral dilemma faced by scientists like Pasteur

(D) failure of some to distinguish between pure and applied science

(E) tendency of unsuccessful scientists to look for simple solutions

页码,15/21The Official SAT Online Course

2006-11-12file://E:\新建文件夹\a3.htm

UnRe

gistered

严禁用于商业用途!

Explanation for Incorrect Answer C :  Choice (C) is incorrect. "Morality," or standards of right and wrong, is unrelated to Pasteur’s concerns respecting spontaneous generation

Explanation for Incorrect Answer D :  Choice (D) is incorrect. Whether or not some people differentiate between "pure," or theoretical, and "applied," or practical, science is unrelated to the difficulty of the problem of spontaneous generation itself.

Explanation for Incorrect Answer E :  Choice (E) is incorrect. Pasteur does not criticize proposed solutions to the problem of spontaneous generation on grounds of simplicity; rather, he says they were "metaphysical" and not truly scientific.

17 Pasteur characterizes “past discourses on spontaneous generation” (line 9) as having

ANSWERS AND EXPLANATIONS Explanation for Correct Answer D :  Choice (D) is correct. Pasteur acknowledges that past experiments had been conducted with passion but argues that they were not tested in the laboratory and therefore did not increase the body of scientific knowledge on the subject.

Explanation for Incorrect Answer A :  Choice (A) is incorrect. Pasteur does not describe past research as demonstrating the "futility," or pointlessness, of practical studies. On the contrary, he criticizes past studies for failing to be practical or scientific.

Explanation for Incorrect Answer B :  Choice (B) is incorrect. "Nowhere" in his discussion of past research on spontaneous generation does Pasteur mention the sterilization of any laboratory apparatus, or tool.

Explanation for Incorrect Answer C :  Choice (C) is incorrect. Pasteur does not discuss his own early experiments in the context of past research on spontaneous generation.

Explanation for Incorrect Answer E :  Choice (E) is incorrect. Pasteur claims that past experiments failed to increase scientific knowledge. Therefore, it does not make sense to say that they "resolved," or settled, the "controversy," or debate, around the question of spontaneous generation.

(A) demonstrated the futility of practical scientific studies

(B) failed because of incomplete knowledge about sterilization of apparatus

(C) enabled him to understand inconsistencies in his early experiments

(D) failed to increase scientific knowledge

(E) resolved much of the controversy surrounding the issue

18The “little stone” (lines 12-13) refers to the

ANSWERS AND EXPLANATIONS

(A) slight addition that Pasteur hoped to make to the existing body of facts

(B) small effect that Pasteur wanted to have on one person’s learning

(C) minor disappointment Pasteur felt at being rebuffed by his colleagues

(D) narrow-mindedness of those who cling to scientific fallacies

(E) imperceptible progress that Pasteur had made in understanding spontaneous generation

页码,16/21The Official SAT Online Course

2006-11-12file://E:\新建文件夹\a3.htm

UnRe

gistered

严禁用于商业用途!

Explanation for Correct Answer A :  Choice (A) is correct. Pasteur refers to his desire to bring a “little stone” to the “frail edifice,” or unsteady structure, of the human knowledge of life and death. In other words, the goal of his research was to make a contribution to the existing body of scientific fact, thereby increasing human knowledge.

Explanation for Incorrect Answer B :  Choice (B) is incorrect. Pasteur does not want to have a small effect on one person; he wants to have an effect on scientific knowledge in general.

Explanation for Incorrect Answer C :  Choice (C) is incorrect. The “little stone” Pasteur is speaking of is his own potential contribution to science, not the "rebuff," or criticism, he received at the hands of his fellow scientists.

Explanation for Incorrect Answer D :  Choice (D) is incorrect. It does not make sense to say that Pasteur desired to contribute to the narrow-mindedness of others.

Explanation for Incorrect Answer E :  Choice (E) is incorrect. The “little stone” refers to the contribution Pasteur desired to make when he began his research, not the results of his experiments.

19 Pasteur’s pronouncement about preparation and chance in lines 20-23 implies that

ANSWERS AND EXPLANATIONS Explanation for Correct Answer D :  Choice (D) is correct. In the lecture, Pasteur encourages his students to pursue projects whose applications are not yet known so that they will be better able to recognize and "exploit," or take advantage of, chance discoveries in the laboratory.

Explanation for Incorrect Answer A :  Choice (A) is incorrect. The opposite is true: Pasteur believes that pursuing projects without immediate applications prepares a scientist for future discoveries and is as important as the practical work itself.

Explanation for Incorrect Answer B :  Choice (B) is incorrect. According to the lecture, it is not practice but a solid grounding in theory that prepares a scientist to make "significant," or important, discoveries.

Explanation for Incorrect Answer C :  Choice (C) is incorrect. The statement is concerned not with the number of scientists who create useful theories but with the best way to prepare oneself for scientific discovery.

Explanation for Incorrect Answer E :  Choice (E) is incorrect. Pasteur does not claim that good luck and timing result in discoveries of any kind. Rather, he argues that pursuing research whose applications are unknown prepares a scientist to better understand and make use of later discoveries.

(A) only projects that have an immediate application are important

(B) practice improves a scientist’s chances of making a significant discovery

(C) few scientists are lucky enough to devise useful theories

(D) work on projects that have no immediate application prepares scientists to exploit chance discoveries

(E) most scientific discoveries that have no immediate application are the result of good luck and timing

20 In context, the reference to the Academy of Sciences (line 35) serves to suggest why

页码,17/21The Official SAT Online Course

2006-11-12file://E:\新建文件夹\a3.htm

UnRe

gistered

严禁用于商业用途!

ANSWERS AND EXPLANATIONS Explanation for Correct Answer D :  Choice (D) is correct. The passage indicates that after failing to convince the academy of his findings, Pasteur felt it was necessary to design more "persuasive," or convincing, experiments.

Explanation for Incorrect Answer A :  Choice (A) is incorrect. The fact that Pasteur argued unsuccessfully for his research before the academy may have motivated him to prove his critics wrong, but it does not explain his initial desire to contribute to scientific knowledge.

Explanation for Incorrect Answer B :  Choice (B) is incorrect. Pasteur "replicated," or repeated, Spallanzani’s experiments before he presented his findings to the academy.

Explanation for Incorrect Answer C :  Choice (C) is incorrect. The reference to the academy illustrates how difficult it was for Pasteur’s experiments to be accepted by the scientific community, not that his research joined with that of others who were also disproving spontaneous generation.

Explanation for Incorrect Answer E :  Choice (E) is incorrect. The incident at the Academy of Sciences does not suggest that Pasteur’s colleagues thought spontaneous generation was an inappropriate research project. Rather, it shows that they were not convinced by his findings.

(A) Pasteur was so determined to make a significant contribution to scientific knowledge

(B) Pasteur felt compelled to replicate Spallanzani’s experiments

(C) spontaneous generation had already begun to be discredited when Pasteur began his experimentation

(D) Pasteur believed he needed to design experiments that were more persuasive

(E) spontaneous generation was viewed by Pasteur’s colleagues as a topic that was unfit for scientific study

21 In line 41, "rich" most nearly means

ANSWERS AND EXPLANATIONS Explanation for Correct Answer C :  Choice (C) is correct. In this context, which speaks of "a rich growth of bacteria," "rich" means abundant, or existing in large quantities.

Explanation for Incorrect Answer A :  Choice (A) is incorrect. It does not make sense to describe the growth of bacteria as "precious," or rare.

Explanation for Incorrect Answer B :  Choice (B) is incorrect. "Vital," or indispensable for survival, does not describe the growth of bacteria in Pasteur’s flasks.

Explanation for Incorrect Answer D :  Choice (D) is incorrect. Although the presence of bacteria was "meaningful," or

(A) precious

(B) vital

(C) abundant

(D) meaningful

(E) productive

页码,18/21The Official SAT Online Course

2006-11-12file://E:\新建文件夹\a3.htm

UnRe

gistered

严禁用于商业用途!

significant, for Pasteur’s research, it does not make sense to describe the growth itself as such.

Explanation for Incorrect Answer E :  Choice (E) is incorrect. In this context, "rich" is used to describe the quantity of bacteria present. It does not make sense to define that amount as "productive," or able to create something.

22 The "conclusive experiment" (line 56) performed by Pasteur was designed to answer critics who argued that

ANSWERS AND EXPLANATIONS Explanation for Correct Answer E :  Choice (E) is correct. Pasteur's critics claimed that by using heat to seal the neck of the bottle in earlier experiments, Pasteur had made the air unsuitable for spontaneous generation to take place. In his last experiment, he answered this opposition by using a special flask that permitted fresh air, but not bacteria, to come into contact with the liquid.

Explanation for Incorrect Answer A :  Choice (A) is incorrect. Pasteur's critics argued that sterilizing, or purifying, the flasks had made the air unfit for spontaneous generation. In other words, there had been too much sterilization, not an inadequate amount.

Explanation for Incorrect Answer B :  Choice (B) is incorrect. The last experiment was designed to answer critics who maintained confidence in the theory of spontaneous generation, not those who argued against nonapplied science.

Explanation for Incorrect Answer C :  Choice (C) is incorrect. The "conclusive experiment" was not designed to address inconsistencies between Pasteur's and his colleagues' data, which the passage attributes to carelessness. It was designed to satisfy those who argued that the liquid must come into contact with fresh air in order for spontaneous generation to occur.

Explanation for Incorrect Answer D :  Choice (D) is incorrect. The nutritional content of the various broths is unrelated to the results or design of the experiments.

(A) the apparatus used in Pasteur's earlier experiments had not been adequately sterilized

(B) Pasteur's experiments related to spontaneous generation had no immediate application

(C) the results of Pasteur's experiments in the Alps and in the cellar could not be replicated

(D) the broth in the flasks of Pasteur's earlier experiments was not nutritious enough

(E) heating made the air in the flasks of the earlier experiments unfit for spontaneous generation

23In the context of the passage as a whole, the “vital force” (line 60) is best described as

ANSWERS AND EXPLANATIONS

(A) what Pasteur called the basic unit of life

(B) a term that was outdated in Pasteur’s time

(C) nutrients necessary for sustaining life

(D) that which has the power to destroy life

(E) what opponents of Pasteur believed to be a source of life

页码,19/21The Official SAT Online Course

2006-11-12file://E:\新建文件夹\a3.htm

UnRe

gistered

严禁用于商业用途!

Explanation for Correct Answer E :  Choice (E) is correct. In the context of the passage, it makes sense to say that the “vital force” is the element that Pasteur’s opponents believed to be responsible for the spontaneous generation of life.

Explanation for Incorrect Answer A :  Choice (A) is incorrect. The “vital force” is not what Pasteur called the basic unit of life; that was his fellow scientists’ word for an energy or substance that caused living matter to appear.

Explanation for Incorrect Answer B :  Choice (B) is incorrect. Nothing in the passage indicates that this term was no longer in use in Pasteur’s time. On the contrary, the fact that it was used by his colleagues indicates that it was a popular notion.

Explanation for Incorrect Answer C :  Choice (C) is incorrect. The “vital force” is what was thought to cause life, not the nutrients, or nourishing material.

Explanation for Incorrect Answer D :  Choice (D) is incorrect. The “vital force” was not seen as something that detroys life but as something that brings life into being.

24 In his conclusive experiment, Pasteur kept the flasks vertical (line 64) in order to

ANSWERS AND EXPLANATIONS Explanation for Correct Answer C :  Choice (C) is correct. When kept in the vertical position, the design of the flasks prevented the fluid in the bowl from touching bacteria trapped in the curved neck.

Explanation for Incorrect Answer A :  Choice (A) is incorrect. The opposite of this statement is true. The passage states that in the vertical position, there was no "hindrance," or obstacle, to the entrance of fresh air.

Explanation for Incorrect Answer B :  Choice (B) is incorrect. Pasteur did not position the flasks vertically to "retain," or keep, the liquid inside; he did it to control the flow of air and bacteria.

Explanation for Incorrect Answer D :  Choice (D) is incorrect. Pasteur kept the flasks in the vertical position not to avoid disturbing the liquid but to trap the bacteria and prevent it from coming into contact with the liquid.

Explanation for Incorrect Answer E :  Choice (E) is incorrect. Pasteur revised his method and used the curved flask not to "replicate," or repeat, his previous experiments but to improve upon them.

(A) prevent fresh air from entering them

(B) retain the boiling liquid inside the flasks

(C) prevent the fluid from touching trapped bacteria

(D) avoid disturbing the solution inside

(E) replicate his previous experiments exactly

    

Back to Score Report  

Copyright © 2006 The College Board. All rights reserved. Privacy Policy Terms of Use Contact Us

页码,20/21The Official SAT Online Course

2006-11-12file://E:\新建文件夹\a3.htm

UnRe

gistered

严禁用于商业用途!

 

页码,21/21The Official SAT Online Course

2006-11-12file://E:\新建文件夹\a3.htm

UnRe

gistered

严禁用于商业用途!

Help | Profile | My Organizer | My Bookmarks | Logout

Answers and Explanations

Test Sections

Section 1

Section 2

Section 3

Section 5

Section 6

Section 7

Section 8

Section 9

Section 10

Back to Score Report  

View Answers and Explanations     Online - Practice Test #1

1 Which of the following represents the total cost, in dollars, of compact discs at each and compact disc cases at each? (Disregard sales tax.)

ANSWERS AND EXPLANATIONS Explanation for Correct Answer A :  Choice (A) is correct. The cost of the compact discs (CDs) purchased at a price of each is dollars. The cost of the CD cases purchased at

a price of each is dollars. Therefore, the total cost, in dollars, is

Explanation for Incorrect Answer B :  Choice (B) is not correct. There were CDs purchased (not ), and CD cases purchased (not ).

Explanation for Incorrect Answer C :  Choice (C) is not correct. This equation corresponds to the situation where all the items purchased had a price of not the situation where some had a

price of and others had a price of .

Explanation for Incorrect Answer D :  Choice (D) is not correct. See the explanation for the correct response (A).

Explanation for Incorrect Answer E :  Choice (E) is not correct. See the explanation for the correct response (A).

(A)

(B)

(C)

(D)

(E)

2

If the areas of the two rectangles in the figure above are equal, which of the following could be the coordinates of point ?

(A)

页码,1/15The Official SAT Online Course

2006-11-12file://E:\新建文件夹\a4.htm

UnRe

gistered

严禁用于商业用途!

ANSWERS AND EXPLANATIONS Explanation for Correct Answer C :  Choice (C) is correct. Since is in the second quadrant, it must have a negative -coordinate and a positive -coordinate. If has coordinates

then the area of the rectangle in the second quadrant is For

the two rectangles to be of equal area, must equal The only

answer choice that fits both of these criteria is

Explanation for Incorrect Answer A : 

Choice (A) is not correct. is in the second quadrant, and is a

point in the third quadrant.

Explanation for Incorrect Answer B : 

Choice (B) is not correct. If has coordinates then the rectangle in

the second quadrant has only half the area of the rectangle in the first quadrant, and the problem requires that they have the same area.

Explanation for Incorrect Answer D : 

Choice (D) is not correct. is in the second quadrant, and is a

point in the fourth quadrant.

Explanation for Incorrect Answer E : 

Choice (E) is not correct. is in the second quadrant, and is a point

in the first quadrant.

(B)

(C)

(D)

(E)

3 A box contains solid-colored marbles that are either orange, blue, or green. If percent of the marbles are orange and percent of the marbles are blue, what

percent are green?

ANSWERS AND EXPLANATIONS Explanation for Correct Answer B :  Choice (B) is correct. Since percent of the marbles are orange, percent are blue, and the remaining marbles are green, it follows that

percent of the marbles are green.

Explanation for Incorrect Answer A :  Choice (A) is not correct. This is the percentage of the marbles that are blue (and also the percentage of the marbles that are orange).

Explanation for Incorrect Answer C :  Choice (C) is not correct. See the explanation for the correct response (B).

Explanation for Incorrect Answer D : 

(A)

(B)

(C)

(D)

(E)

页码,2/15The Official SAT Online Course

2006-11-12file://E:\新建文件夹\a4.htm

UnRe

gistered

严禁用于商业用途!

Choice (D) is not correct. This is the percentage of the marbles that are not green.

Explanation for Incorrect Answer E :  Choice (E) is not correct. This is the percentage of the marbles that are not orange (and also the percentage that are not blue).

4

Sets and are shown above. If is a member of set and is a member of

set , which of the following CANNOT be equal to the product

ANSWERS AND EXPLANATIONS Explanation for Correct Answer A :  Choice (A) is correct. The only ways to write as the product of two positive integers (ignoring the order of the factors) are or

While and are both members of neither nor is a

member of Also, while is a member of is not a member of Therefore, cannot be written as the product of two integers such that one is a member of and the other is a member of

Explanation for Incorrect Answer B :  Choice (B) is not correct. The number can be written as where is a member of and is a member of The question asks for a number that cannot be written as the product of a member of with a member of

Explanation for Incorrect Answer C :  Choice (C) is not correct. The number can be written as where is a member of and is a member of The question asks for a number that cannot be written as the product of a member of with a member of

Explanation for Incorrect Answer D :  Choice (D) is not correct. The number can be written as where is a member of and is a member of The question asks for a number that cannot be written as the product of a member of with a member of

Explanation for Incorrect Answer E :  Choice (E) is not correct. The number can be written as where is a member of and is a member of The question asks for a number that cannot be written as the product of a member of with a member of

(A)

(B)

(C)

(D)

(E)

5If then could be which of the following?

页码,3/15The Official SAT Online Course

2006-11-12file://E:\新建文件夹\a4.htm

UnRe

gistered

严禁用于商业用途!

ANSWERS AND EXPLANATIONS Explanation for Correct Answer E : 

Choice (E) is correct. If then Of the choices,

the only value of that satisfies this condition is

Explanation for Incorrect Answer A : 

Choice (A) is not correct. If then However,

Explanation for Incorrect Answer B : 

Choice (B) is not correct. If then However,

Explanation for Incorrect Answer C : 

Choice (C) is not correct. If then However,

Explanation for Incorrect Answer D : 

Choice (D) is not correct. If then However,

(A)

(B)

(C)

(D)

(E)

6

If which of the following must be true?

ANSWERS AND EXPLANATIONS Explanation for Correct Answer B :  Choice (B) is correct. The right side of the equation can be expanded:

Then can be subtracted from both sides, leaving

Explanation for Incorrect Answer A :  Choice (A) is not correct. See the explanation for the correct response (B).

(A)

(B)

(C)

(D)

(E)

页码,4/15The Official SAT Online Course

2006-11-12file://E:\新建文件夹\a4.htm

UnRe

gistered

严禁用于商业用途!

Explanation for Incorrect Answer C :  Choice (C) is not correct. If were substituted for then the equation would be which simplifies to So could be equal to but the equation is only true when is

Explanation for Incorrect Answer D :  Choice (D) is not correct. If were substituted for then the equation would be which simplifies to So could be equal to but the equation is only true when is

Explanation for Incorrect Answer E :  Choice (E) is not correct. If were substituted for then the equation would be which simplifies to So could be equal to but the equation is only true when is

Questions 7-9 refer to the following definition.

Let be defined by for all numbers and

7

ANSWERS AND EXPLANATIONS Explanation for Correct Answer C : 

Choice (C) is correct.

Explanation for Incorrect Answer A :  Choice (A) is not correct. but is not equal to

Explanation for Incorrect Answer B : 

Choice (B) is not correct. but is not equal to

Explanation for Incorrect Answer D :  Choice (D) is not correct. but

Explanation for Incorrect Answer E :  Choice (E) is not correct. but is not equal to

(A)

(B)

(C)

(D)

(E)

8

If then

ANSWERS AND EXPLANATIONS Explanation for Correct Answer A :  Choice (A) is correct. Since this expression is equal to it follows that Therefore, which

(A)

(B)

(C)

(D)

(E)

页码,5/15The Official SAT Online Course

2006-11-12file://E:\新建文件夹\a4.htm

UnRe

gistered

严禁用于商业用途!

gives so .

Explanation for Incorrect Answer B :  Choice (B) is not correct. If then but the problem states that

Explanation for Incorrect Answer C :  Choice (C) is not correct. If then but the problem states that

Explanation for Incorrect Answer D :  Choice (D) is not correct. If then but the problem states that

Explanation for Incorrect Answer E :  Choice (E) is not correct. If then but the problem states that

9 For what value of is the statement always true?

ANSWERS AND EXPLANATIONS Explanation for Correct Answer B :  Choice (B) is correct. Since the equation can be rewritten as Subtracting from both sides yields

or Thus, either or Therefore, the

only value of that always makes true is

Explanation for Incorrect Answer A :  Choice (A) is not correct. If then This is only equal to or in this case when but the question asks for a value of that will make the equation true for all values of

Explanation for Incorrect Answer C :  Choice (C) is not correct. If then This is only equal to or in this case when but the question asks for a value of that will make the equation true for all values of

Explanation for Incorrect Answer D :  Choice (D) is not correct. If then This is only equal to or in this case when but the question asks for a value of that will make the equation true for all values of

Explanation for Incorrect Answer E :  Choice (E) is not correct. If then This is only equal to or in this case when but the question asks for a value of that will make the equation true for all values of

(A)

(B)

(C)

(D)

(E)

10

页码,6/15The Official SAT Online Course

2006-11-12file://E:\新建文件夹\a4.htm

UnRe

gistered

严禁用于商业用途!

In the equation above, is a constant. If the roots of the equation are and what is the value of

ANSWERS AND EXPLANATIONS Explanation for Correct Answer A :  Choice (A) is correct. Recall that a root of an equation is a number that, when substituted for the variable in the equation, reduces the equation to an

identity. If and are the roots of the equation then

and Since the left-hand sides are

each equal to zero, it follows that Since is a constant, this is true for all values of not merely and

Explanation for Incorrect Answer B :  Choice (B) is not correct. See the explanation for the correct response (A).

Explanation for Incorrect Answer C :  Choice (C) is not correct. See the explanation for the correct response (A).

Explanation for Incorrect Answer D :  Choice (D) is not correct. See the explanation for the correct response (A).

Explanation for Incorrect Answer E :  Choice (E) is not correct. See the explanation for the correct response (A).

(A)

(B)

(C)

(D)

(E)

11 Which of the following represents the area, of a circle as a function of its diameter,

ANSWERS AND EXPLANATIONS Explanation for Correct Answer E :  Choice (E) is correct. The area of a circle is given by the formula where is the radius of the circle. The diameter of a circle, is twice the

radius, so Substituting for gives the equation

(A)

(B)

(C)

(D)

(E)

页码,7/15The Official SAT Online Course

2006-11-12file://E:\新建文件夹\a4.htm

UnRe

gistered

严禁用于商业用途!

Explanation for Incorrect Answer A :  Choice (A) is not correct. If is the radius of the circle, then

This is the circumference of the circle.

Explanation for Incorrect Answer B :  Choice (B) is not correct. If is the radius of the circle, then

This is twice the circumference of the circle.

Explanation for Incorrect Answer C :  Choice (C) is not correct. If is the radius of the circle, then

This is four times the area of the circle.

Explanation for Incorrect Answer D :  Choice (D) is not correct. If is the radius of the circle, then

This is twice the area of the circle.

12

   

The table above shows some values for the functions and If and are linear functions, what is the value of

ANSWERS AND EXPLANATIONS Explanation for Correct Answer C : 

Choice (C) is correct. Since is a linear function, it can be written in the

form From two of the data points in the table, you can write

two equations: and Substituting for yields the equation which simplifies to and

further to so Since it follows that

and Knowing the values of and you can

calculate Therefore,

Similarly, since is a linear function, it can be written in the form

From two of the data points in the table, you can write two

equations: and Substituting for yields the

(A)

(B)

(C)

(D)

(E)

页码,8/15The Official SAT Online Course

2006-11-12file://E:\新建文件夹\a4.htm

UnRe

gistered

严禁用于商业用途!

equation which simplifies to and further to

so Since it follows that and

Knowing the values of and you can calculate

Therefore,

Therefore,

Explanation for Incorrect Answer A :  Choice (A) is not correct. See the explanation for the correct response (C).

Explanation for Incorrect Answer B :  Choice (B) is not correct. See the explanation for the correct response (C).

Explanation for Incorrect Answer D :  Choice (D) is not correct. See the explanation for the correct response (C).

Explanation for Incorrect Answer E :  Choice (E) is not correct. See the explanation for the correct response (C).

13

The depth of a lake is the difference between the altitude at the surface and at the lowest point of the lake. If the five lakes in the graph above were listed in order from the greatest depth to the least depth, which lake would be third in the list?

ANSWERS AND EXPLANATIONS Explanation for Correct Answer D :  Choice (D) is correct. To determine the lake with the third greatest depth, you must determine the depth of each lake by subtracting the lowest point from the highest point. The depth of Lake Superior is

the depth of Michigan is the depth of Huron is

the depth of Erie is and the depth of

Ontario is Ontario is the lake with the third greatest depth, at feet.

(A) Erie

(B) Huron

(C) Michigan

(D) Ontario

(E) Superior

页码,9/15The Official SAT Online Course

2006-11-12file://E:\新建文件夹\a4.htm

UnRe

gistered

严禁用于商业用途!

Explanation for Incorrect Answer A :  Choice (A) is not correct. See the explanation for the correct response (D).

Explanation for Incorrect Answer B :  Choice (B) is not correct. See the explanation for the correct response (D).

Explanation for Incorrect Answer C :  Choice (C) is not correct. See the explanation for the correct response (D).

Explanation for Incorrect Answer E :  Choice (E) is not correct. See the explanation for the correct response (D).

14

In the figure above, and If is an integer, what is the greatest

possible value of

ANSWERS AND EXPLANATIONS Explanation for Correct Answer B :  Choice (B) is correct. Since the fact that implies

or The equation can be rewritten as Substituting for in the previous inequality yields

which can be simplified to Since is an integer,

it follows that must also be an integer, and the greatest integer that satisfies the inequality is

Explanation for Incorrect Answer A :  Choice (A) is not correct. Although is a possible value for there are greater possible values of and the question asks for the greatest possible value.

Explanation for Incorrect Answer C :  Choice (C) is not correct. If then and

However, the problem states that

Explanation for Incorrect Answer D :  Choice (D) is not correct. If then and

However, the problem states that

Explanation for Incorrect Answer E :  Choice (E) is not correct. If then and

However, the problem states that

(A)

(B)

(C)

(D)

(E)

15Molly is inches tall. At 10:00 A.M. one day, her shadow is inches long, and

页码,10/15The Official SAT Online Course

2006-11-12file://E:\新建文件夹\a4.htm

UnRe

gistered

严禁用于商业用途!

the shadow of a nearby tree is inches long. In terms of what is the height, in inches, of the tree?

ANSWERS AND EXPLANATIONS Explanation for Correct Answer D :  Choice (D) is correct. The ratio of Molly’s height ( inches) to the length of her shadow ( inches) is proportional to the ratio of the tree’s height (

) to the length of its shadow ( ). So This simplifies to

Explanation for Incorrect Answer A :  Choice (A) is not correct. Height is proportional to the length of the shadow; it is not the length of the shadow plus a constant.

Explanation for Incorrect Answer B :  Choice (B) is not correct. See the explanation for the correct response (D).

Explanation for Incorrect Answer C :  Choice (C) is not correct. The height of the tree is times the length of its

shadow, not the length of its shadow.

Explanation for Incorrect Answer E :  Choice (E) is not correct. See the explanation for the correct response (D).

(A)

(B)

(C)

(D) 4s

(E)

16If a number is chosen at random from the set what is the

probability that it is a member of the solution set of both and

ANSWERS AND EXPLANATIONS Explanation for Correct Answer C :  Choice (C) is correct. To determine this probability, you must first look at the solution sets of the inequalities. The inequality simplifies to and simplifies to Of the five numbers in the set, only

and are greater than and less than Therefore, the probability of picking a number from the set that is in the solution set of both these

inequalities is

(A)

(B)

(C)

(D)

(E)

页码,11/15The Official SAT Online Course

2006-11-12file://E:\新建文件夹\a4.htm

UnRe

gistered

严禁用于商业用途!

Explanation for Incorrect Answer A :  Choice (A) is not correct. See the explanation for the correct response (C).

Explanation for Incorrect Answer B :  Choice (B) is not correct. See the explanation for the correct response (C).

Explanation for Incorrect Answer D :  Choice (D) is not correct. See the explanation for the correct response (C).

Explanation for Incorrect Answer E :  Choice (E) is not correct. See the explanation for the correct response (C).

17 If the length of is and the length of is which of the following could be

the length of

ANSWERS AND EXPLANATIONS Explanation for Correct Answer A :  Choice (A) is correct. If and lie on a line, the greatest possible

length of is If and do not lie on the same line,

then a triangle is formed by these three points, and the length of must

be less than the sum of the lengths of and which is The only

choice that is less than or equal to is

Explanation for Incorrect Answer B :  Choice (B) is not correct. The greatest possible length of is equal to

Explanation for Incorrect Answer C :  Choice (C) is not correct. The greatest possible length of is equal to

Explanation for Incorrect Answer D :  Choice (D) is not correct. The greatest possible length of is equal to

Explanation for Incorrect Answer E :  Choice (E) is not correct. The greatest possible length of is equal to

(A)

(B)

(C)

(D)

(E)

18

页码,12/15The Official SAT Online Course

2006-11-12file://E:\新建文件夹\a4.htm

UnRe

gistered

严禁用于商业用途!

In triangle above, if and what is the area of triangle

ANSWERS AND EXPLANATIONS Explanation for Correct Answer C :  Choice (C) is correct. If the base of triangle is then its height is Substituting the values given in the problem, the area of triangle

is

Explanation for Incorrect Answer A :  Choice (A) is not correct. See the explanation for the correct response (C).

Explanation for Incorrect Answer B :  Choice (B) is not correct. This is the area of triangle

Explanation for Incorrect Answer D :  Choice (D) is not correct. This is the area of triangle

Explanation for Incorrect Answer E :  Choice (E) is not correct. See the explanation for the correct response (C).

(A)

(B)

(C)

(D)

(E)

19

If and are two different integers and the product is the square of an integer, which of the following could be equal to

ANSWERS AND EXPLANATIONS Explanation for Correct Answer D : 

Choice (D) is correct. The prime factorization of is Thus, since

is the square of an integer, where is an

integer. It follows that If then

Explanation for Incorrect Answer A : 

Choice (A) is not correct. which is not an integer.

The question asks for a value of that makes the square of an integer.

(A)

(B)

(C)

(D)

(E)

页码,13/15The Official SAT Online Course

2006-11-12file://E:\新建文件夹\a4.htm

UnRe

gistered

严禁用于商业用途!

Explanation for Incorrect Answer B : 

Choice (B) is not correct. which is not an

integer. The question asks for a value of that makes the square of an integer.

Explanation for Incorrect Answer C : 

Choice (C) is not correct. which is not an

integer. The question asks for a value of that makes the square of an integer.

Explanation for Incorrect Answer E : 

Choice (E) is not correct. which is not an

integer. The question asks for a value of that makes the square of an integer.

20

On the number line above, the tick marks are equally spaced. Which of the lettered points represents

ANSWERS AND EXPLANATIONS Explanation for Correct Answer E :  Choice (E) is correct. Regardless of the position of on the number line,

which is the average of and will always be exactly halfway

between and Since is units to the left of the point labeled

it follows that must be units to the right, or at the point labeled .

Explanation for Incorrect Answer A :  Choice (A) is not correct. See the explanation for the correct response (E).

Explanation for Incorrect Answer B :  Choice (B) is not correct. See the explanation for the correct response (E).

Explanation for Incorrect Answer C :  Choice (C) is not correct. See the explanation for the correct response (E).

Explanation for Incorrect Answer D :  Choice (D) is not correct. See the explanation for the correct response (E).

(A) A

(B) B

(C) C

(D) D

(E) E

页码,14/15The Official SAT Online Course

2006-11-12file://E:\新建文件夹\a4.htm

UnRe

gistered

严禁用于商业用途!

    

Back to Score Report  

Copyright © 2006 The College Board. All rights reserved. Privacy Policy Terms of Use Contact Us

 

页码,15/15The Official SAT Online Course

2006-11-12file://E:\新建文件夹\a4.htm

UnRe

gistered

严禁用于商业用途!

Help | Profile | My Organizer | My Bookmarks | Logout

Answers and Explanations

Test Sections

Section 1

Section 2

Section 3

Section 5

Section 6

Section 7

Section 8

Section 9

Section 10

Back to Score Report  

View Answers and Explanations     Online - Practice Test #1

1

After winning the lottery, John bought sports cars, built a mansion, and wore designer suits; however, by thus ------- his -------, he alienated his friends.

ANSWERS AND EXPLANATIONS Explanation for Correct Answer B :  Choice (B) is correct. "Flaunting" means displaying showily or pretentiously, and "prosperity" means economic well-being. If one were to insert these terms into the text, the sentence would read "After winning the lottery, John bought sports cars, built a mansion, and wore designer suits; however, by thus flaunting his prosperity, he alienated his friends." The word "thus" indicates that the missing terms will paraphrase John's actions after he won the lottery, so it makes sense to say that John's pretentious displays of wealth alienated, or aroused hostile feelings in, his friends.

Explanation for Incorrect Answer A :  Choice (A) is incorrect. "Enduring" means undergoing or tolerating, and "hardship" means misfortune. If one were to insert these terms into the text, the sentence would read "After winning the lottery, John bought sports cars, built a mansion, and wore designer suits; however, by thus enduring his hardship, he alienated his friends." It makes little sense to say that winning a lottery is a misfortune that has to be endured.

Explanation for Incorrect Answer C :  Choice (C) is incorrect. "Undermining" means weakening gradually and secretly, and "image" means impression. If one were to insert these terms into the text, the sentence would read "After winning the lottery, John bought sports cars, built a mansion, and wore designer suits; however, by thus undermining his image, he alienated his friends." It is more probable that John made these showy purchases to build, not undermine, his image.

Explanation for Incorrect Answer D :  Choice (D) is incorrect. "Calculating" means figuring out or estimating, and "successes" are things that turn out well. If one were to insert these terms into the text, the sentence would read "After winning the lottery, John bought sports cars, built a mansion, and wore designer suits; however, by thus calculating his successes, he alienated his friends." Even though, John may have calculated his successess, this probably would not have been why he alienated his friends.

Explanation for Incorrect Answer E :  Choice (E) is incorrect. "Moderating" means lessening in extent or intensity, and "consumption," in this context, means spending money. If one were to insert these terms into the text, the sentence would read "After winning the lottery, John bought sports cars, built a mansion, and wore designer suits; however, by thus moderating his consumption, he alienated his friends." Since the word "thus" indicates that the missing terms will paraphrase John's actions after he won the lottery, inserting these terms makes a completely illogical sentence. After winning the lottery, John did not spend less money; he spent so much money that he alienated his friends.

(A) enduring. . hardship

(B) flaunting . . prosperity

(C) undermining. . image

(D) calculating . . successes

(E) moderating. . consumption

页码,1/21The Official SAT Online Course

2006-11-12file://E:\新建文件夹\a5.htm

UnRe

gistered

严禁用于商业用途!

2 The study of biology, once considered the key to solving nature’s mysteries, has instead served to emphasize nature’s incredible -------.

ANSWERS AND EXPLANATIONS Explanation for Correct Answer E :  Choice (E) is correct. “Complexity” means intricacy. If one were to insert this term into the text, the sentence would read “The study of biology, once considered the key to solving nature’s mysteries, has instead served to emphasize nature’s incredible complexity.” Since the word “instead” indicates that what follows will contrast in meaning with “solving nature’s mysteries,” it makes sense to say that biology has not solved nature’s mysteries but has instead uncovered new, perhaps unexplained, intricacies.

Explanation for Incorrect Answer A :  Choice (A) is incorrect. “Tranquility” means freedom from commotion or disturbance. If one were to insert this term into the text, the sentence would read “The study of biology, once considered the key to solving nature’s mysteries, has instead served to emphasize nature’s incredible tranquility.” Since the word “instead” indicates that what follows will contrast in meaning with “solving nature’s mysteries,” it makes sense to say that biology, rather than solving nature’s mysteries, has uncovered nature’s lack of commotion or disturbance.

Explanation for Incorrect Answer B :  Choice (B) is incorrect. “Immobility” means to be motionlessness. If one were to insert this term into the text, the sentence would read “The study of biology, once considered the key to solving nature’s mysteries, has instead served to emphasize nature’s incredible immobility.” Since the word “instead” indicates that what follows will contrast in meaning with “solving nature’s mysteries,” it makes sense to say that biology, rather than solving nature’s mysteries, biology has uncovered nature’s lack of movement.

Explanation for Incorrect Answer C :  Choice (C) is incorrect. “Consistency” means uniformity. If one were to insert this term into the text, the sentence would read “The study of biology, once considered the key to solving nature’s mysteries, has instead served to emphasize nature’s incredible consistency.” Since the word “instead” indicates that what follows will contrast in meaning with “solving nature’s mysteries,” it makes sense to say that biology has uncovered nature’s uniformity, or predictability, rather than solving its mysteries.

Explanation for Incorrect Answer D :  Choice (D) is incorrect. “Desirability” means pleasing. If one were to insert this term into the text, the sentence would read “The study of biology, once considered the key to solving nature’s mysteries, has instead served to emphasize nature’s incredible desirability.” Since the word “instead” indicates that what follows will contrast in meaning with “solving nature’s mysteries,” it makes sense to say that biology has uncovered nature’s pleasing qualities rather than solving its mysteries.

(A) tranquility

(B) immobility

(C) consistency

(D) desirability

(E) complexity

3 In 1575 Venetians instituted an annual celebration to ------- the end of the ------- that had struck the city.

(A) lament . . turmoil

(B) commemorate. . plague

(C) eulogize . . pestilence

(D) hail . . prosperity

页码,2/21The Official SAT Online Course

2006-11-12file://E:\新建文件夹\a5.htm

UnRe

gistered

严禁用于商业用途!

ANSWERS AND EXPLANATIONS Explanation for Correct Answer B :  Choice (B) is correct. "Commemorate" means a remembrance, and a "plague" is an epidemic disease. If one were to insert these terms into the text, the sentence would read "In 1575 Venetians instituted an annual celebration to commemorate the end of the plague that had struck the city." Since the first missing word refers to the celebration's purpose, and the second to the event whose end was being remembered, choice (B) makes sense: it is logical to rejoice over the memory of the end of a plague.

Explanation for Incorrect Answer A :  Choice (A) is incorrect. "Lament" means to express sorrow, and "turmoil" means a state of commotion. If one were to insert these terms into the text, the sentence would read "In 1575 Venetians instituted an annual celebration to lament the end of the turmoil that had struck the city." It would not make sense for citizens to be sad about the end of a period of commotion or distress.

Explanation for Incorrect Answer C :  Choice (C) is incorrect. "Eulogize" means to praise, and "pestilence" is another word for plague. If one were to insert these terms into the text, the sentence would read "In 1575 Venetians instituted an annual celebration to eulogize the end of the pestilence that had struck the city." Although citizens would certainly be happy that a plague had come to an end, it is unlikely that they would arrange for a yearly celebration merely to "praise" the plague's end.

Explanation for Incorrect Answer D :  Choice (D) is incorrect. "Hail" means to salute, and "prosperity" means economic well-being. If one were to insert these terms into the text, the sentence would read "In 1575 Venetians instituted an annual celebration to hail the end of the prosperity that had struck the city." Venetians would be quite unlikely to salute the end of a period of prosperity.

Explanation for Incorrect Answer E :  Choice (E) is incorrect. "Solemnize" means to celebrate with dignity and gravity, and "fame" means popular acclaim or renown. If one were to insert these terms into the text, the sentence would read "In 1575 Venetians instituted an annual celebration to solemnize the end of the fame that had struck the city." Although citizens might approach an annual celebration with solemnity and gravity, it does not make sense to say that "fame" had struck the city.

(E) solemnize. . fame

4Lena Horne’s singing style is such that she can invest even the most ------- lyrics with dramatic meaning.

ANSWERS AND EXPLANATIONS Explanation for Correct Answer D :  Choice (D) is correct. "Vapid" means dull and lacking interest. If one were to insert this term into the text, the sentence would read "Lena Horne's singing style is such that she can invest even the most vapid lyrics with dramatic meaning." Since the word "even" is used as an intensifier in this sentence to indicate that the missing term describes lyrics that are lacking in dramatic meaning, choice (D) works here.

Explanation for Incorrect Answer A : 

(A) harmonious

(B) sensational

(C) impeccable

(D) vapid

(E) esteemed

页码,3/21The Official SAT Online Course

2006-11-12file://E:\新建文件夹\a5.htm

UnRe

gistered

严禁用于商业用途!

Choice (A) is incorrect. "Harmonious" means musically concordant or pleasing. If one were to insert this term into the text, the sentence would read "Lena Horne's singing style is such that she can invest even the most harmonious lyrics with dramatic meaning." It makes no sense to speak of "harmonious lyrics"; harmony has to do with music, not the words set to music.

Explanation for Incorrect Answer B :  Choice (B) is incorrect. "Sensational" means exaggeratedly dramatic. If one were to insert this term into the text, the sentence would read "Lena Horne's singing style is such that she can invest even the most sensational lyrics with dramatic meaning." Choice (B) does not work here—"sensational" lyrics are already dramatic, even excessively so.

Explanation for Incorrect Answer C :  Choice (C ) is incorrect. "Impeccable" means faultless. If one were to insert this term into the text, the sentence would read "Lena Horne's singing style is such that she can invest even the most impeccable lyrics with dramatic meaning." Since "impeccable" lyrics are by definition faultless, it is illogical that they would need to be enhanced by a singer.

Explanation for Incorrect Answer E :  Choice (E) is incorrect. "Esteemed" means highly regarded. If one were to insert this term into the text, the sentence would read "Lena Horne's singing style is such that she can invest even the most esteemed lyrics with dramatic meaning." If lyrics were held in high regard, they would already be considered of high quality and therefore would not need to be enhanced.

5A long-standing theory about the migration of green turtles was ------- by an innovative marine biologist who graciously defused potential ------- by dedicating her work to the original researcher.

ANSWERS AND EXPLANATIONS Explanation for Correct Answer E :  Choice (E) is correct. "Repudiate" means, in this context, to reject as untrue, and "acrimony" means a biting sharpness of speech or manner. If one were to insert these terms into the text, the sentence would read "A long-standing theory about the migration of green turtles was repudiated by an innovative marine biologist who graciously defused potential acrimony by dedicating her work to the original researcher." Since the first missing term refers to what the biologist did to the theory, and the second missing term relates to consequences of research conducted to reexamine the theory, choice (E) makes sense: the second researcher "defused," or made less potent, any bitterness that might arise out of her disproving the first researcher's theory by dedicating the work to the first researcher, possibly acknowledging a debt to the latter.

Explanation for Incorrect Answer A :  Choice (A) is incorrect. "To instigate" means to provoke or urge forward, and "rancor" means ill will or bitterness. If one were to insert these terms into the text, the sentence would read "A long-standing theory about the migration of green turtles was instigated by an innovative marine biologist who graciously defused potential rancor by dedicating her work to the original researcher." Although it makes sense to say that the biologist defused potential ill will with a gracious gesture, it does not make sense to say that she provoked or urged on the theory of the original researcher and that her doing so might cause bitterness.

Explanation for Incorrect Answer B :  Choice (B) is incorrect. "Renounce" means to refuse to recognize or acknowledge something further, and "approval" means acceptance of something as satisfactory. If one were to insert these terms into the text, the sentence would read "A long-standing theory about the migration of green turtles was renounced by an innovative marine biologist who graciously defused potential approval by dedicating

(A) instigated . . rancor

(B) renounced. . approval

(C) displaced . . attribution

(D) enhanced . . alteration

(E) repudiated . . acrimony

页码,4/21The Official SAT Online Course

2006-11-12file://E:\新建文件夹\a5.htm

UnRe

gistered

严禁用于商业用途!

her work to the original "researcher." Although a theory might be renounced by a forward-thinking biologist, it does not make sense to say that she avoided potential approval, since it would be highly illogical to do so, especially with a gracious gesture.

Explanation for Incorrect Answer C :  Choice (C ) is incorrect. "Displace" means to remove or to banish, and "attribution" means the ascribing of a work to an author. If one were to insert these terms into the text, the sentence would read "A long-standing theory about the migration of green turtles was displaced by an innovative marine biologist who graciously defused potential attribution by dedicating her work to the original researcher." Although a theory might conceivably be displaced, or put aside, by an improved theory, it makes no sense to speak of "defusing," or disarming, attribution of authorship.

Explanation for Incorrect Answer D :  Choice (D) is incorrect. "Enhance" means to heighten or increase, and "alteration" means modification. If one were to insert these terms into the text, the sentence would read "A long-standing theory about the migration of green turtles was enhanced by an innovative marine biologist who graciously defused potential alteration by dedicating her work to the original researcher." Although a theory might well be improved upon by new discoveries, it makes no sense to speak of potential alteration, being defused

      On the morning of June 13, 1998, a 4.6-billion-year-old

 extraterrestrial object streaked into Earth’s atmosphere and

 blew to pieces in the sky somewhere in the neighborhood

Lineof Nelda Wallace’s backyard. A dark basketball-size object

5dropped with a loud ssshhht into Wallace’s garden, and

 fragments pelted other properties—only the first of many

 strange things soon to occur in town. For meteorites are

 more than just stars of science-fiction movies. Scientists

 covet them, private dealers scoop them up for resale at

10spiraling prices, and professional searchers travel the

 world to hunt them down. Nelda Wallace’s town was

 about to be invaded by meteorite dealers, meteorite

 fans, meteorite poachers, and other alien life-forms.

6The sentence in lines 1-4 (“On the morning . . . backyard”) is best characterized as

(A) ironic

(B) dramatic

(C) comical

(D) nostalgic

(E) celebratory

页码,5/21The Official SAT Online Course

2006-11-12file://E:\新建文件夹\a5.htm

UnRe

gistered

严禁用于商业用途!

ANSWERS AND EXPLANATIONS Explanation for Correct Answer B :  Choice (B) is correct. This sentence describes an extraordinary event, emphasizing its dramatic nature with terms like "streaked" and "blew to pieces." The sentence is a forceful description of a spectacular event.

Explanation for Incorrect Answer A :  Choice (A) is incorrect. There is no irony in this sentence; it is a description of an event.

Explanation for Incorrect Answer C :  Choice (C) is incorrect. The language in this sentence is straightforward, not humorous.

Explanation for Incorrect Answer D :  Choice (D) is incorrect. Nostalgia is about remembering something in the past. This sentence describes how an event occurred, not how people felt when remembering it.

Explanation for Incorrect Answer E :  Choice (E) is incorrect. The event may be something to celebrate, but the sentence describes the event objectively.

7 The reference to the “alien life-forms” (line 13) primarily serves to

ANSWERS AND EXPLANATIONS Explanation for Correct Answer E :  Choice (E) is correct. The sentence is intended to mock gently the circuslike atmosphere coming soon to Nelda Wallace's town. The author labels the unusual visitors to the normally quiet town as "alien life-forms" to describe humorously the types of people who are interested in meteorites.

Explanation for Incorrect Answer A :  Choice (A) is incorrect. There is nothing menacing or suggestive of danger in the paragraph. "Alien life-forms" refers to human visitors, not to dangerous creatures.

Explanation for Incorrect Answer B :  Choice (B) is incorrect. The subtle mockery here is of those who will invade the town in search of meteorites, not of the general public.

Explanation for Incorrect Answer C :  Choice (C ) is incorrect. The passage does not discuss the realibility of information about meteorites.

Explanation for Incorrect Answer D :  Choice (D) is incorrect. The author of this paragraph clearly understands what occurs when metorites fall to earth.

(A) hint at the dangers posed by some unexpected visitors

(B) mock the public’s fascination with extraterrestrial beings

(C) indicate the dearth of reliable information about a subject

(D) acknowledge a lack of familiarity with a scientific phenomenon

(E) provide a humorous label for a certain kind of zealotry

      Apes raised by humans seem to pretend more frequently

页码,6/21The Official SAT Online Course

2006-11-12file://E:\新建文件夹\a5.htm

UnRe

gistered

严禁用于商业用途!

 than do apes in the wild. Animal handlers see behaviors

 they interpret as pretending practically every day. But Anne

LineRusson, a psychologist, says she has found only about 20

5recorded cases of possible pretending in free-ranging

 orangutans, culled from thousands of hours of observation.

 One possible reason, she noted in an e-mail interview from

 her field station in Borneo, is that researchers have not been

 looking for such behavior. But many researchers believe

10that interaction with humans—and the encouragement to

 pretend that comes with it—may play a major role in why

  domesticated apes playact more.

8 Russon’s hypothesis would be most fully tested by which possible research project?

ANSWERS AND EXPLANATIONS Explanation for Correct Answer B :  Choice (B) is correct. Researchers had not been studying pretending behavior in free-ranging orangutans; if they had done so, they would have had data to compare with the data about orangutans raised by humans.

Explanation for Incorrect Answer A :  Choice (A) is incorrect. Orangutans are the only type of ape Russon has studied; research on other apes would not be relevant.

Explanation for Incorrect Answer C :  Choice (C ) is incorrect. Lines 1-3 make it clear that most research on playacting behaviors has focused on orangutans raised by humans, not on wild orangutans.

Explanation for Incorrect Answer D :  Choice (D) is incorrect. Only research on the free-ranging orangutans is needed to fully test the hypothesis.

Explanation for Incorrect Answer E :  Choice (E) is incorrect. When such behavior was first recorded is not the issue; the frequency of such behavior is the concern.

(A) Examining data from observations of pretending behavior in apes other than orangutans

(B) Expanding ongoing observations of orangutans to include pretending behavior

(C) Documenting pretending behavior among orangutans raised by humans

(D) Comparing specific pretending behaviors in free-ranging and domesticated orangutans

(E) Reviewing existing data on free-ranging orangutans to determine the earliest record of pretending behavior

9Which theoretical statement about pretending behavior in apes would be supported

页码,7/21The Official SAT Online Course

2006-11-12file://E:\新建文件夹\a5.htm

UnRe

gistered

严禁用于商业用途!

most fully by the “many researchers” mentioned in line 9?

ANSWERS AND EXPLANATIONS Explanation for Correct Answer E :  Choice (E) is correct. The implication of the paragraph's last sentence is that many researchers feel that contact with humans determines the frequency of pretending behavior in orangutans.

Explanation for Incorrect Answer A :  Choice (A) is incorrect. Performing might be a consequence of pretending behavior, but it is not an explanation of that behavior.

Explanation for Incorrect Answer B :  Choice (B) is incorrect. The researchers think that pretending behavior is learned from humans, so they would not expect wild orangutans to demonstrate it.

Explanation for Incorrect Answer C :  Choice (C) is incorrect. Geographical or demographic influences are not discussed in the passage.

Explanation for Incorrect Answer D :  Choice (D) is incorrect. The researchers in the passage are not assuming that the apes' behavior has been misinterpreted; instead, they are arguing that it is real and has been learned from humans.

(A) Having the ability to pretend has enabled apes, such as chimpanzees, to be trained as performers.

(B) All types of apes, both wild and domesticated, can pretend with human companions.

(C) Pretending behavior for wild apes may vary considerably by region and population.

(D) Handlers of domesticated apes do not always have the rigorous observational training of scientists.

(E) Wild apes living apart from humans pretend only rarely.

This excerpt is from a short story by a Japanese American writer. The narrator reflects on her family’s past as she helps her mother prepare to move from her home.

      There’s a photograph of my mother standing on the pier

 in Honolulu in 1932, the year she left Hawaii to attend the

 University of California. She’s loaded to the ears with leis.

Line She’s wearing a fedora1 pulled smartly to the side. She is

5not smiling. Of my mother’s two years at the university,

 my grandmother recalled that she received good grades

 and never wore a kimono again. My second cousin, with

 whom my mother stayed when she first arrived, said she

 was surprisingly sophisticated—she liked hats. My mother

10 said that she was homesick. Her favorite class

页码,8/21The Official SAT Online Course

2006-11-12file://E:\新建文件夹\a5.htm

UnRe

gistered

严禁用于商业用途!

was biology

 and she entertained thoughts of becoming a scientist. Her

 father, however, wanted her to become a teacher, and his

 wishes prevailed, even though he would not have forced

  them upon her. She was a dutiful daughter.

15     During her second year, she lived near campus with a

 mathematics professor and his wife. In exchange for room

 and board she cleaned house, ironed, and helped prepare

 meals. One of the things that survives from this period is a

 black composition book entitled Recipes of California. As

20a child, I read it like a book of mysteries for clues to a life

 both alien and familiar. Some entries she had copied by

 hand; others she cut out of magazines and pasted on the

 page, sometimes with a picture or drawing. The margins

 contained her cryptic comments: “Saturday bridge club,”

25 “From Mary G. Do not give away.”

      That book holds part of the answer to why our family

 rituals didn’t fit the norm either of our relatives or of the

 larger community in which we grew up. At home, we ate

 in fear of the glass of spilled milk, the stray elbow on the

30table, the boarding house reach. At my grandparents’, we

  slurped our chasuke2. We wore tailored dresses and black

 shoes with white socks; however, what we longed for were

 the lacy colorful dresses that other girls wore to church on

 Sunday. For six years, I marched to Japanese language

35school after my regular classes; however, we only spoke

 English at home. We talked too loudly and all at once,

页码,9/21The Official SAT Online Course

2006-11-12file://E:\新建文件夹\a5.htm

UnRe

gistered

严禁用于商业用途!

 which mortified my mother, but she was always complaining

 about Japanese indirectness. I know that she smarted under

 a system in which the older son is the center of the familial

40universe, but at thirteen I had a fit of jealous rage over her

  fawning attention to our only male cousin.

      My sister has found a photograph of our mother, a

 round-faced and serious twelve or thirteen, dressed in a

 kimono and seated, on her knees, on the tatami mat. She is

45playing the koto, a difficult stringed instrument thought to

 teach girls discipline. Of course, everything Japanese was

 a lesson in discipline—flower arranging, embroidery,

 everything. One summer my sister and I had to take

 ikebana, the art of flower arrangement, at our grandfather’s

50school. The course was taught by Mrs. Oshima, a soft-

 spoken, terrifying woman, and my supplies were provided

 by my grandmother, whose tastes ran to the oversized.

 I remember little of that class and its principles. What I

 remember most clearly is having to walk home carrying

55one of our creations, which, more often than not, towered

  above our heads.

      How do we choose among what we experience, what

 we are taught, what we run into by chance, or what is

 forced upon us? What is the principle of selection? My

60sisters and I are not bound by any of our mother’s obli-

 gations, nor do we follow the rituals that seemed so

 important. My sister once asked, do you realize that when

 she’s gone that’s it? She was talking about how to make

页码,10/21The Official SAT Online Course

2006-11-12file://E:\新建文件夹\a5.htm

UnRe

gistered

严禁用于商业用途!

  sushi3, but it was a more profound question

nonetheless.

 

1 A fedora is a soft felt hat popular in the United States in the 1930’s. 2 Chasuke is a rice and tea mixture. 3 Sushi is cold rice shaped into small cakes and sometimes topped or wrapped with garnishes.

10 The thematic focus of the passage is on the

ANSWERS AND EXPLANATIONS Explanation for Correct Answer B :  Choice (B) is correct. The passage offers examples of many American and Japanese values and practices that must be balanced: table manners, clothing, and speech are among the areas considered.

Explanation for Incorrect Answer A :  Choice (A) is incorrect. Although the narrator is aware of different standards of behavior in the two households, no conflict is ever mentioned.

Explanation for Incorrect Answer C :  Choice (C) is incorrect. Although examples in the passage do illustrate some assimilation of the narrator’s mother, the narrator does not discuss any immigrants beyond her own family.

Explanation for Incorrect Answer D :  Choice (D) is incorrect. Although the narrator gives examples of Japanese traditions valued by her grandmother and mother, the passage focuses on the challenge of change rather than the desirability of maintaining traditions.

Explanation for Incorrect Answer E :  Choice (E) is incorrect. The passage does address both traditional customs and modern society, but the issue of irrelevance is never raised.

(A) conflicts between the narrator’s mother and grandmother

(B) challenge of balancing conflicting values and practices

(C) widespread assimilation of immigrants into the culture of the United States

(D) desirability of maintaining traditions

(E) irrelevance of traditional customs to modern society

11 The grandmother’s comments in lines 5-7 imply that her daughter’s experiences at the university were characterized by

ANSWERS AND EXPLANATIONS

(A) success and camaraderie

(B) accomplishment and assimilation

(C) enlightenment and introspection

(D) diligence and homesickness

(E) scholarship and competition

页码,11/21The Official SAT Online Course

2006-11-12file://E:\新建文件夹\a5.htm

UnRe

gistered

严禁用于商业用途!

Explanation for Correct Answer B :  Choice (B) is correct. The fact that the daughter earns good grades shows her academic accomplishment; that she never wears a kimono again shows that in her manner of dress she is assimilated into American culture.

Explanation for Incorrect Answer A :  Choice (A) is incorrect. While the daughter does succeed in "[getting] good grades," no mention is made of friends or companions, which would evidence "camaraderie."

Explanation for Incorrect Answer C :  Choice (C) is incorrect. The grandmother’s comments about grades and dress do not touch on either "enlightenment" (spiritual insight) or "introspection" (self-examination).

Explanation for Incorrect Answer D :  Choice (D) is incorrect. Although "good grades" may have been a result of "diligence," or hard work, the grandmother’s comments do not address homesickness.

Explanation for Incorrect Answer E :  Choice (E) is incorrect. No competition with, for instance, other students is necessarily suggested by the daughter’s "good grades," though scholarship may have helped produce them.

12 In line 11, the word "entertained" most nearly means

ANSWERS AND EXPLANATIONS Explanation for Correct Answer D :  Choice (D) is correct. The act of "entertaining" thoughts involves "harboring," or holding them in the mind.

Explanation for Incorrect Answer A :  Choice (A) is incorrect. To "regale" someone is to provide him or her with great enjoyment; it has nothing to do with holding thoughts in the mind.

Explanation for Incorrect Answer B :  Choice (B) is incorrect. In this context, the word "entertained" has to do with thought, not with entertaining people.

Explanation for Incorrect Answer C :  Choice (C) is incorrect. The thoughts that the daughter entertains remain internal and cannot therefore be "flaunted," or shown off.

Explanation for Incorrect Answer E :  Choice (E) is incorrect. Although the word "entertained" means that the daughter thought about being a biologist, it does not suggest that she took great pleasure in this.

(A) regaled

(B) hosted

(C) flaunted

(D) harbored

(E) welcomed

13 The narrator’s statement in line 14 (“She . . . daughter”) serves to

(A) defend her mother’s interest in science

(B) justify her mother’s decision to leave home

页码,12/21The Official SAT Online Course

2006-11-12file://E:\新建文件夹\a5.htm

UnRe

gistered

严禁用于商业用途!

ANSWERS AND EXPLANATIONS Explanation for Correct Answer C :  Choice (C) is correct. The narrator’s mother becomes a teacher because “her father wanted it” and because she allowed “his wishes to prevail.”

Explanation for Incorrect Answer A :  Choice (A) is incorrect. The narrator’s statement that her mother was “a dutiful daughter” emphasizes the mother’s willingness to put aside her interest in science and accept her father’s plans for her.

Explanation for Incorrect Answer B :  Choice (B) is incorrect. The narrator’s comment is made about a period of time after her mother had already left home for college.

Explanation for Incorrect Answer D :  Choice (D) is incorrect. Although the narrator’s mother is a “dutiful daughter” to her father, there is no suggestion that there is a tradition of her always acceding to his demands.

Explanation for Incorrect Answer E :  Choice (E) is incorrect. The narrator simply describes her mother’s response to her grandfather’s wishes; she does not describe doing anything.

(C) explain why her mother became a teacher

(D) question the relevance of established customs

(E) rationalize her grandfather’s actions

14

The narrator suggests that as a child she read her mother’s book of recipes in order to

ANSWERS AND EXPLANATIONS Explanation for Correct Answer B :  Choice (B) is correct. The narrator reads the book in hopes of understanding her mother, who is mysterious to her.

Explanation for Incorrect Answer A :  Choice (A) is incorrect. The narrator does not doubt her mother's loyalty to the family; rather, she finds her puzzling as a person.

Explanation for Incorrect Answer C :  Choice (C) is incorrect. The narrator does not say that she had fantasies about her mother.

Explanation for Incorrect Answer D :  Choice (D) is incorrect. The narrator does not suggest that her mother is unwilling to talk about the past.

Explanation for Incorrect Answer E :  Choice (E) is incorrect. The narrator’s mother was homesick during her early time in California, but the narrator offers no evidence of any unhappiness at the time she read her mother’s recipe book.

(A) seek proof of her mother’s devotion to the family

(B) understand more fully the contradictions in her mother’s behavior

(C) perpetuate the fantasy she created about her mother

(D) search for clues to her mother’s reluctance to discuss her past

(E) discover the cause of her mother’s unhappiness

页码,13/21The Official SAT Online Course

2006-11-12file://E:\新建文件夹\a5.htm

UnRe

gistered

严禁用于商业用途!

15 The description of the reaction of the mother to her children’s manner of speaking (lines 36-38) highlights how she

ANSWERS AND EXPLANATIONS Explanation for Correct Answer C :  Choice (C) is correct. Despite her complaints about "Japanese indirectness," the mother is still embarrassed by her children’s loud voices and tendency to talk "all at once."

Explanation for Incorrect Answer A :  Choice (A) is incorrect. The children are loud and speak "all at once," but they are not naïve. The mother is "mortified" by their volume and manners but not fearful for them.

Explanation for Incorrect Answer B :  Choice (B) is incorrect. Although the mother complains about "Japanese indirectness," she does not express any dislike for the language lessons.

Explanation for Incorrect Answer D :  Choice (D) is incorrect. Only English is spoken in the narrator’s home, and the passage describes the mother as having mixed feelings about Japanese customs.

Explanation for Incorrect Answer E :  Choice (E) is incorrect. The mother’s concern about the children’s way of speaking has to do with their volume and their talking simultaneously, not with their Japanese language skills.

(A) feared that her children’s naïveté would invite trouble

(B) shared her children’s distaste for Japanese language lessons

(C) was still imbued with the lessons of her culture

(D) insisted on maintaining a strong Japanese influence in her home

(E) wanted her children to be fluent in the Japanese language

16 The narrator repeats the word "everything" in lines 46-48 to

ANSWERS AND EXPLANATIONS Explanation for Correct Answer D :  Choice (D) is correct. The repetition of "everything" in a list of occupations requiring great control calls attention to the pervasiveness of discipline in her mother’s Japanese upbringing.

Explanation for Incorrect Answer A :  Choice (A) is incorrect. Although the sisters both studied ikebana, the narrator is concerned with discipline, not competition.

Explanation for Incorrect Answer B :  Choice (B) is incorrect. The narrator lists representative activities not to show their variety but to illustrate the range of behaviors that require discipline.

Explanation for Incorrect Answer C : 

(A) explain the intensity of her competition with her sister

(B) characterize the diverse achievements of Japanese Americans

(C) describe her mother’s single-minded pursuit of perfection

(D) emphasize the extent to which discipline governed Japanese life

(E) highlight the extraordinary skill required to master the koto

页码,14/21The Official SAT Online Course

2006-11-12file://E:\新建文件夹\a5.htm

UnRe

gistered

严禁用于商业用途!

Choice (C) is incorrect. The narrator is interested in the ways discipline pervaded her mother’s life, not suggesting that her mother strove for perfection.

Explanation for Incorrect Answer E :  Choice (E) is incorrect. Although the narrator mentions that the koto was "thought to teach girls discipline," she repeats "everything" to emphasize the extent to which discipline was required in other activities as well.

17 In lines 53-56, the narrator’s description of childhood walks home from ikebana class conveys a sense of

ANSWERS AND EXPLANATIONS Explanation for Correct Answer D :  Choice (D) is correct. The picture of young girls dwarfed by flower arrangements suggests a sense of "absurdity," or ridiculous incongruity.

Explanation for Incorrect Answer A :  Choice (A) is incorrect. The description does not indicate any sense of excitement or "adventure."

Explanation for Incorrect Answer B :  Choice (B) is incorrect. The narrator would not have been relieved to be walking home with a flower arrangement that "more often than not, towered above" her head.

Explanation for Incorrect Answer C :  Choice (C) is incorrect. The description offers a picture that is odd but not sad.

Explanation for Incorrect Answer E :  Choice (E) is incorrect. The description of the girls and the flower arrangements does not carry any particular sense of "vitality," or energy.

(A) adventure

(B) relief

(C) melancholy

(D) absurdity

(E) vitality

18To the narrator, her sister’s question (lines 62-63) implies that the

ANSWERS AND EXPLANATIONS Explanation for Correct Answer A :  Choice (A) is correct. The narrator understands her sister’s question to refer to the making of sushi as well as to every other aspect of their Japanese heritage.

Explanation for Incorrect Answer B :  Choice (B) is incorrect. The rest of the passage makes clear the ways the mother has made her daughters aware of their heritage and background.

(A) mother represented the last true vestige of the sisters’ Japanese heritage

(B) mother should have made more of an effort to educate her daughters about their background

(C) mother’s education in California extended beyond the confines of the university

(D) sisters were saddened by their mother’s decision to move

(E) sisters would not regret the absence of traditional family rituals

页码,15/21The Official SAT Online Course

2006-11-12file://E:\新建文件夹\a5.htm

UnRe

gistered

严禁用于商业用途!

Explanation for Incorrect Answer C :  Choice (C) is incorrect. The question refers to the end of cultural influence, not to the scope of the mother’s education.

Explanation for Incorrect Answer D :  Choice (D) is incorrect. The sister asked the question sometime in the past, presumably before the mother thought of moving.

Explanation for Incorrect Answer E :  Choice (E) is incorrect. Though the sisters do not follow the traditional customs themselves, the question strongly suggests that they will miss such customs.

The following passage is excerpted from a historian’s examination of European attitudes toward childhood.

      Medieval European art until about the twelfth century

 did not know childhood or did not attempt to portray it.

 It is hard to believe that this neglect was due to incompe-

Linetence or incapacity; it seems more probable that there was

5no place for childhood in the medieval world. A miniature

 painted during the twelfth century provides us with a

 striking example of the deformity that an artist at that time

 would inflict on the representation of children’s bodies.

 The subject is a Biblical scene in which Jesus is surrounded

10by little children. Yet the miniaturist has grouped around

 Jesus what are obviously eight men, without any charac-

 teristics of childhood; they have simply been depicted on

 a smaller scale. In a French miniature of the late eleventh

 century, three children brought to life by a saint are also

15reduced to a smaller scale than the adults, without any

 other difference in expression or features. A painter would

 not hesitate to give the body of a child the musculature of

  an adult.

      In the world of pictorial formulas inherited from

20ancient Rome, right up to the end of the thirteenth century,

页码,16/21The Official SAT Online Course

2006-11-12file://E:\新建文件夹\a5.htm

UnRe

gistered

严禁用于商业用途!

 there are no children characterized by a special expression,

 but only adults on a reduced scale. This refusal to accept

  child morphology * in art is to be found too in

most of the

 ancient civilizations. A fine Sardinian bronze of the ninth

25century B.C. shows a mother holding in her arms the bulky

 body of her son. The museum catalog tells us: “the little

 masculine figure could also be a child which, in accor-

 dance with the formula adopted in ancient times by other

 peoples, had been represented as an adult.” Everything in

30fact would seem to suggest that the realistic representation

 of children or the idealization of childhood was confined

 to ancient Greek art. Representations of Eros, the Greek

 child god of love, proliferated in that Hellenistic period,

 but childhood disappeared from art together with the other

35Hellenistic themes, and the subsequent Romanesque art

 returned to the rejection of the special features of

  childhood.

      This is no mere coincidence. Our starting point in

 this study is a world of pictorial representation in which

40childhood is unknown; literary historians such as Calvé

 have made the same observation about the medieval epic,

 in which child prodigies behave with the courage and

 physical strength of doughty warriors. This undoubtedly

 meant that the people of the tenth and eleventh centuries

45did not dwell on the image of childhood and that the

 image had neither interest nor even reality for them. It

页码,17/21The Official SAT Online Course

2006-11-12file://E:\新建文件夹\a5.htm

UnRe

gistered

严禁用于商业用途!

 suggests too that in the realm of real life, and not simply

 in that of aesthetic translation, childhood was a period of

 transition that passed quickly and that was just as quickly

50 forgotten.

 

* Structure and form

19 The first two paragraphs (lines 1-37) primarily serve to

ANSWERS AND EXPLANATIONS Explanation for Correct Answer C :  Choice (C) is correct. The first two paragraphs offer several examples in support of the claim that medieval European art did not accurately portray children's specific physical characteristics.

Explanation for Incorrect Answer A :  Choice (A) is incorrect. Lines 1-37 do not argue that children should not be portrayed; rather, they describe how they are portrayed.

Explanation for Incorrect Answer B :  Choice (B) is incorrect. Although the second paragraph suggests that medieval art was conservative in that it preserved an old tradition, the main focus of the two paragraphs is on representations of children.

Explanation for Incorrect Answer D :  Choice (D) is incorrect. The first two paragraphs primarily supply examples of unrealistic representations. The second paragraph mentions a period in Greek art when realistic representation of children flourished, but it makes clear that this period was followed by a return to lack of realism.

Explanation for Incorrect Answer E :  Choice (E) is incorrect. The purpose of the first two paragraphs is not to address the themes of medieval art but to address the lack of accurate representations of childhood. Possible theories about this lack of realistic childhood imagery are put forward only in the third paragraph.

(A) argue against the depiction of children in artwork

(B) suggest that medieval Western art was particularly conservative

(C) describe the unrealistic portrayal of children in medieval art

(D) trace the evolution of realistic representation in Western art

(E) postulate a theory about the thematic focuses of medieval Western art

20The author’s argument about the depiction of children in medieval art assumes that the depictions

ANSWERS AND EXPLANATIONS

(A) suggest the connection between medieval art and religion

(B) prefigure the gradual shift to realism

(C) are too varied to support any one argument

(D) reflect earlier civilizations’ corruption

(E) offer an indication of commonly held attitudes

页码,18/21The Official SAT Online Course

2006-11-12file://E:\新建文件夹\a5.htm

UnRe

gistered

严禁用于商业用途!

Explanation for Correct Answer E :  Choice (E) is correct. The author's argument assumes that inaccurate depictions of children in medieval art are a direct result of the era's widespread lack of interest in the nature of children or in childhood images.

Explanation for Incorrect Answer A :  Choice (A) is incorrect. The author is not concerned with the connection between religion and medieval art. The passage mentions Biblical scenes in art only to examine the way in which children are depicted in them.

Explanation for Incorrect Answer B :  Choice (B) is incorrect. The author is concerned not with tracing the evolution of artistic depictions of childhood but with indicating a direct connection between artistic interpretations of children and real-life medieval attitudes toward them.

Explanation for Incorrect Answer C :  Choice (C) is incorrect. The basis of the author's argument is that depictions of children in medieval art were not varied but were almost uniformly inaccurate.

Explanation for Incorrect Answer D :  Choice (D) is incorrect. Although the passage mentions childhood images that date to the ninth century B.C., there is no indication that the author views these earlier cultures as corrupt.

21 The author’s argument is developed primarily by

ANSWERS AND EXPLANATIONS Explanation for Correct Answer B :  Choice (B) is correct. Most of the passage is devoted to examples of depictions of children in medieval paintings.

Explanation for Incorrect Answer A :  Choice (A) is incorrect. The author doesn't quote any literary sources and only mentions in passing one literary historian.

Explanation for Incorrect Answer C :  Choice (C) is incorrect. Although the passage offers a fleeting sociological analysis of medieval people—that childhood images "had neither interest nor even reality for them"—the author does not provide any analyses of medieval artists.

Explanation for Incorrect Answer D :  Choice (D) is incorrect. The passage does not mention modern images of the body; it discusses only medieval and ancient images of children's bodies.

Explanation for Incorrect Answer E :  Choice (E) is incorrect. The passage does not consider the nature of childhood; rather, it infers from the way children are depicted in medieval art that medieval Europeans did not consider childhood to have a philosophical nature worth reflecting upon.

(A) quotations from literary sources

(B) descriptions of visual evidence

(C) psychological analyses of medieval artists

(D) comparisons of modern and medieval images of the body

(E) reflections on the philosophical nature of childhood

22 The last sentence of the passage (lines 46-50) primarily serves to

(A) define an important term that is central to the author’s argument

页码,19/21The Official SAT Online Course

2006-11-12file://E:\新建文件夹\a5.htm

UnRe

gistered

严禁用于商业用途!

ANSWERS AND EXPLANATIONS Explanation for Correct Answer C :  Choice (C) is correct. The last sentence offers an explanation for why children were so rarely accurately depicted in medieval art—namely, because childhood was considered a forgettable state that did not merit accurate depiction.

Explanation for Incorrect Answer A :  Choice (A) is incorrect. The last sentence does not define any of the terms of the argument; it suggests an explanation for the examples of childhood depictions listed earlier in the passage.

Explanation for Incorrect Answer B :  Choice (B) is incorrect. The last sentence expands on the author's thesis that images of childhood held no interest for people of the tenth and eleventh centuries, but no objections to this thesis are indicated in the passage.

Explanation for Incorrect Answer D :  Choice (D) is incorrect. The last sentence does not introduce any examples; it provides an explanation for the examples of inaccurate childhood depictions in art introduced earlier in the passage.

Explanation for Incorrect Answer E :  Choice (E) is incorrect. The historian Calvé is mentioned earlier in the same paragraph; however, neither his nor any other historian's views are discussed in the last sentence, which attempts to explain why childhood was virtually ignored in medieval art.

(B) dismiss objections to the author’s thesis

(C) provide an explanation for the phenomenon discussed in the previous paragraphs

(D) introduce examples from other time periods and other forms of representational art

(E) summarize the views of other historians of medieval art

23In line 48, “translation” most nearly means

ANSWERS AND EXPLANATIONS Explanation for Correct Answer C :  Choice (C) is correct. In line 48, "translation" refers to the "representation," or artistic depiction, of childhood.

Explanation for Incorrect Answer A :  Choice (A) is incorrect. In line 48, "translation" refers not to a "substitution" for life but to a rendering of life that is unrealistic.

Explanation for Incorrect Answer B :  Choice (B) is incorrect. "Explanation" is not a possible meaning in the context of line 48, since the "translation" doesn't attempt to explain or even recognize childhood.

Explanation for Incorrect Answer D :  Choice (D) is incorrect. In the context of line 48, "translation" is not used to mean

(A) substitution

(B) explanation

(C) representation

(D) transportation

(E) correction

页码,20/21The Official SAT Online Course

2006-11-12file://E:\新建文件夹\a5.htm

UnRe

gistered

严禁用于商业用途!

something that "transports" its viewers.

Explanation for Incorrect Answer E :  Choice (E) is incorrect. The "translation" discussed in line 48 is not a "correction" of real life as much as it is a rendering of it.

24 The author offers which explanation for the way that medieval painters depicted children?

ANSWERS AND EXPLANATIONS Explanation for Correct Answer D :  Choice (D) is correct. The passage indicates that "childhood was a period of transition" that was seen not as a separate phase but as a quickly forgotten period of pre-adulthood.

Explanation for Incorrect Answer A :  Choice (A) is incorrect. There is no mention in the passage of artists' models, either children or adults.

Explanation for Incorrect Answer B :  Choice (B) is incorrect. Children may well have been more difficult to paint than adults, but the passage indicates that "incompetence or incapacity" (lines 3-4) on the part of an artist was not a likely explanation for the absence of accurate childhood depictions.

Explanation for Incorrect Answer C :  Choice (C) is incorrect. The passage points out that "representations of Eros, the Greek child god of love," were plentiful in ancient Greek paintings.

Explanation for Incorrect Answer E :  Choice (E) is incorrect. According to the passage, one medieval painting was a depiction of Jesus surrounded by small children, indicating that childhood was indeed recognized in medieval theology (even if painters depicted children as small adults).

(A) Children were discouraged from becoming artists’ models.

(B) Children were more difficult to paint than adults.

(C) Children had never been a subject of art in Western traditions.

(D) Childhood was not understood as a separate phase of life.

(E) Childhood was not recognized in medieval theology.

    

Back to Score Report  

Copyright © 2006 The College Board. All rights reserved. Privacy Policy Terms of Use Contact Us

 

页码,21/21The Official SAT Online Course

2006-11-12file://E:\新建文件夹\a5.htm

UnRe

gistered

严禁用于商业用途!

Help | Profile | My Organizer | My Bookmarks | Logout

Answers and Explanations

Test Sections

Section 1

Section 2

Section 3

Section 5

Section 6

Section 7

Section 8

Section 9

Section 10

Back to Score Report  

View Answers and Explanations     Online - Practice Test #1

1 Which of the following triples does NOT satisfy the equation

ANSWERS AND EXPLANATIONS Explanation for Correct Answer E : 

Choice (E) is correct. The triple results in

Therefore, this triple does not satisfy the equation.

Explanation for Incorrect Answer A : 

Choice (A) is not correct. The triple results in

However, the question asks for a triple that does not

satisfy the equation.

Explanation for Incorrect Answer B : 

Choice (B) is not correct. The triple results in

However, the question asks for a triple that does not

satisfy the equation.

Explanation for Incorrect Answer C : 

Choice (C) is not correct. The triple results in However,

the question asks for a triple that does not satisfy the equation.

Explanation for Incorrect Answer D : 

Choice (D) is not correct. The triple results in

However, the question asks for a triple that does not satisfy the equation.

(A)

(B)

(C)

(D)

(E)

2 An amusement park charges more for an adult’s admission than for a child’s

admission. If a group of adults and children spent on admission, what is the price of admission for one child?

ANSWERS AND EXPLANATIONS

(A)

(B)

(C)

(D)

(E)

页码,1/9The Official SAT Online Course

2006-11-12file://E:\新建文件夹\a6.htm

UnRe

gistered

严禁用于商业用途!

Explanation for Correct Answer B :  Choice (B) is correct. Let be the price of admission, in dollars, for a child. The price of admission for an adult is then Since the total cost of admission for

adults and children is it follows that This

simplifies to Thus, and

Explanation for Incorrect Answer A :  Choice (A) is not correct. See the explanation for the correct response (B).

Explanation for Incorrect Answer C :  Choice (C) is not correct. See the explanation for the correct response (B).

Explanation for Incorrect Answer D :  Choice (D) is not correct. See the explanation for the correct response (B).

Explanation for Incorrect Answer E :  Choice (E) is not correct. See the explanation for the correct response (B).

3

The figure above shows four apartments in a building. In this building, each apartment is occupied by only one person. Alice lives next to Sam, and Paul lives next to Alice and Dara. In which apartment could Alice live?

ANSWERS AND EXPLANATIONS Explanation for Correct Answer D :  Choice (D) is correct. It is given that Alice lives next to Sam. From the fact that Paul lives next to Alice, you can conclude that Alice must live in an apartment that has neighbors on both sides (Sam and Paul do not live in the same apartment). The only two apartments that satisfy this condition are apartments 2 and 3. Both of these apartments are possible, because the following two arrangements meet all the conditions given in the problem. Sam could live in apartment 1, Alice in apartment 2, Paul in apartment 3, and Dara in apartment 4. Dara could live in apartment 1, Paul in apartment 2, Alice in apartment 3, and Sam in apartment 4.

Explanation for Incorrect Answer A :  Choice (A) is not correct. Apartment 1 only has one neighboring apartment, and Alice must live in an apartment with two neighboring apartments.

Explanation for Incorrect Answer B :  Choice (B) is not correct. While apartment 2 is a valid answer, apartment 3 is also a valid answer.

Explanation for Incorrect Answer C :  Choice (C) is not correct. While apartment 3 is a valid answer, apartment 2 is also a valid answer.

(A) only

(B) only

(C) only

(D) or

(E) or

页码,2/9The Official SAT Online Course

2006-11-12file://E:\新建文件夹\a6.htm

UnRe

gistered

严禁用于商业用途!

Explanation for Incorrect Answer E :  Choice (E) is not correct. Neither apartment 1 nor apartment 4 is a valid choice for where Alice could live, because Alice must live in an apartment that has two neighboring apartments.

4 What is the ratio of the radius of a circle to the circumference of the circle?

ANSWERS AND EXPLANATIONS Explanation for Correct Answer A :  Choice (A) is correct. The circumference of a circle with radius is equal to

Thus, the ratio of the radius to the circumference is This simplifies to

Explanation for Incorrect Answer B :  Choice (B) is not correct. This is the ratio of the diameter to the circumference.

Explanation for Incorrect Answer C :  Choice (C) is not correct. This is the ratio of the radius to the area.

Explanation for Incorrect Answer D :  Choice (D) is not correct. This is the ratio of the circumference to the diameter.

Explanation for Incorrect Answer E :  Choice (E) is not correct. This is the ratio of the circumference to the radius.

(A)

(B)

(C)

(D)

(E)

5

The graph above shows various temperatures from 10 A.M. to 6 P.M. of a given day. Which of the following situations best fits the information on the graph?

ANSWERS AND EXPLANATIONS Explanation for Correct Answer B :  Choice (B) is correct. The graph shows a sharp decrease in temperature in the

(A) It rained a little, and then the Sun came out and warmed things up.

(B) The mild temperature was lowered by a heavy rain in the morning, and the temperature dropped lower by evening.

(C) It was more windy in the morning than it was in the evening, and the temperature was mild throughout.

(D) The morning was cold, but the Sun later came out and raised the temperature.

(E) The temperature decreased at a constant rate from 10 A.M. to 6 P.M.

页码,3/9The Official SAT Online Course

2006-11-12file://E:\新建文件夹\a6.htm

UnRe

gistered

严禁用于商业用途!

morning and a slight further decrease in temperature in the afternoon. The situation described by choice (B) best fits this information.

Explanation for Incorrect Answer A :  Choice (A) is not correct. If this description were true, you would expect to see an increase in temperature throughout the day. The graph, however, shows a decrease in temperature.

Explanation for Incorrect Answer C :  Choice (C) is not correct. If this description were true, you would expect to see roughly constant temperatures throughout the day. The graph, however, shows a decrease in temperature.

Explanation for Incorrect Answer D :  Choice (D) is not correct. If this description were true, you would expect to see an increase in temperature in the afternoon. The graph, however, shows a decrease in temperature in the afternoon.

Explanation for Incorrect Answer E :  Choice (E) is not correct. If this description were true, you would expect to see the temperature decrease at a constant rate throughout the day. The graph, however, shows a sharper decrease in temperature in the morning than in the afternoon.

6 Rita’s dog weighed pounds when she bought it. Over the next several years, the dog’s weight increased by percent per year. Which of the following functions gives the weight, in pounds, of the dog after years of weight gain at this rate?

ANSWERS AND EXPLANATIONS Explanation for Correct Answer D :  Choice (D) is correct. The dog’s weight increases by percent per year. Each year, therefore, the dog’s new weight is the dog’s weight the previous year plus

times the dog’s weight the previous year. This is the same as multiplying the

previous weight by Therefore, after years, the initial weight of pounds

has increased to pounds.

Explanation for Incorrect Answer A :  Choice (A) is not correct. See the explanation for the correct response (D).

Explanation for Incorrect Answer B :  Choice (B) is not correct. See the explanation for the correct response (D).

Explanation for Incorrect Answer C :  Choice (C) is not correct. See the explanation for the correct response (D).

Explanation for Incorrect Answer E :  Choice (E) is not correct. See the explanation for the correct response (D).

(A)

(B)

(C)

(D)

(E)

7

If all four interior angles of quadrilateral have the same measure, which of the following statements must be true?

页码,4/9The Official SAT Online Course

2006-11-12file://E:\新建文件夹\a6.htm

UnRe

gistered

严禁用于商业用途!

I. All sides of have equal length.

II. The diagonals of are perpendicular.

III. The measure of each interior angle is

ANSWERS AND EXPLANATIONS Explanation for Correct Answer D :  Choice (D) is correct.

I. This statement is not necessarily true. A rectangle that is not a square is an example of a quadrilateral in which all four interior angles do have the same measure but the four sides do not all have the same length.

II. This statement is not necessarily true. A rectangle that is not a square is an example of a quadrilateral in which all four interior angles do have the same measure but the diagonals are not perpendicular.

III. This statement must be true. The sum of the measures of the interior angles in a quadrilateral is If all four interior angles have the same measure,

they each have a measure of

Explanation for Incorrect Answer A :  Choice (A) is not correct. Statement III must be true.

Explanation for Incorrect Answer B :  Choice (B) is not correct. Statement I is not true for rectangles that are not squares.

Explanation for Incorrect Answer C :  Choice (C) is not correct. Statement II is not true for rectangles that are not squares

Explanation for Incorrect Answer E :  Choice (E) is not correct. While statement III must be true, neither statement I nor II is necessarily true.

(A) None

(B) I only

(C) II only

(D) III only

(E) I, II, and III

8If and what is in terms of

ANSWERS AND EXPLANATIONS Explanation for Correct Answer C :  Choice (C) is correct. Substituting for in the equation yields the equation From this, it follows that

(A)

(B)

(C)

(D)

(E)

页码,5/9The Official SAT Online Course

2006-11-12file://E:\新建文件夹\a6.htm

UnRe

gistered

严禁用于商业用途!

Explanation for Incorrect Answer A : 

Choice (A) is not correct.

Explanation for Incorrect Answer B : 

Choice (B) is not correct.

Explanation for Incorrect Answer D : 

Choice (D) is not correct.

Explanation for Incorrect Answer E : 

Choice (E) is not correct.

9

A snack machine has buttons arranged as shown above. If a selection is made by choosing a letter followed by a one-digit number, what is the greatest number of different selections that could be made?

Your Response:  

Correct Response(s):  54

Explanation:  

The correct answer is 54. To determine the number of possible different selections, you multiply the number of possible choices of a letter by the number of possible choices of a number. Therefore, the greatest number of different selections is 6 × x 9 = 54 selections.

10 If and and are positive integers, what is one possible value of

Your Response:  

Correct Response(s):  3 or 6 or 9

Explanation:  

The three possible correct answers are and It is given that and are positive integers. Since is even and is odd, must be odd. If then and If then and If then

and If then but it is given that must be positive. So there are three possible values of and

11A rectangular-shaped field has a perimeter of feet and a width of feet. What

页码,6/9The Official SAT Online Course

2006-11-12file://E:\新建文件夹\a6.htm

UnRe

gistered

严禁用于商业用途!

is the area of the field in square feet?

Your Response:  

Correct Response(s):  9600

Explanation:  

The correct answer is To determine the area of the field, you must first determine its length. The formula for the perimeter of a rectangle is The perimeter is given as and the width is given as Therefore,

which simplifies to feet. The area of a rectangle is the

length multiplied by the width, so the area equals square feet.

12If what is the value of

Your Response:  

Correct Response(s):  2

Explanation:  

The correct answer is . The equation can

be solved for as follows:

13 If divided by one-half is , what is the value of

Your Response:  

Correct Response(s):  25

Explanation:  

The correct answer is Dividing by one-half is the same as multiplying by two. So and

14

页码,7/9The Official SAT Online Course

2006-11-12file://E:\新建文件夹\a6.htm

UnRe

gistered

严禁用于商业用途!

In above, what is the length of

Your Response:  

Correct Response(s):  9

Explanation:  

The correct answer is Since has a measure of and the sum of the

measures of angles and is it follows that must

have a measure of Therefore, the measure of is

This identifies as a special right triangle, which

has the property that the shortest side (opposite the angle) is half as long as

the hypotenuse of the triangle. Therefore,

15 The sum of the positive odd integers less than is subtracted from the sum of the positive even integers less than or equal to . What is the resulting difference?

Your Response:  

Correct Response(s):  50

Explanation:  

The correct answer is The sum of the even integers less than or equal to

is The sum of the odd integers less than is You can see that the first term in the first sum is

more than the first term in the second sum, the second term in the first sum is more than the second term in the second sum, and so on. Since there are such

pairs of terms, the difference between the two sums is

You can also approach this problem algebraically by considering the fact that every positive odd integer less than can be written as and every positive even integer less than or equal to can be written as where is greater than and less than or equal to Since there are terms in each sum and the difference between each pair and is the difference between the two sums is

16

The function above is defined in terms of another function for all values of

where is a constant. If is a number for which and what does

equal?

Your Response:  

Correct Response(s):  3.5, 7/2

Explanation:  

The correct answer is or Substituting the given values for and

into the given equation yields the equation This equation simplifies to

and so The answer can be entered into the grid as either a

fraction or a decimal.

页码,8/9The Official SAT Online Course

2006-11-12file://E:\新建文件夹\a6.htm

UnRe

gistered

严禁用于商业用途!

17 Fifty percent of the songs played on a certain radio station are minutes long, percent are minutes long, and percent are minutes long. What is the average (arithmetic mean) number of minutes per song played on this radio station?

Your Response:  

Correct Response(s):  3.4, 17/5

Explanation:  

The correct answer is or To determine the average length of songs

played on this radio station, you can take the appropriate fraction of each of the lengths and add them together. Since of the songs are minutes long,

are minutes long, and are minutes long, the average length is

minutes. This value can also be expressed and

gridded as

18 A large solid cube is assembled by gluing together identical unpainted small cubic blocks. All six faces of the large cube are then painted red. If exactly 27 of the small cubic blocks that make up the large cube have no red paint on them, how many small cubic blocks make up the large cube?

Your Response:  

Correct Response(s):  125

Explanation:  

The correct answer is The core of the large cube is a cube of unpainted

blocks, or a cube with length, width, and height Around this core is a shell of blocks, one block thick, all of which are painted. This shell adds block to each side of the core cube, or blocks total to each dimension of the core cube, making the length, width, and height of the large cube each equal to A cube of this size

is made up of blocks.

    

Back to Score Report  

Copyright © 2006 The College Board. All rights reserved. Privacy Policy Terms of Use Contact Us

 

页码,9/9The Official SAT Online Course

2006-11-12file://E:\新建文件夹\a6.htm

UnRe

gistered

严禁用于商业用途!

Help | Profile | My Organizer | My Bookmarks | Logout

Answers and Explanations

Test Sections

Section 1

Section 2

Section 3

Section 5

Section 6

Section 7

Section 8

Section 9

Section 10

Back to Score Report  

View Answers and Explanations     Online - Practice Test #1

1 William Faulkner, being that he was a Southern writer, used Mississippi as a setting for most of his novels.

ANSWERS AND EXPLANATIONS Explanation for Correct Answer B :  Choice (B) is correct. It avoids the error of the original by reducing the subordinate clause introduced by "being that" to the modifying phrase "a Southern writer."

Explanation for Incorrect Answer A :  Choice (A) involves the use of an improper idiom. The phrase "being that" is improperly used to introduce a subordinate clause ("he was a Southern writer") where it would be more idiomatic to say, "William Faulkner, who was a Southern writer."

Explanation for Incorrect Answer C :  Choice (C) is unsatisfactory because it results in an illogical sentence. The word "while" indicates a contrast in ideas where there is no contrast.

Explanation for Incorrect Answer D :  Choice (D) involves the use of an improper idiom. It provides the modifying phrase "in origin a writer of the South" where it would be more idiomatic to use a phrase such as "a writer originally from the South."

Explanation for Incorrect Answer E :  Choice (E) results in an unsatisfactory sentence. The sentence would be clearer if the awkward phrase "because of him being" were deleted and the simple modifying phrase "a writer from the South" were left.

(A) being that he was a Southern writer

(B) a Southern writer

(C) while a writer from the South

(D) in origin a writer of the South

(E) because of him being a writer from the South

2Tadpoles hatch and metamorphose into small replicas of adult frogs although remaining in their aquatic birthplace.

ANSWERS AND EXPLANATIONS Explanation for Correct Answer B :  Choice (B) is correct. It avoids the error of the original by replacing the adverb “although,” which indicates contrast, with the adverb “while,” which indicates

(A) although remaining

(B) while remaining

(C) in spite of it remaining

(D) due to their remaining

(E) in the course of which they remain

页码,1/21The Official SAT Online Course

2006-11-12file://E:\新建文件夹\a7.htm

UnRe

gistered

严禁用于商业用途!

the time during which tadpoles change.

Explanation for Incorrect Answer A :  Choice (A) involves improper modification. The adverb "although" illogically indicates a contrast between the two parts of the sentence, which are not contrasting ideas.

Explanation for Incorrect Answer C :  Choice (C) involves the use of a vague pronoun. It is not clear what the pronoun “it” refers to.

Explanation for Incorrect Answer D :  Choice (D) results in an illogical sentence. The phrase "due to" illogically indicates that the development of tadpoles into frogs is caused by their remaining in water.

Explanation for Incorrect Answer E :  Choice (E) involves the use of a vague pronoun. The sentence is unsatisfactory because the pronoun "which" appears to refer to an entire clause ("Tadpoles... frogs").

3 You cannot expect to treat your friends badly and no one notices.

ANSWERS AND EXPLANATIONS Explanation for Correct Answer B :  Choice (B) is correct. It avoids the error of the original by replacing the phrase "and no one notices," with the more idiomatic and parallel phrase "and have no one notice."

Explanation for Incorrect Answer A :  Choice (A) involves the improper use of an idiom. It provides the phrase "and no one notices" where it would be more idiomatic to use the phrase "and have no one notice."

Explanation for Incorrect Answer C :  Choice (C) involves improper diction. The sentence uses the word "notice" as a noun. This use results in an illogical sentence because the noun "notice" has a different meaning from the verb "notice."

Explanation for Incorrect Answer D :  Choice (D) is unsatisfactory because the use of the preposition "without," in combination with the phrase "no one," results in a double negative.

Explanation for Incorrect Answer E :  Choice (E) involves wordiness. The phrase "the result of" is awkward and unnecessary.

(A) and no one notices

(B) and have no one notice

(C) without notice by someone

(D) without notice by no one

(E) without the result of somebody noticing

4 The memoirs of President Clinton begin with his childhood in Arkansas and culminate in his rise to the presidency.

(A) begin with his childhood in Arkansas and culminate

(B) that begin with his childhood in Arkansas and culminate

(C) have begun with his childhood in Arkansas and culminate

页码,2/21The Official SAT Online Course

2006-11-12file://E:\新建文件夹\a7.htm

UnRe

gistered

严禁用于商业用途!

ANSWERS AND EXPLANATIONS Explanation for Correct Answer A :  Choice (A) is correct. The verbs "begin" and "culminate" are both in present tense, and the two verbs join with prepositional phrases to produce parallel constructions.

Explanation for Incorrect Answer B :  Choice (B) creates a sentence fragment. The only verbs (“begin” and “culminate”) are in a dependent clause introduced by “that,” so a complete thought is not stated.

Explanation for Incorrect Answer C :  Choice (C) involves incorrect tense sequence. The present perfect tense of the verb "have begun" is not consistent with the present tense of "culminate."

Explanation for Incorrect Answer D :  Choice (D) fails to maintain parallelism. The verbal "culminating" is not parallel with the earlier verb "have begun."

Explanation for Incorrect Answer E :  Choice (E) exhibits incorrect tense sequence. The past tense of the verb "began" is not consistent with the present tense of "are culminated."

(D) have begun with his childhood in Arkansas and culminating

(E) began with his childhood in Arkansas and are culminated

5

Because of ignoring its potential, biofeedback is a medical therapy most physicians reject.

ANSWERS AND EXPLANATIONS Explanation for Correct Answer D :  Choice (D) is correct. It avoids the error of the original by using a clause with a clearly identified actor—the pronoun "they" (which refers to "most physicians").

Explanation for Incorrect Answer A :  Choice (A) involves loose modification and inappropriate word order. The introductory phrase, "Because of ignoring its potential," is far removed from the only noun in the sentence it could logically modify—"physicians."

Explanation for Incorrect Answer B :  Choice (B) uses an inappropriate idiom. After the conjunction "because," the phrase "of their ignoring" is less idiomatic than "they ignore."

Explanation for Incorrect Answer C :  Choice (C) creates a fragment. Since the phrase contains no verb (only the verbals "ignoring" and "rejecting"), it does not state a complete thought.

Explanation for Incorrect Answer E :  Choice (E) displays improper modification. The verbal "caused" does not logically modify the noun immediately before it, "physicians."

(A) Because of ignoring its potential, biofeedback is a medical therapy most physicians reject.

(B) Biofeedback is rejected by most physicians because of their ignoring its potential.

(C) Most physicians, because of ignoring the potential of biofeedback, and rejecting it.

(D) Most physicians reject biofeedback because they ignore its potential.

(E) A medical therapy rejected by most physicians, caused by ignoring its potential, is biofeedback.

页码,3/21The Official SAT Online Course

2006-11-12file://E:\新建文件夹\a7.htm

UnRe

gistered

严禁用于商业用途!

6 In Death of a Salesman Willy Loman mistakenly believes that his sons have no flaws, believing which leads to many problems for the entire family.

ANSWERS AND EXPLANATIONS Explanation for Correct Answer B :  Choice (B) is correct. It avoids the error of the original by replacing the vague pronoun "which" with the noun "belief" to identify Willy Loman's mistaken idea.

Explanation for Incorrect Answer A :  Choice (A) exhibits loose pronoun reference. The pronoun "which" refers to the idea expressed by an entire clause ("that his sons have no flaws"), not to any specific noun.

Explanation for Incorrect Answer C :  Choice (C) involves loose pronoun reference. The pronoun "which" refers to the idea expressed by an entire clause ("that his sons have no flaws"), not to any specific noun.

Explanation for Incorrect Answer D :  Choice (D) displays wordiness. The phrase "of which" is not needed.

Explanation for Incorrect Answer E :  Choice (E) uses improper coordination. It joins two complete thoughts ("In Death . . . no flaws" and "his believing . . . entire family") with only a comma.

(A) believing which leads

(B) a belief that leads

(C) and which is to lead

(D) the belief of which leads

(E) his believing this leads

7Clara Barton, an American nurse, whose influence as a reformer in the field of health care almost equals that of Florence Nightingale.

ANSWERS AND EXPLANATIONS Explanation for Correct Answer D :  Choice (D) is correct. It avoids the error of the original by forming an independent clause with “Clara Barton” as the subject and “was” as the verb.

Explanation for Incorrect Answer A :  Choice (A) creates a sentence fragment. Since the only verb, “equals,” is within a dependent clause, the sentence does not state a complete thought.

Explanation for Incorrect Answer B :  Choice (B) produces a sentence fragment. Since the only verb, “equals,” is within a dependent clause, the sentence does not state a complete thought.

Explanation for Incorrect Answer C : 

(A) Clara Barton, an American nurse, whose influence as a reformer

(B) Clara Barton, who was an American nurse and whose influence as a reformer

(C) An American with influence as a nursing reformer, Clara Barton

(D) Clara Barton was an American nurse whose influence as a reformer

(E) An American, Clara Barton who was a nursing reformer and whose influence

页码,4/21The Official SAT Online Course

2006-11-12file://E:\新建文件夹\a7.htm

UnRe

gistered

严禁用于商业用途!

Choice (C) makes an illogical comparison. It compares a person, “Clara Barton,” with the influence of another person (“that of Florence Nightingale”).

Explanation for Incorrect Answer E :  Choice (E) involves a sentence fragment. The only verbs, "was" and "equals," are in dependent clauses, so a complete thought is not stated.

8 During the summer months, several thousand people a day visit the park, which is known for its waterfalls and rock formations.

ANSWERS AND EXPLANATIONS Explanation for Correct Answer A :  Choice (A) is correct. The dependent clause introduced by "which" comes immediately after the noun it modifies, "park," and the adverb phrase "During the summer months" comes near the verb it modifies, "visit."

Explanation for Incorrect Answer B :  Choice (B) displays improper modification. The introductory phrase ("Known for its waterfalls and rock formations") cannot logically modify the noun that immediately follows, "people."

Explanation for Incorrect Answer C :  Choice (C) involves improper modification and incorrect word order. Since the phrase "known for its waterfalls and rock formations" cannot logically modify the noun "months," the phrase needs to be placed next to the noun it does apparently modify, "park."

Explanation for Incorrect Answer D :  Choice (D) uses incorrect word order. Since the adverbial phrase "during the summer months" tells when people went to the park (not when it was known for its waterfalls and rocks), the phrase is improperly separated from the verb it modifies, "had visited."

Explanation for Incorrect Answer E :  Choice (E) exhibits incorrect word order. Since the adverbial phrase "during the summer months" tells when people go to the park (not when they know about its waterfalls and rocks), the phrase is improperly separated from the verb it modifies, "visit."

(A) During the summer months, several thousand people a day visit the park, which is known for its waterfalls and rock formations.

(B) Known for its waterfalls and rock formations, several thousand people a day visit the park during the summer months.

(C) Several thousand people a day visit the park during the summer months known for its waterfalls and rock formations.

(D) Several thousand people had visited the park a day, which is known for its waterfalls and rock formations during the summer months.

(E) During the summer months, knowing its waterfalls and rock formations, several thousand people a day visit the park.

9Although the candidate promised both to cut taxes and improve services, he failed to keep either of them after the election.

ANSWERS AND EXPLANATIONS

(A) Although the candidate promised both to cut taxes and improve services, he

(B) The candidate, having promised both to cut taxes and improve services,

(C) Although the candidate made promises both to cut taxes and improve services, he

(D) Having promised, first, to cut taxes and, second, to improve services, the candidate

(E) The candidate’s promises were both to cut taxes and improve services, he

页码,5/21The Official SAT Online Course

2006-11-12file://E:\新建文件夹\a7.htm

UnRe

gistered

严禁用于商业用途!

Explanation for Correct Answer C :  Choice (C) is correct. It avoids the error of the original by including the noun "promises," to which the pronoun "them" can refer.

Explanation for Incorrect Answer A :  Choice (A) exhibits vague pronoun reference. The sentence contains no noun to which the pronoun "them" (apparently meaning "promises") can refer.

Explanation for Incorrect Answer B :  Choice (B) displays vague pronoun reference. The pronoun "them" (apparently meaning "promises") does not refer to any specific noun in the sentence.

Explanation for Incorrect Answer D :  Choice (D) involves vague pronoun reference. The sentence provides no noun to which the pronoun "them" (apparently meaning "promises") can refer.

Explanation for Incorrect Answer E :  Choice (E) uses improper coordination. It joins two complete thoughts ("The candidate's . . . improve services" and "he failed . . . after the election") with only a comma.

10 The students’ fieldwork in the state forest proved more exciting and more dangerous than any of them had anticipated, having to be rescued by helicopter during a fire.

ANSWERS AND EXPLANATIONS Explanation for Correct Answer C :  Choice (C) is correct. It avoids the error of the original by describing the need to be rescued in an independent clause with the pronoun "they" as its subject.

Explanation for Incorrect Answer A :  Choice (A) displays improper modification. The phrase "having to be rescued" (presumably referring to students) has no noun or pronoun to modify.

Explanation for Incorrect Answer B :  Choice (B) uses improper coordination. It uses a semicolon to join unequal sentence parts (a complete thought before the semicolon and a dependent clause after it).

Explanation for Incorrect Answer D :  Choice (D) involves improper modification. The sentence contains no noun that the connecting pronoun "which" can modify.

Explanation for Incorrect Answer E :  Choice (E) exhibits ineffective coordination. It uses the connecting phrase "and so" to join two complete thoughts, but this linkage incorrectly suggests that the first clause describes a cause and the second a result.

(A) anticipated, having to be

(B) anticipated; when they had to be

(C) anticipated: they had to be

(D) anticipated: among which was their

(E) anticipated, and so they had been

11 Chinese watercolors have become more popular than American and European artists who are their contemporaries.

(A) American and European artists who are their contemporaries

(B) contemporary American and European artists

页码,6/21The Official SAT Online Course

2006-11-12file://E:\新建文件夹\a7.htm

UnRe

gistered

严禁用于商业用途!

ANSWERS AND EXPLANATIONS Explanation for Correct Answer E :  Choice (E) is correct. It avoids the error of the original by including the words "those by" so that watercolors from China are compared with watercolors from Europe and America.

Explanation for Incorrect Answer A :  Choice (A) contains an illogical comparison. It compares things, "watercolors," with people, "American and European artists."

Explanation for Incorrect Answer B :  Choice (B) makes an illogical comparison. It compares things, "watercolors," with people, "American and European artists."

Explanation for Incorrect Answer C :  Choice (C) displays wordiness. The phrase "of the period" is not needed since it merely repeats the time specified earlier by the adjective "contemporary."

Explanation for Incorrect Answer D :  Choice (D) exhibits wordiness. The phrase "those of" is not needed and confusing since the phrase also contains the noun "pictures."

(C) those by contemporary American and European artists of the period

(D) those of American and European pictures of the same period

(E) those by contemporary American and European artists

12

By next year the old vaudeville theater converted into two theaters

films can be . 

ANSWERS AND EXPLANATIONS Corrected Sentence: By next year the old vaudeville theatre will have been converted into two small theaters in which films can be shown.

Explanation for Correct Answer A :  The error in this sentence occurs at (A), which involves an improper sequence of tenses. The pluperfect verb phrase, "had been," does not agree with the phrase "By next year" or with the tense of the verb phrase "can be shown" later in the sentence.

Explanation for Incorrect Answer B :  There is no error at (B). The adjective "small" properly modifies the noun "theaters."

Explanation for Incorrect Answer C :  There is no error at (C). The preposition "in" properly combines with the pronoun "which" to refer to the two small theaters where the films can be shown.

Explanation for Incorrect Answer D :  There is no error at (D). The word "shown," a past participle of the verb "to show," properly combines with the phrase "can be" to produce an appropriate verb phrase.

Explanation for Incorrect Answer E :  There is an error in the sentence.

had been small

in which shown No error

页码,7/21The Official SAT Online Course

2006-11-12file://E:\新建文件夹\a7.htm

UnRe

gistered

严禁用于商业用途!

13

Some scholars the writings of Emily Brontë  

interested in the relationships between her siblings and . 

ANSWERS AND EXPLANATIONS Corrected Sentence: Some scholars studying the writings of Emily Brontë have become increasingly interested in the relationships between her siblings and her.

Explanation for Correct Answer D :  The error in this sentence occurs at (D), where an improper pronoun case is used. The nominative pronoun "she" is used in the prepositional phrase introduced by the word "between" instead of the objective pronoun "her."

Explanation for Incorrect Answer A :  There is no error at (A). The participle “studying” properly introduces the modifying phrase, “studying the writings of Emily Brontë,” which modifies “scholars.”

Explanation for Incorrect Answer B :  There is no error at (B). The plural verb phrase "have become" agrees with its plural subject, "scholars."

Explanation for Incorrect Answer C :  There is no error at (C). The adverb "increasingly" properly modifies the adjective "interested."

Explanation for Incorrect Answer E :  There is an error in the sentence.

14

At the 1984 Olympic Games, John Moffet and Pablo Morales,   swimmers on

the United States team,   . 

ANSWERS AND EXPLANATIONS Corrected Sentence: 

Explanation for Correct Answer E :  There is no error in this sentence.

Explanation for Incorrect Answer A :  There is no error at (A). The pronoun "who," which can be either singular or plural, agrees with its plural subject, "John Moffet and Pablo Morales," and is properly used to introduce a subordinate clause.

Explanation for Incorrect Answer B :  There is no error at (B). The plural verb "were" agrees with its plural subject, "who."

Explanation for Incorrect Answer C :  There is no error at (C). The past-tense verb "set" functions logically within the 1984 time frame of the sentence.

studying have become increasingly

she No error

who were

set world records No error

页码,8/21The Official SAT Online Course

2006-11-12file://E:\新建文件夹\a7.htm

UnRe

gistered

严禁用于商业用途!

Explanation for Incorrect Answer D :  There is no error at (D). The plural noun phrase, "world records," properly indicates that Moffet and Morales had separate record-setting performances.

15

Both Dorothy Sayers Carolyn Heilbrun scholarly works

popular murder mysteries, demonstrating a talents and interests.

 

ANSWERS AND EXPLANATIONS Corrected Sentence: Both Dorothy Sayers and Carolyn Heilbrun have written scholarly works as well as popular murder mysteries, demonstrating a diversity of talents and interests.

Explanation for Correct Answer A :  The error in this sentence occurs at (A), where the idiom is inappropriate. The connecting word "plus" is not the correct term for use in a phrase beginning with "both."

Explanation for Incorrect Answer B :  There is no error at (B). The plural verb "have" agrees with its plural subject ("Dorothy Sayers" and "Carolyn Heilbrun"), and the present perfect tense is appropriate to indicate action begun in the past and continuing to the present.

Explanation for Incorrect Answer C :  There is no error at (C). The phrase "as well as" functions properly after the noun "works" to indicate the addition of another item, "popular murder mysteries."

Explanation for Incorrect Answer D :  There is no error at (D). The noun "diversity" functions correctly as the object of the verbal "demonstrating," and "of" is the appropriate preposition to introduce a phrase modifying "diversity."

Explanation for Incorrect Answer E :  There is an error in the sentence.

16

the coach had the team would have a winning season, the

surprised by the the young, inexperienced players.  

ANSWERS AND EXPLANATIONS Corrected Sentence: 

Explanation for Correct Answer E :  There is no error in this sentence.

Explanation for Incorrect Answer A :  There is no error at (A). The subordinating conjunction "Although" properly introduces a subordinate adverbial clause.

plus have written as well as

diversity of

No error

Although predicted that

fans were success of No error

页码,9/21The Official SAT Online Course

2006-11-12file://E:\新建文件夹\a7.htm

UnRe

gistered

严禁用于商业用途!

Explanation for Incorrect Answer B :  There is no error at (B). The word "predicted" combines with the word "had" to produce an appropriate tense of the verb. The verb tense indicates that the coach's prediction happened before the fans' reaction. The relative pronoun "that" properly introduces the noun clause to state what the coach predicted.

Explanation for Incorrect Answer C :  There is no error at (C). The plural verb "were" agrees with its plural subject, "fans," and its tense combines logically with those of the other verbs in the sentence.

Explanation for Incorrect Answer D :  There is no error at (D). The prepositional phrase "of... players" correctly modifies the noun "success."

17

Because they with a large chain of super-stores that can afford to

charge very low rates for certain items, the owners of small hardware stores know

that are unlikely profit , in fact, go bankrupt.  

ANSWERS AND EXPLANATIONS Corrected Sentence: Because they must compete with a large chain of super-stores that can afford to charge very low rates for certain items, the owners of small hardware stores know that they are unlikely to make much profit and may, in fact, go bankrupt.

Explanation for Correct Answer B :  The error in this sentence occurs at (B), where there is a shift of pronoun. The second-person pronoun "you" does not agree with the third-person pronoun "they" earlier in the sentence (which in turn refers to the plural subject of the sentence, "owners").

Explanation for Incorrect Answer A :  There is no error at (A). The verb phrase "must compete" agrees with its subject, "they."

Explanation for Incorrect Answer C :  There is no error at (C). The infinitive “to make” modifies the adjective “unlikely” to produce an appropriate idiom. Furthermore, the adjective “much” properly modifies the noun “profit.”

Explanation for Incorrect Answer D :  There is no error at (D). The verb phrase "may... go bankrupt," introduced by the word "and," is properly parallel with the preceding verb phrase "are unlikely... profit."

Explanation for Incorrect Answer E :  There is an error in the sentence.

18

Although science offers the hope serious genetic diseases,

difficult ethical questions the altering human heredity. 

must compete

you to make much and may No error

of preventing there is

raised by possibility of No error

页码,10/21The Official SAT Online Course

2006-11-12file://E:\新建文件夹\a7.htm

UnRe

gistered

严禁用于商业用途!

ANSWERS AND EXPLANATIONS Corrected Sentence: Although science offers the hope of preventing serious genetic diseases, there are difficult ethical questions raised by the possibility of altering human heredity.

Explanation for Correct Answer B :  The error in this sentence occurs at (B), where there is a problem with subject-verb agreement. In this inverted construction, the singular verb "is" does not agree with its plural subject, "questions."

Explanation for Incorrect Answer A :  There is no error at (A). The preposition "of" and the verbal "preventing" correctly introduce a phrase that modifies the previous noun, "hope."

Explanation for Incorrect Answer C :  There is no error at (C). The verbal "raised" and the preposition "by" correctly begin a phrase that modifies the noun "questions."

Explanation for Incorrect Answer D :  There is no error at (D). The noun "possibility" is used correctly as the object of the preposition "by," and the preposition "of" correctly begins another phrase that modifies "possibility."

Explanation for Incorrect Answer E :  There is an error in the sentence.

19

In the nineteenth century, careers in business and law prestigious, but

  practitioners college degrees. 

ANSWERS AND EXPLANATIONS Corrected Sentence: In the nineteenth century, careers in business and law were prestigious, but they did not require practitioners to hold college degrees.

Explanation for Correct Answer B :  The error in this sentence occurs at (B), where the number of the pronoun is incorrect. The singular pronoun "it" does not agree with the plural noun "careers," to which it refers.

Explanation for Incorrect Answer A :  There is no error at (A). The plural verb "were" agrees with its plural subject, "careers."

Explanation for Incorrect Answer C :  There is no error at (C). The verb in past tense, "did require," is consistent with the earlier past-tense verb, "were," and the adverb "not" is the appropriate negative word.

Explanation for Incorrect Answer D :  There is no error at (D). The verbal "to hold" is the appropriate phrase to use between the nouns "practitioners" and "college degrees."

Explanation for Incorrect Answer E :  There is an error in the sentence.

20

were

it did not require to hold No error

页码,11/21The Official SAT Online Course

2006-11-12file://E:\新建文件夹\a7.htm

UnRe

gistered

严禁用于商业用途!

Chess players find that playing against a computer is helpful   skills,

no chess-playing computer has yet a championship.  

ANSWERS AND EXPLANATIONS Corrected Sentence: Chess players find that playing against a computer is helpful in improving their skills, even though no chess-playing computer has yet won a championship.

Explanation for Correct Answer A :  The error in this sentence occurs at (A), where an improper idiom is used. The infinitive “to improve” is used in combination with the verb phrase “is helpful,” where it would be more idiomatic to use the phrase “in improving.”

Explanation for Incorrect Answer B :  There is no error at (B). The plural possessive pronoun "their" is properly used to refer to the subject, "chess players."

Explanation for Incorrect Answer C :  There is no error at (C). The subordinating conjunction “even though” properly introduces the subordinate adverbial clause, “even though no chess-playing . . . championship.”

Explanation for Incorrect Answer D :  There is no error at (D). The word "won," the past participle of the verb "to win," combines with the word "has" to produce the appropriate verb tense.

Explanation for Incorrect Answer E :  There is an error in the sentence.

21

lengths of twelve inches, banana slugs the of all the slug

species that North America.  

ANSWERS AND EXPLANATIONS Corrected Sentence: Reaching lengths of twelve inches, banana slugs are the largest of all the slug species that inhabit North America.

Explanation for Correct Answer C :  The error in this sentence occurs at (C), where the comparative term is incorrect. The phrase "much larger" implies a comparison between only two items, not a comparison of banana slugs with all other slug species in North America.

Explanation for Incorrect Answer A :  There is no error at (A). The verbal "Reaching" properly introduces a participial phrase that modifies the noun "slugs."

Explanation for Incorrect Answer B :  There is no error at (B). The plural verb "are" agrees with its plural subject, "slugs."

Explanation for Incorrect Answer D :  There is no error at (D). The plural verb "inhabit" agrees with the plural noun phrase "all the slug species," and the present tense of "inhabit" is consistent with

to improve their

even though won No error

Reaching are much larger

inhabit No error

页码,12/21The Official SAT Online Course

2006-11-12file://E:\新建文件夹\a7.htm

UnRe

gistered

严禁用于商业用途!

the tense of the earlier verb "are."

Explanation for Incorrect Answer E :  There is an error in the sentence.

22

Maya Madera wore her sister’s for winter

campers lighted numerals and sends out a loud alarm

when  

ANSWERS AND EXPLANATIONS Corrected Sentence: Maya Madera proudly wore her sister’s most popular invention, a watch for winter campers that flashes the temperature in lighted numerals and sends out a loud alarm when one presses a button.

Explanation for Correct Answer D :  The error in this sentence occurs at (D), where modification is improper. The phrase "pressing a button" cannot logically modify the noun just before it, "alarm," or any other word in the sentence.

Explanation for Incorrect Answer A :  There is no error at (A). The adverb "proudly" (telling how) is used correctly to modify the verb "wore."

Explanation for Incorrect Answer B :  There is no error at (B). The noun phrase "most popular invention" functions correctly as the direct object of the verb "wore," and the noun "watch" is placed properly as an appositive to "invention."

Explanation for Incorrect Answer C :  There is no error at (C). The dependent clause "that flashes the temperature" correctly serves as an adjective describing the watch.

Explanation for Incorrect Answer E :  There is an error in the sentence.

23

, Whistler’s paintings, , are   subject matter.

 

ANSWERS AND EXPLANATIONS Corrected Sentence: Obviously, Whistler’s paintings, unlike those of Klee, are conventional in their subject matter.

Explanation for Correct Answer B :  The error in this sentence occurs at (B), where the comparison is not logical. Works of art ("Whistler's paintings") are improperly compared with an artist ("Klee").

Explanation for Incorrect Answer A : 

proudly most popular invention, a watch

that flashes the temperature in

pressing a button. No error

Obviously unlike Klee conventional in their

No error

页码,13/21The Official SAT Online Course

2006-11-12file://E:\新建文件夹\a7.htm

UnRe

gistered

严禁用于商业用途!

There is no error at (A). The adverb "obviously" correctly modifies "are" and indicates a high degree of certainty.

Explanation for Incorrect Answer C :  There is no error at (C). The adjective "conventional" is used appropriately after the linking verb "are," and the preposition "in" correctly introduces a phrase that modifies "conventional."

Explanation for Incorrect Answer D :  There is no error at (D). The plural pronoun "their" agrees with the plural noun "paintings," to which it refers.

Explanation for Incorrect Answer E :  There is an error in the sentence.

24

A newly formed organization of homeowners and business people the

transportation department its   a shopping mall in the

community. 

ANSWERS AND EXPLANATIONS Corrected Sentence: A newly formed organization of homeowners and business people has met with the transportation department to voice its concerns about plans for a shopping mall in the community.

Explanation for Correct Answer A :  The error in this sentence occurs at (A), where there is a problem with subject-verb agreement. The plural verb "have" agrees with the interrupting nouns ("homeowners and business people") but not with its singular subject, "organization."

Explanation for Incorrect Answer B :  There is no error at (B). The infinitive "to voice" correctly introduces a verbal phrase that modifies "met" and indicates the purpose of the meeting.

Explanation for Incorrect Answer C :  There is no error at (C). The noun "concerns" serves correctly as the direct object of the infinitive "to voice," and the preposition "about" is appropriate to begin a phrase that modifies "concerns."

Explanation for Incorrect Answer D :  There is no error at (D). The noun "plans" serves correctly as the object of the preposition "about," and the preposition "for" appropriately begins another prepositional phrase that modifies "plans."

Explanation for Incorrect Answer E :  There is an error in the sentence.

25

she considers her chemistry research , she her

professor’s advice and three additional experiments. 

ANSWERS AND EXPLANATIONS

have met with

to voice concerns about plans for

No error

Although complete has heeded

is conducting No error

页码,14/21The Official SAT Online Course

2006-11-12file://E:\新建文件夹\a7.htm

UnRe

gistered

严禁用于商业用途!

Corrected Sentence: 

Explanation for Correct Answer E :  There is no error in this sentence.

Explanation for Incorrect Answer A :  There is no error at (A). The subordinating conjunction "Although" properly introduces the subordinate adverbial clause, "she considers... complete."

Explanation for Incorrect Answer B :  There is no error at (B). The adjective "complete" properly modifies the noun phrase, "her chemistry research."

Explanation for Incorrect Answer C :  There is no error at (C). The singular verb phrase "has heeded" agrees with its singular subject, "she."

Explanation for Incorrect Answer D :  There is no error at (D). The singular verb phrase "is conducting" agrees with its singular subject, "she," and properly introduces the second half of the compound predicate of the main clause.

26

Princeton University officials first a tradition honorary degrees

only when they awarded to author Willa Cather. 

ANSWERS AND EXPLANATIONS Corrected Sentence: Princeton University officials first broke with a tradition of awarding honorary degrees only to men when they awarded such a degree to author Willa Cather.

Explanation for Correct Answer D :  The error in this sentence occurs at (D), where the pronoun reference is vague. The sentence contains no singular noun to which the pronoun "it" (presumably meaning an honorary degree) can refer.

Explanation for Incorrect Answer A :  There is no error at (A). The past tense verb "broke" is used correctly to describe action already completed, and the preposition "with" is used properly to begin a phrase modifying "broke."

Explanation for Incorrect Answer B :  There is no error at (B). The preposition "of" is used correctly to begin a phrase modifying "tradition," and the verbal "awarding" functions properly as the object of the preposition.

Explanation for Incorrect Answer C :  There is no error at (C). The prepositional phrase "to men" is used correctly to identify those awarded honorary degrees.

Explanation for Incorrect Answer E :  There is an error in the sentence.

27

His love of politics   to volunteer in local campaigns as in a

broke with of awarding

to men it No error

led him as well a job

页码,15/21The Official SAT Online Course

2006-11-12file://E:\新建文件夹\a7.htm

UnRe

gistered

严禁用于商业用途!

government office in the state capital.  

ANSWERS AND EXPLANATIONS Corrected Sentence: His love of politics led him to volunteer in local campaigns as well as to work in a government office in the state capital.

Explanation for Correct Answer D :  The error in this sentence occurs at (D), where there is a flaw in parallelism. The noun phrase "a job" is not parallel with the earlier verbal phrase "to volunteer."

Explanation for Incorrect Answer A :  There is no error at (A). The past-tense verb "led" correctly describes actions already completed.

Explanation for Incorrect Answer B :  There is no error at (B). Since the pronoun "him" is the direct object of the verb "led," the objective case is correct.

Explanation for Incorrect Answer C :  There is no error at (C). The connecting phrase "as well" functions correctly to introduce an additional activity.

Explanation for Incorrect Answer E :  There is an error in the sentence.

28

Now that Michiko the research, she feels  

her paper on the the progressive movement in the United States. 

ANSWERS AND EXPLANATIONS Corrected Sentence: Now that Michiko has finished the research, she feels reasonably confident about writing her paper on the rise of the progressive movement in the United States.

Explanation for Correct Answer A :  The error in this sentence occurs at (A), where the tense sequence is incorrect. The past tense of the verb, "finished," is inappropriate because the adverb "now" indicates action begun in the past but continuing until the present (action described correctly by the present perfect tense).

Explanation for Incorrect Answer B :  There is no error at (B). The adjective "confident" is used correctly after the verb "feels," and the adverb "reasonably" properly modifies "confident."

Explanation for Incorrect Answer C :  There is no error at (C). The prepositional phrase "about writing" functions properly as an adverb modifying the adjective "confident."

Explanation for Incorrect Answer D :  There is no error at (D). The noun "rise" serves appropriately as the object of the preposition "on," and the preposition "of" introduces another prepositional phrase that modifies "rise."

Explanation for Incorrect Answer E :  There is an error in the sentence.

No error

finished reasonably confident about writing

rise of No error

页码,16/21The Official SAT Online Course

2006-11-12file://E:\新建文件夹\a7.htm

UnRe

gistered

严禁用于商业用途!

29

The condition known as laryngitis the vocal cords and surrounding

tissue to swell, preventing the cords   . 

ANSWERS AND EXPLANATIONS Corrected Sentence: The condition known as laryngitis usually causes the vocal cords and surrounding tissue to swell, thus preventing the cords from moving freely.

Explanation for Correct Answer C :  The error in this sentence occurs at (C), where the idiom is inappropriate. After the verbal "preventing" the phrase "to move" is not idiomatic.

Explanation for Incorrect Answer A :  There is no error at (A). The adverb "usually" correctly modifies the verb "causes," and that singular verb agrees with its singular subject, "condition."

Explanation for Incorrect Answer B :  There is no error at (B). The connecting word "thus" is used correctly to link a cause (swelling) with its effect (prevention of movement).

Explanation for Incorrect Answer D :  There is no error at (D). The adverb "freely" is used correctly to tell how the cords normally move

Explanation for Incorrect Answer E :  There is an error in the sentence.

(1) A significant problem all across our state is garbage. (2) Our landfills are full. (3) It seems that we must either find new sites for landfills or employ other methods of disposal, like incineration. (4) Unfortunately, there are drawbacks to every solution that they think of. (5) Polluted runoff water often results from landfills. (6) With incineration of trash, you get air pollution. (7) People are criticized for not wanting to live near a polluting waste disposal facility, but really, can you blame them? 

(8) Recycling can be an effective solution, but owners of apartment complexes and other businesses complain that recycling adds to their expenses. (9)  Local governments enjoy the benefits of taxes collected from business and industry. (10)  They tend to shy away from pressuring such heavy contributors to recycle. 

(11) Perhaps those of us being concerned should encourage debate about what other levels of government can do to solve the problems of waste disposal.  (12) We should make a particular effort to cut down on the manufacture and use of things that will not decompose quickly.  (13) Certainly we should press individuals, industries, and all levels of government to take responsible action while we can still see green grass and trees between the mountains of waste. 

30Which of the following would fit most logically between sentences 1 and 2?

(A) A sentence citing examples of states that have used up available landfills

(B) A sentence citing examples of successful alternatives to landfills

(C) A sentence citing the number of new landfills in the state

(D) A sentence citing the number of illnesses blamed on polluted water in the state

usually causes

thus to move freely No error

页码,17/21The Official SAT Online Course

2006-11-12file://E:\新建文件夹\a7.htm

UnRe

gistered

严禁用于商业用途!

ANSWERS AND EXPLANATIONS Explanation for Correct Answer E :  Choice (E) is correct. Information about the amount of trash disposed of annually in the state explains how the state's landfills came to be full.

Explanation for Incorrect Answer A :  Choice (A) is unsatisfactory because it deals with other states; it does not explain how landfills in the writer's own state became full.

Explanation for Incorrect Answer B :  Choice (B) is unsatisfactory because what is needed is a sentence about landfills, not alternatives to landfills.

Explanation for Incorrect Answer C :  Choice (C) is unsatisfactory because the information that would fit most logically is not about the number of landfills but the amount of garbage that goes into them.

Explanation for Incorrect Answer D :  Choice (D) is unsatisfactory because polluted water, a byproduct of landfills, is discussed later in the essay; what is needed here is information about how the landfills themselves became full.

(E) A sentence citing the average amount of trash disposed of annually by each person in the state

31Which of the following is the best way to phrase the underlined portion of sentence 4 (reproduced below) ? Unfortunately, there are drawbacks to every solution that they think of.

ANSWERS AND EXPLANATIONS Explanation for Correct Answer E :  Choice (E) is correct. It avoids the error of the original by providing a passive verb phrase, "has been proposed," and gets rid of the vague pronoun "they."

Explanation for Incorrect Answer A :  Choice (A) is unsatisfactory because it involves the use of a vague pronoun. Neither the sentence nor its context mentions anyone to whom the pronoun "they" can logically refer.

Explanation for Incorrect Answer B :  Choice (B) is unsatisfactory because the past tense of the verb does not fit logically into the context, which describes the situation entirely in the present tense.

Explanation for Incorrect Answer C :  Choice (C) is unsatisfactory because there is nothing in the sentence to which the pronoun “they” can logically refer. Furthermore, the adverb “previously” does not make sense in the context of the sentence.

Explanation for Incorrect Answer D :  Choice (D) is unsatisfactory because it results in an illogical sentence. It makes no sense to speak of proposals that are made to solutions; proposals are made not to solutions, but to people or groups of people.

(A) (as it is now)

(B) that was thought of

(C) that they have previously come up with

(D) to which there are proposals

(E) that has been proposed

页码,18/21The Official SAT Online Course

2006-11-12file://E:\新建文件夹\a7.htm

UnRe

gistered

严禁用于商业用途!

32 Which of the following is the best way to revise and combine sentences 5 and 6 (reproduced below) ? Polluted runoff water often results from landfills. With incineration of trash, you get air pollution.

ANSWERS AND EXPLANATIONS Explanation for Correct Answer C :  Choice (C) is correct. It combines the two sentences using the same grammatical structure in both clauses and linking them with the word "and."

Explanation for Incorrect Answer A :  Choice (A) is unsatisfactory because it involves improper coordination: the two clauses are improperly linked with both "and" and "whereas."

Explanation for Incorrect Answer B :  Choice (B) is unsatisfactory because the wordy phrases “While on the one hand . . . on the other hand” illogically indicate a contrast between the two ideas. Also, the sentence does not make clear the link between landfills and polluted runoff water; it merely says that both exist.

Explanation for Incorrect Answer D :  Choice (D) is unsatisfactory because it is a fragment; the sentence has no main verb.

Explanation for Incorrect Answer E :  Choice (E) is unsatisfactory because the verbs "is" and "is" do not indicate causal connections; also, the word "new" changes the meaning of the first clause, probably making that statement inaccurate.

(A) With landfills, polluted runoff water will result, and whereas with incineration of trash, you get air pollution.

(B) While on the one hand are landfills and polluted runoff water, on the other hand you have air pollution in the case of incineration of trash.

(C) Landfills often produce polluted runoff water, and trash incineration creates air pollution.

(D) Landfills and incineration that produce water and air pollution.

(E) Runoff water is from new landfills; from incineration of trash, there is air pollution.

33If sentence 8 were rewritten to begin with the clause “Although recycling can be an effective solution,” the next words would most logically be

ANSWERS AND EXPLANATIONS Explanation for Correct Answer C :  Choice (C) is correct. The main clause completes the sentence by describing a negative reaction and needs no connecting word to link it with the introductory dependent clause.

(A) and owners of apartment complexes and other businesses complain

(B) yet owners of apartment complexes and other businesses complain

(C) owners of apartment complexes and other businesses complain

(D) mostly owners of apartment complexes and other businesses are complaining

(E) owners of apartment complexes and other business complained

页码,19/21The Official SAT Online Course

2006-11-12file://E:\新建文件夹\a7.htm

UnRe

gistered

严禁用于商业用途!

Explanation for Incorrect Answer A :  Choice (A) is unsatisfactory because the coordinating conjunction "and" is used incorrectly to join an independent clause ("owners of apartment complexes and other businesses complain") to an introductory dependent clause (“Although recycling can be an effective solution”).

Explanation for Incorrect Answer B :  Choice (B) is unsatisfactory because the coordinating conjunction "yet" is used incorrectly to join an independent clause ("owners of apartment complexes and other businesses complain") to an introductory dependent clause (“Although recycling can be an effective solution”). The contrast signalled by "yet" is unnecessary since "Although" is now at the beginning of the sentence.

Explanation for Incorrect Answer D :  Choice (D) is unsatisfactory because the adverb "mostly" is unnecessary and alters the meaning of the original.

Explanation for Incorrect Answer E :  Choice (E) is unsatisfactory because the past tense of the verb "complained" is inconsistent with the present tense of the earlier verb "can be."

34 In context, which of the following is the best way to combine sentences 9 and 10?

ANSWERS AND EXPLANATIONS Explanation for Correct Answer B :  Choice (B) is correct. The subordinating conjunction "Because" clearly establishes a cause-effect relationship between the benefits described in the introductory clause and the tendency described in the main clause.

Explanation for Incorrect Answer A :  Choice (A) is unsatisfactory because the conjunction "as" may suggest only that the two conditions exist at the same time, not that one causes the other.

Explanation for Incorrect Answer C :  Choice (C) is unsatisfactory because it uses incorrect coordination. Two complete thoughts ("However, local governments enjoy . . . from business and industry" and "they tend . . . to recycle") are joined by only a comma.

Explanation for Incorrect Answer D :  Choice (D) is unsatisfactory because the phrase "In addition to" does not suggest a cause-effect relationship.

Explanation for Incorrect Answer E :  Choice (E) is unsatisfactory because the pronoun "they" unnecessarily repeats the subject of the sentence, "Local governments."

(A)Local governments enjoy the benefits of taxes collected from business and industry, as they tend to shy away from pressuring such heavy contributors to recycle.

(B)Because local governments enjoy the benefits of taxes collected from business and industry, they tend to shy away from pressuring such heavy contributors to recycle.

(C)However, local governments enjoy the benefits of taxes collected from business and industry, they tend to shy away from pressuring such heavy contributors to recycle.

(D)In addition to enjoying the benefits of taxes collected from business and industry, local governments tend to shy away from pressuring business and industry into recycling

(E) Local governments, enjoying the benefits of taxes collected from business and industry, they tend to shy away from pressure to recycle.

35 Which of the following is the best phrasing for the underlined portion of sentence 11 (reproduced below)? Perhaps those of us being concerned should encourage debate about what other levels of government can do to solve the problems of waste disposal.

页码,20/21The Official SAT Online Course

2006-11-12file://E:\新建文件夹\a7.htm

UnRe

gistered

严禁用于商业用途!

ANSWERS AND EXPLANATIONS Explanation for Correct Answer B :  Choice (B) is correct. The dependent clause "who are concerned" is an appropriate idiom to follow the pronoun "us," and the entire phrase uses no unnecessary words.

Explanation for Incorrect Answer A :  Choice (A) is unsatisfactory because, after the pronoun "us," the verbal phrase "being concerned" is less idiomatic than the dependent clause "who are oncerned."

Explanation for Incorrect Answer C :  Choice (C) is unsatisfactory because it inappropriately separates the pronoun "those" from the prepositional phrase "of us."

Explanation for Incorrect Answer D :  Choice (D) is unsatisfactory because the phrase "being among those" is not needed.

Explanation for Incorrect Answer E :  Choice (E) is unsatisfactory because the words "those being" are not necessary.

(A) (as it is now)

(B) those of us who are concerned

(C) those concerned ones of us

(D) we, being among those who are concerned,

(E) we who are those being concerned

    

Back to Score Report  

Copyright © 2006 The College Board. All rights reserved. Privacy Policy Terms of Use Contact Us

 

页码,21/21The Official SAT Online Course

2006-11-12file://E:\新建文件夹\a7.htm

UnRe

gistered

严禁用于商业用途!

Help | Profile | My Organizer | My Bookmarks | Logout

Answers and Explanations

Test Sections

Section 1

Section 2

Section 3

Section 5

Section 6

Section 7

Section 8

Section 9

Section 10

Back to Score Report  

View Answers and Explanations     Online - Practice Test #1

1 Most pioneers ------- this valley on their journey to the West because its rugged terrain and frequent landslides made it a ------- place for travelers.

ANSWERS AND EXPLANATIONS Explanation for Correct Answer E :  Choice (E) is correct. "Skirted" means avoided, and "hazardous" means dangerous. If one were to insert these terms into the text, the sentence would read "Most pioneers skirted this valley on their journey to the West because its rugged terrain and frequent landslide made it a hazardous place for travelers." Since the word "because" indicates that the activity in the first part of the sentence is directly affected by the factors mentioned in the second part of the sentence, choice (E) makes sense: The pioneers avoided the valley as a direct result of their awareness of its dangerous terrain.

Explanation for Incorrect Answer A :  Choice (A) is incorrect. "Flanked" means moved around the side of, and "fascinating" means alluring. If one were to insert these terms into the text, the sentence would read "Most pioneers flanked this valley on their journey to the West because its rugged terrain and frequent landslides made it a fascinating place for travelers." It is unlikely that settlers would find frequent landslides "fascinating," and even if they had, choosing to move around the side of the valley would not have been a reasonable expression of this fascination.

Explanation for Incorrect Answer B :  Choice (B) is incorrect. "Avoided" means kept away from, and "necessary" means essential. If one were to insert these terms into the text, the sentence would read "Most pioneers avoided this valley on their journey to the West because its rugged terrain and frequent landslides made it a necessary place for travelers." Although it would have been reasonable for pioneers to avoid the dangerous valley, it makes no sense to say that they avoided it because its hazards made it a "necessary place for travelers."

Explanation for Incorrect Answer C :  Choice (C) is incorrect. "Encompassed" means enclosed, and "curious," in this context, means unusual. If one were to insert these terms into the text, the sentence would read "Most pioneers encompassed this valley on their journey to the West because its rugged terrain and frequent landslides made it a curious place for travelers." It is not reasonable to characterize the threat of injury or death as "curious," and "encompassing," or surrounding, the valley during the pioneers' journey would not have been a logical result of their discovering the valley's danger.

Explanation for Incorrect Answer D :  Choice (D) is incorrect. "Enjoyed" means took pleasure in, and "troublesome" means difficult. If one were to insert these terms into the text, the sentence would read "Most pioneers enjoyed this valley on their journey to the West because its rugged terrain and frequent landslides made it a troublesome place for travelers." Dangerous conditions would certainly make the valley a difficult place for travelers, but they would almost certainly not find the place pleasurable.

(A) flanked. . fascinating

(B) avoided. . necessary

(C) encompassed. . curious

(D) enjoyed. . troublesome

(E) skirted. . hazardous

页码,1/16The Official SAT Online Course

2006-11-12file://E:\新建文件夹\a8.htm

UnRe

gistered

严禁用于商业用途!

2 Ballads often praise popular figures who have performed feats that many perceive as -------, such as defending the poor or resisting ------- authority.

ANSWERS AND EXPLANATIONS Explanation for Correct Answer D :  Choice (D) is correct. "Admirable" means deserving approval, and "unjust" means unfair. If one were to insert these terms into the text, the sentence would read "Ballads often praise popular figures who have performed feats that many perceive as admirable, such as defending the poor or resisting unjust authority." Because the figures mentioned in the first part of the sentence are praised for their "feats," or notable deeds, the first missing term will have a positive meaning. The second missing term, used to describe the type of authority that these figures were praised for resisting, will have a negative meaning. Since feats such as helping the poor and resisting unjust authority are seen as admirable and therefore are praised by balladeers, choice (D) is correct.

Explanation for Incorrect Answer A :  Choice (A) is incorrect. "Modest" means humble, and "acceptable" means adequate. If one were to insert these terms into the text, the sentence would read "Ballads often praise popular figures who have performed feats that many perceive as modest, such as defending the poor or resisting acceptable authority." Resisting "acceptable authority" would not be viewed as a modest, or humble feat, nor would performers of such modest feats be likely to become "popular figures" to be praised in a ballad.

Explanation for Incorrect Answer B :  Choice (B) is incorrect. "Inescapable" means unavoidable, and "legitimate" means approved by law. If one were to insert these terms into the text, the sentence would read "Ballads often praise popular figures who have performed feats that many perceive as inescapable, such as defending the poor or resisting legitimate authority." Resisting "legitimate authority" would not be seen as unavoidable; therefore, this type of behavior would not be likely to be praised.

Explanation for Incorrect Answer C :  Choice (C ) is incorrect. "Insufficient" means inadequate, and something that is "overpowering" is characterized by overwhelming intensity. If one were to insert these terms into the text, the sentence would read "Ballads often praise popular figures who have performed feats that many perceive as insufficient, such as defending the poor or resisting overpowering authority." Even though figures might resist "overpowering authority," it is not reasonable to suggest that these figures would also perform feats that are considered insufficient and that they would be popular and deserving of praise.

Explanation for Incorrect Answer E :  Choice (E) is incorrect. "Unbelievable" means too improbable for belief, and "tolerable" means fairly good. If one were to insert these terms into the text, the sentence would read "Ballads often praise popular figures who have performed feats that many perceive as unbelievable, such as defending the poor or resisting tolerable authority." Resisting tolerable authority would hardly be seen as an unbelievable feat and, if it were, those performing the feat would not be popular figures.

(A) modest. . acceptable

(B) inescapable . . legitimate

(C) insufficient . . overpowering

(D) admirable. . unjust

(E) unbelievable. . tolerable

3 The serious purpose of the paper-airplane flying contest, which attracted many novel and sometimes truly ------- entries, was to determine whether any ------- aerodynamic designs could be discovered.

(A) unorthodox . . conventional

页码,2/16The Official SAT Online Course

2006-11-12file://E:\新建文件夹\a8.htm

UnRe

gistered

严禁用于商业用途!

ANSWERS AND EXPLANATIONS Explanation for Correct Answer B :  Choice (B) is correct. "Bizarre" means oddly eccentric, and "revolutionary" means marked by radical change. If one were to insert these terms into the text, the sentence would read "The serious purpose of the paper-airplane flying contest, which attracted many novel and sometimes truly bizarre entries, was to determine whether any revolutionary aerodynamic designs could be discovered." The structure of the sentence indicates that the correct response is the only pairing where the first missing term parallels the word "novel," or new, and makes sense in the first part of the sentence without contradicting the meaning of the second missing word. Since it is logical that the organizers of the contest hoped to discover a revolutionary design from among the entries that were so new that they seemed bizarre, choice (B) makes sense.

Explanation for Incorrect Answer A :  Choice (A) is incorrect. "Unorthodox" means extraordinary, and "conventional" means ordinary. If one were to insert these terms into the text, the sentence would read "The serious purpose of the paper-airplane flying contest, which attracted many novel and sometimes truly unorthodox entries, was to determine whether any conventional aerodynamic designs could be discovered." The terms "unorthodox" and "conventional" have opposite meanings. Therefore, the organizers could not have expected to find a conventional, or traditional, design by choosing from among novel and unconventional entries.

Explanation for Incorrect Answer C :  Choice (C) is incorrect. "Derivative" means unoriginal, and "imaginative" means creative. If one were to insert these terms into the text, the sentence would read "The serious purpose of the paper-airplane flying contest, which attracted many novel and sometimes truly derivative entries, was to determine whether any imaginative aerodynamic designs could be discovered." If the entries were based on already established designs, the contest would not have been likely to yield many imaginative or original designs, so choice (C) does not make sense.

Explanation for Incorrect Answer D :  Choice (D) is incorrect. "Mundane" means ordinary, and "predictable" means easily foreseen, or routine. If one were to insert these terms into the text, the sentence would read "The serious purpose of the paper-airplane flying contest, which attracted many novel and sometimes truly mundane entries, was to determine whether any predictable aerodynamic designs could be discovered." It would not make sense for the organizers of the contest to try to discover predictable designs, since the point of discovery is to find something new; ordinary, entries are unlikely to be novel. For these reasons, choice (D) does not make sense.

Explanation for Incorrect Answer E :  Choice (E) is incorrect. "Ungainly" means graceless and unwieldly, and "aesthetic" means tastefully attractive. If one were to insert these terms into the text, the sentence would read "The serious purpose of the paper-airplane flying contest, which attracted many novel and sometimes truly ungainly entries, was to determine whether any aesthetic aerodynamic designs could be discovered." The terms "ungainly" and "aesthetic" have opposite meaning, therefore a contest that attracted graceless entries would not be likely to yield tastefully attractive designs.

(B) bizarre. . revolutionary

(C) derivative. . imaginative

(D) mundane . . predictable

(E) ungainly. . aesthetic

4 Carson presents her case so strongly and logically that only the prejudiced or the ------- will attempt to ------- her.

(A) impartial. . defy

(B) doubtful. . champion

(C) gullible. . believe

(D) obstinate . . contradict

页码,3/16The Official SAT Online Course

2006-11-12file://E:\新建文件夹\a8.htm

UnRe

gistered

严禁用于商业用途!

ANSWERS AND EXPLANATIONS Explanation for Correct Answer D :  Choice (D) is correct. "Obstinate" means stubborn, and "contradict" means to assert the opposite of. If one were to insert these terms into the text, the sentence would read "Carson presents her case so strongly and logically that only the prejudiced or the obstinate will attempt to contradict her." The sentence is structured so that the first missing word will be a negative one that parallels "prejudiced," or having a preconceived opinion, and the second missing word will be something that someone with the negative characteristic described by the first missing word would attempt to do. Choice (D) makes sense, because only a stubborn person would attempt to contradict Carson's strong and logical case.

Explanation for Incorrect Answer A :  Choice (A) is incorrect. "Impartial" means fair, and "defy" means to challenge. If one were to insert these terms into the text, the sentence would read "Carson presents her case so strongly and logically that only the prejudiced or the impartial will attempt to defy her." Since an impartial, person would not be likely to challenge a strong and logical argument, choice (A) does not make sense.

Explanation for Incorrect Answer B :  Choice (B) is incorrect. "Doubtful" means undecided or skeptical, and "to champion" means to support. If one were to insert these terms into the text, the sentence would read "Carson presents her case so strongly and logically that only the prejudiced or the doubtful will attempt to champion her. Someone who is doubtful about the strength of Carson's case is not likely to support her.

Explanation for Incorrect Answer C :  Choice (C) is incorrect. "Gullible" means easily fooled, and "believe" means to accept as true. If one were to insert these terms into the text, the sentence would read "Carson presents her case so strongly and logically that only the prejudiced or the gullible will attempt to believe her." If only someone who can be easily fooled is likely to believe Carson's case, then her case cannot be described as strong and logical.

Explanation for Incorrect Answer E :  Choice (E) is incorrect. "Irrational" means not led by reason or logic, and "follow," in this context, means to agree with or adhere to. If Carson presents her case strongly and logically, it would not make sense that someone who is irrational, or illogical, would necessarily tend to agree with her.

(E) irrational . . follow

5Over the years the anthropologist’s opinions had -------: he refused to tolerate new ideas and nothing could change his mind.

ANSWERS AND EXPLANATIONS Explanation for Correct Answer C :  Choice (C) is correct. "Ossified" means became rigidly set. If one were to insert this term into the text, the sentence would read "Over the years the anthropologist's opinions had ossified: he refused to tolerate new ideas and nothing could change his mind." Since the colon indicates that the phrases on either side of it have the same meaning, the word "ossified" makes sense: by ignoring new ideas, the anthropologist has allowed his opinions to become rigid and unchanging.

(A) digressed

(B) proliferated

(C) ossified

(D) germinated

(E) incubated

页码,4/16The Official SAT Online Course

2006-11-12file://E:\新建文件夹\a8.htm

UnRe

gistered

严禁用于商业用途!

Explanation for Incorrect Answer A :  Choice (A) is incorrect. "Digressed" means strayed from the subject. If one were to insert this term into the text, the sentence would read "Over the years the anthropologists's opinions had digressed: he refused to tolerate new ideas and nothing could change his mind." Since the colon in this sentence indicates that the missing term should mean the same as mentally rigid and inflexible, "digressed" does not make sense here.

Explanation for Incorrect Answer B :  Choice (B) is incorrect. "Proliferated" means multiplied rapidly. If one were to insert this term into the text, the sentence would read "Over the years the anthropologist's opinions had proliferated: he refused to tolerate new ideas and nothing could change his mind." The colon in this sentence indicates that the missing term should reflect the anthropologist's mental inflexibility, but the term "proliferated" instead suggests that his opinions rapidly multiplied.

Explanation for Incorrect Answer D :  Choice (D) is incorrect. "Germinated" means sprouted. If one were to insert this term into the text, the sentence would read "Over the years the anthropologist's opinions had germinated: he refused to tolerate new ideas and nothing could change his mind." The term "germinated" does not make sense in this context because the anthropologist was not open to new ideas.

Explanation for Incorrect Answer E :  Choice (E) is incorrect. "Incubated" means formed or considered slowly. If one were to insert this term into the text, the sentence would read "Over the years the anthropologist's opinions had incubated: he refused to tolerate new ideas and nothing could change his mind." "Germination" does not make sense here because the anthropologist "refused to tolerate new ideas."

6 As the first ------- of the political campaign, the senator unleashed a spirited verbal attack on her leading opponent.

ANSWERS AND EXPLANATIONS Explanation for Correct Answer A :  Choice (A) is correct. A "salvo" is a forceful assault. If one were to insert this term into the text, the sentence would read "As the first salvo of the political campaign, the senator unleashed a spirited verbal attack on her leading opponent." It makes sense that the senator would launch a verbal "assault" on her opponent at the opening of a political campaign.

Explanation for Incorrect Answer B :  Choice (B) is incorrect. An "encore" is an additional performance at an audience's request . If one were to insert this term into the text, the sentence would read "As the first encore of the political campaign, the senator unleashed a spirited verbal attack on her leading opponent." It does not make sense to say that the senator led off a political campaign with an additional performance.

Explanation for Incorrect Answer C :  Choice (C) is incorrect. A "palliative" is something that soothes. If one were to insert this term into the text, the sentence would read "As the first palliative of the political campaign, the senator unleashed a spirited verbal attack on her leading opponent." It does not make sense to say that a politician would "attack" her opponent with something meant to soothe.

Explanation for Incorrect Answer D :  Choice (D) is incorrect. A "concession" is something yielded or granted. If one were to insert this term into the text, the sentence would read "As the first concession of the political campaign, the senator unleashed a spirited verbal attack on her leading opponent." It does not make sense to say that a politician attacked her opponent by yielding or granting something to the opponent.

(A) salvo

(B) encore

(C) palliative

(D) concession

(E) demurral

页码,5/16The Official SAT Online Course

2006-11-12file://E:\新建文件夹\a8.htm

UnRe

gistered

严禁用于商业用途!

Explanation for Incorrect Answer E :  Choice (E) is incorrect. A "demurral" is a mild expression of disagreement. If one were to insert this term into the text, the sentence would read "As the first demurral of the political campaign, the senator unleashed a spirited verbal attack on her leading opponent." This sentence makes no sense because a demurral would not be described as a spirited attack.

The following passages are taken from testimony given before congressional committees about how government funding affects the arts. The author of the first passage is a writer and radio entertainer; the author of the second passage is a novelist and critic.

Passage 1

      All governments have honored artists when they are

 old and saintly and successful and almost dead, but twenty-

 five years ago Congress decided to boldly and blindly

Linesupport the arts—support the act of creation itself—and

5to encourage artists who are young and dangerous and

 unknown and very much alive. This courageous legislation

  has changed American life.

      Forty years ago, if American men or women meant to

 have artistic careers, they got on the train to New York.

10Today, you can be a violinist in North Carolina, a writer in

 Iowa, a painter in Utah. This is a small and lovely revolu-

 tion that the National Endowment for the Arts (NEA) has

 helped to bring about. The Endowment has fostered thou-

 sands of artistic works—many of which will outlive you

15and me—but even more important, the Endowment has

 changed how we think about the arts. Today, no American

 family can be secure against the danger that one of its chil-

  dren may decide to become an artist.

      I grew up in a family that never attended concerts or

20museums, never bought books. I never imagined that a

  person could be a writer.

页码,6/16The Official SAT Online Course

2006-11-12file://E:\新建文件夹\a8.htm

UnRe

gistered

严禁用于商业用途!

Passage 2

      Twice in my life, at crucial times, grants from the

 Endowment made it possible for me to be a writer. The

 first, in 1969, arrived when I was young, broke, married

25with a baby, living on very little cash and a big vegetable

 garden. I was writing for The New Yorker at the time, but

 they weren’t aware of it. I wrote every morning and every

 night. I often had fantasies of finding a patron—a beggar

 would appear at my door, I’d give him an egg salad sand-

30wich, and suddenly he’d turn into a man in a pinstripe suit,

 Prince Bob from a philanthropic foundation. But instead, I

 got a letter offering me a job for one month in the Writers

 in the Schools program in Minneapolis, funded by the

 NEA, which sent young writers into the schools to read

35 and teach.

      In 1974 a grant from the NEA enabled me and my col-

 leagues at a public radio station to start a new radio series.

 By the time the show became popular, the Endowment had

  vanished from the credits, its job done.

40     When you’re starting out, it seems like nobody wants

 to give you a dime, and then, when you have big success

 and have everything you could ever want, people can’t do

 enough for you. The Endowment is there at the beginning,

  and that’s the beauty of it.

45     I love my country’s government for its attempt in a pre-

 carious world to sustain a peaceful order in which work can

 be done and happiness can be pursued, not for the good of

页码,7/16The Official SAT Online Course

2006-11-12file://E:\新建文件夹\a8.htm

UnRe

gistered

严禁用于商业用途!

 the state, but in a state that exists for our good.

      I love my government not least for the extent to which

50it leaves me alone. My personal ambition has been simply

 to live by the work of my pen. This is not a very fastidious

 ambition. If I were aware of large amounts of federal money

 available to purveyors of the written word, I would attempt

 to gain access to it and hope to please the administrators of

55this fund as I hope to please magazine editors and book

  buyers.

      But I would rather have as my patron a host of anony-

 mous citizens digging into their own pockets for the price

 of a book or a magazine than a small body of enlightened

60and responsible people administering public funds. I would

 rather chance my personal vision of the truth striking home

 here and there in the chaos of publication that exists than

 attempt to filter it through a few sets of official, honorable,

  and public-spirited scrutinizers.

65     The realms of scientific research are now inextricably

 involved with government funding. Can we fear that the

 humanities might become similarly dependent? If I try to

 think of who in the last century has most brilliantly illumi-

 nated our sense of humanity, which I take to be the end

70purpose of the humanities, I think of Freud and Kafka, of

 Proust and Joyce, of Whitman, of Henry James. I wonder

 how many of these brave, strange, stubborn spirits would

  have wanted subsidies from their governments.

      How can public-salaried officials not think in terms of

75 respectability, of social optimism, of broad and

页码,8/16The Official SAT Online Course

2006-11-12file://E:\新建文件夹\a8.htm

UnRe

gistered

严禁用于商业用途!

uncontro-

 versial appeal? How can legislators, asked to vote tax money

 away, not begin to think of guidelines that insidiously edge

  toward censorship?

      If government money becomes an increasingly impor-

80tant presence in the financing of the humanities, is there

 a danger, I respectfully ask, of humanists becoming

  politicians?

7 The argument in Passage 1 is supported primarily by

ANSWERS AND EXPLANATIONS Explanation for Correct Answer B :  Choice (B) is correct. The author tells about being given two grants that "made it possible for me to be a writer."

Explanation for Incorrect Answer A :  Choice (A) is incorrect. The author is concerned with discussing the effect of funding on artists, not with discussing how art is created.

Explanation for Incorrect Answer C :  Choice (C) is incorrect. The author of Passage 1 does not name any renowned artists.

Explanation for Incorrect Answer D :  Choice (D) is incorrect. Far from providing evidence of harmful effects of arts funding, the author expresses gratitude for the positive effects of funding.

Explanation for Incorrect Answer E :  Choice (E) is incorrect. The author does not suggest that citizens have a right to arts funding.

(A) a theory of how art is created

(B) the author’s personal experiences

(C) examples of renowned artists

(D) evidence of the harmful effects of arts funding

(E) emotional appeals to uphold the rights of citizens

8 Which is a likely response by the author of Passage 2 to the description of artists as “dangerous” (line 5) in Passage 1?

ANSWERS AND EXPLANATIONS

(A) Practical concerns rarely prevent artists from speaking out on controversial issues.

(B) Artists should not be permitted to undermine the values of their society.

(C) Artists will cease taking risks if they come to depend on government money.

(D) The future of the arts in the United States depends on whether young artists can continue to shock the public.

(E) Experienced artists know better how to excite the public than do young artists.

页码,9/16The Official SAT Online Course

2006-11-12file://E:\新建文件夹\a8.htm

UnRe

gistered

严禁用于商业用途!

Explanation for Correct Answer C :  Choice (C) is correct. The author of Passage 2 suggests that being offered large amounts of government money could cause artists to "hope to please the administrators of this fund" who "think in terms of respectability." In other words, being offered government money would make artists less likely to take artistic risks.

Explanation for Incorrect Answer A :  Choice (A) is incorrect. The author of Passage 2 indicates that dependence on public funding tends to make artists too eager to please "official, honorable" fund administrators.

Explanation for Incorrect Answer B :  Choice (B) is incorrect. By indicating that censorship of artists is unacceptable, the author of Passage 2 implies that freedom of artistic expression is more important than such values as "respectability" or "broad and uncontroversial appeal."

Explanation for Incorrect Answer D :  Choice (D) is incorrect. There is no evidence that the author of Passage 2 believes art must be shocking in order to be good.

Explanation for Incorrect Answer E :  Choice (E) is incorrect. The author of Passage 2 makes no comparison between the effect on the public of the works of experienced artists and those of young artists.

9 In lines 10-11, Passage 1, the author refers to North Carolina, Iowa, and Utah to

ANSWERS AND EXPLANATIONS Explanation for Correct Answer C :  Choice (C) is correct. The author uses these examples—states in several parts of the United States—to show that opportunities for artistic success are less limited than they once were.

Explanation for Incorrect Answer A :  Choice (A) is incorrect. The examples point out that many different kinds of places support artistic creativity, not that such creativity is limited to certain places.

Explanation for Incorrect Answer B :  Choice (B) is incorrect. The examples support the author's argument by exemplifying the variety encouraged by government grants. They are not presented as exceptions to the argument.

Explanation for Incorrect Answer D :  Choice (D) is incorrect. The author of Passage 1 doesn't discuss the relationship between legislators and their constituents.

Explanation for Incorrect Answer E :  Choice (E) is incorrect. The author of Passage 1 does not comment on any relationship between arts funding and the democratic form of government.

(A) prove that certain environments support creativity better than others

(B) support the argument by mentioning how few exceptions there are to it

(C) show that opportunities for artistic success are now widely available

(D) suggest that legislators should listen to their constituents

(E) offer a parallel between arts funding and democracy

10 The statement in16-18, Passage 1 (“Today . . . artist”) is best described as an example of

(A) an ironic comment

页码,10/16The Official SAT Online Course

2006-11-12file://E:\新建文件夹\a8.htm

UnRe

gistered

严禁用于商业用途!

ANSWERS AND EXPLANATIONS Explanation for Correct Answer A :  Choice (A) is correct. An "ironic comment" is one that is humorous because it presents an "incongruity," a situation that is unexpected in a particular context.

Explanation for Incorrect Answer B :  Choice (B) is incorrect. The tone of the sentence is humorous, rather than pleading or emotional.

Explanation for Incorrect Answer C :  Choice (C) is incorrect. A "moral pronouncement" is a formal or authoritative statement about distinctions between right and wrong; it is unlikely to be humorous or to include incongruity.

Explanation for Incorrect Answer D :  Choice (D) is incorrect. The sentence does not present a definition of any kind.

Explanation for Incorrect Answer E :  Choice (E) is incorrect. Although the likelihood that a family's child might become an artist could be a subject for research, the author does not mention any such research.

(B) an emotional plea

(C) a moral pronouncement

(D) a definition of a key concept

(E) a generalization supported by research

11The “man in a pinstripe suit” (line 30, Passage 1) is

ANSWERS AND EXPLANATIONS Explanation for Correct Answer E :  Choice (E) is correct. A "benefactor" is one who gives benefits or gifts, and a "fantasy," like the man in the pinstripe suit, is something imagined.

Explanation for Incorrect Answer A :  Choice (A) is incorrect. Although the pinstripe suit could be seen as a stereotype that suggests a fastidious bureaucrat, a government administrator with high standards, the man is better described as an imagined benefactor in the context of the sentence.

Explanation for Incorrect Answer B :  Choice (B) is incorrect. The man is described as a character in a fantasy about funding, not in a novel.

Explanation for Incorrect Answer C :  Choice (C) is incorrect. The man is described as being from a philanthropic foundation, a group that distributes funds; he is not described as a writer.

Explanation for Incorrect Answer D :  Choice (D) is incorrect. A man from a philanthropic foundation is more likely to distribute funds rather than to serve as a critic.

(A) a fastidious bureaucrat

(B) a character in a novel

(C) a famous writer

(D) an anonymous critic

(E) an imagined benefactor

页码,11/16The Official SAT Online Course

2006-11-12file://E:\新建文件夹\a8.htm

UnRe

gistered

严禁用于商业用途!

12 Lines 40-43 (“When . . . for you”) suggest that the author of Passage 1 would agree with which of the following observations?

ANSWERS AND EXPLANATIONS Explanation for Correct Answer C :  Choice (C) is correct. The sentence indicates that artists who are already successful are likely to continue to be rewarded: being successful leads to more success. Another way of saying this is "Nothing succeeds like success."

Explanation for Incorrect Answer A :  Choice (A) is incorrect. The sentence has to do with others' reactions to artists' success, not with the value artists place on success.

Explanation for Incorrect Answer B :  Choice (B) is incorrect. The sentence does not state or suggest anything about the relationship between money and evil.

Explanation for Incorrect Answer D :  Choice (D) is incorrect. The author does not suggest that artists should look for the advantages of having no funding.

Explanation for Incorrect Answer E :  Choice (E) is incorrect. The sentence has nothing to do with the value of artists' time.

(A) Those who never succeed value success most.

(B) The love of money is the root of all evil.

(C) Nothing succeeds like success.

(D) Make a virtue of necessity.

(E) Time is money.

13

The author of Passage 2 would most likely criticize the author of Passage 1 on the grounds that

ANSWERS AND EXPLANATIONS Explanation for Correct Answer E :  Choice (E) is correct. The author of Passage 2 expresses concern that artists might be tempted to try to please those who provide grants. Such temptation, it can be inferred, might inhibit artists by steering them away from controversial projects.

Explanation for Incorrect Answer A :  Choice (A) is incorrect. Although the author of Passage 1 uses humor more freely in making an argument than does the author of Passage 2, there is no indication that the author of Passage 2 is critical of using humor.

Explanation for Incorrect Answer B :  Choice (B) is incorrect. The author of Passage 2 is not concerned with artists' need but with their tendency to try to please funding administrators.

(A) humor detracts from the seriousness of the issue being discussed

(B) public funding is often given to artists who do not need it

(C) it is invalid to assume that artists can also be teachers

(D) taxes will be too high if the government supports the arts

(E) public funding is just as likely to inhibit artists as it is to encourage them

页码,12/16The Official SAT Online Course

2006-11-12file://E:\新建文件夹\a8.htm

UnRe

gistered

严禁用于商业用途!

Explanation for Incorrect Answer C :  Choice (C) is incorrect. The author of Passage 2 does not comment at all on the ability of artists to be teachers.

Explanation for Incorrect Answer D :  Choice (D) is incorrect. The author of Passage 2 does not express concern about the effect of funding for the arts on taxes.

14 In line 57, Passage 2, “host” most nearly means

ANSWERS AND EXPLANATIONS Explanation for Correct Answer A :  Choice (A) is correct. It would take a "large number" of such citizens to support the author.

Explanation for Incorrect Answer B :  Choice (B) is incorrect. The word "host" is used to mean a "large number," not an organism.

Explanation for Incorrect Answer C :  Choice (C) is incorrect. Although a "host" would, in a sense, be a "provider" for the author, it is clear that the author is speaking of a large number of people.

Explanation for Incorrect Answer D :  Choice (D) is incorrect. This term does not make sense in the context of the sentence: the author's patron wouldn't be a "proprietor" of anonymous citizens.

Explanation for Incorrect Answer E :  Choice (E) is incorrect. In the context of this sentence, it does not make sense to say that the author's patron is a sponsor of anonymous citizens.

(A) large number

(B) sustaining organism

(C) provider

(D) proprietor

(E) sponsor

15

The question in lines 66-67 suggests that the author of Passage 2 believes that federal funding of scientific research

ANSWERS AND EXPLANATIONS Explanation for Correct Answer B :  Choice (B) is correct. Funding that deprives the recipient of independence "undermines autonomy," or weakens one's ability to make one's own decisions.

Explanation for Incorrect Answer A :  Choice (A) is incorrect. The context doesn't suggest that deceit is involved in funding in any way.

(A) encourages deceit

(B) undermines autonomy

(C) encourages an unhealthy competitiveness

(D) develops a superficial sense of loyalty

(E) spends public money under false pretenses

页码,13/16The Official SAT Online Course

2006-11-12file://E:\新建文件夹\a8.htm

UnRe

gistered

严禁用于商业用途!

Explanation for Incorrect Answer C :  Choice (C) is incorrect. Nothing in the passage suggests that a lack of independence fosters competitive behavior.

Explanation for Incorrect Answer D :  Choice (D) is incorrect. The context does not concern researchers' feelings of loyalty but their dependence on government funding.

Explanation for Incorrect Answer E :  Choice (E) is incorrect. The author's remarks about scientific research have nothing to do with the possibility of misrepresentation.

16 The author of Passage 2 most likely thinks that the individuals named in lines 70-71 would have

ANSWERS AND EXPLANATIONS Explanation for Correct Answer E :  Choice (E) is correct. Since the author suggests in lines 65-67 that government funding may limit independence, it can be inferred that these individuals would not want to give control of their creativity to anyone.

Explanation for Incorrect Answer A :  Choice (A) is incorrect. The author questions whether these individuals would want government funding themselves but does not suggest anything about their opinion of government funding for other artists.

Explanation for Incorrect Answer B :  Choice (B) is incorrect Since the context questions whether these individuals would have wanted government funding, it is unlikely that the author would suspect them of avoiding controversy in order to get money.

Explanation for Incorrect Answer C :  Choice (C) is incorrect. There is no indication in Passage 2 that government funding would have made these very influential individuals even more influential.

Explanation for Incorrect Answer D :  Choice (D) is incorrect. The fact that the individuals did not receive government funding does not necessarily mean that they would have thought the image of starvation to be romantic.

(A) supported the idea of providing artists with government funding

(B) avoided writing about controversial topics if doing so brought them more funding

(C) been even more influential if they had received government funding

(D) embraced the romantic image of the starving artist

(E) refused to submit their creativity to outside control

17 The final sentence of Passage 2 serves to

ANSWERS AND EXPLANATIONS Explanation for Correct Answer D :  Choice (D) is correct. It can be inferred that if humanists became "politicians," that is, if artistic decisions were made on the basis of political considerations, artistic

(A) emphasize the moral dilemmas that artists face when selling their work

(B) indicate why artists are so often in need of finan-cial support

(C) suggest that the public should not have to subsidize the art preferred by bureaucrats

(D) warn of the likelihood of artistic compromise

(E) link arts funding in the United States with other social programs

页码,14/16The Official SAT Online Course

2006-11-12file://E:\新建文件夹\a8.htm

UnRe

gistered

严禁用于商业用途!

values would be compromised, or endangered.

Explanation for Incorrect Answer A :  Choice (A) is incorrect. In the passage, no mention has been made of the decisions artists face when selling their work to such people as magazine editors and book buyers.

Explanation for Incorrect Answer B :  Choice (B) is incorrect. The question is about the effects of government financial support, not the reasons why it is needed.

Explanation for Incorrect Answer C :  Choice (C) is incorrect. The question, like the passage, is primarily concerned with the relationship between government funding and artists, not with the involvement of the public.

Explanation for Incorrect Answer E :  Choice (E) is incorrect. There is no mention in the passage of the relationship between funding for the arts and other social programs.

18 Which of the following is an assumption in Passage 2 that the author of Passage 1 would most likely question?

ANSWERS AND EXPLANATIONS Explanation for Correct Answer A :  Choice (A) is correct. Passage 2 suggests that legislators who make funding decisions might use guidelines that "insidiously edge toward censorship." Passage 1 expresses no concern about any negative effects of funding on artists.

Explanation for Incorrect Answer B :  Choice (B) is incorrect. Neither author presents an opinion about whether artistic creation should be free of financial risk.

Explanation for Incorrect Answer C :  Choice (C) is incorrect. The author of Passage 2 is concerned with the effect of government funding on artists' artistic choices, not with the reasons artists appreciate being funded.

Explanation for Incorrect Answer D :  Choice (D) is incorrect. Passage 2 doesn't describe the artistic standards that administrators of arts funding have, though the passage does suggest that those standards, whatever they are, may be more conservative than those of the book-buying public.

Explanation for Incorrect Answer E :  Choice (E) is incorrect. Neither author expresses an opinion about the effect of democracy on artistic self-expression.

(A) Public funding of the arts increases the danger of censorship.

(B) Artistic creation should not involve taking financial risks.

(C) Artists appreciate financial support more if they earn it after suffering hardships.

(D) Administrators of arts funding have higher artistic standards than the general public.

(E) Democracy is conducive to artistic self-expression.

19

Which of the following situations would support the position taken in Passage 1 and provide examples contrary to the argument in Passage 2?

I. A federally sponsored photographer displayed an innovative collection of photographs

页码,15/16The Official SAT Online Course

2006-11-12file://E:\新建文件夹\a8.htm

UnRe

gistered

严禁用于商业用途!

that offended both the sponsor and a large segment of the public.

II. The most original works of a certain brilliant composer were those commissioned by kings.

III. A theatrical troupe from Harlem achieved prominence by drawing large audiences from its local community.

ANSWERS AND EXPLANATIONS Explanation for Correct Answer C :  Choice (C) is correct. In both situation I and situation II, governments support acts of creation; in each case, artistic expression remains unaffected.

Explanation for Incorrect Answer A :  Choice (A) is incorrect. Situation I supports the praise of government funding in Passage 1 and counters the argument in Passage 2 that funding can hinder creativity. However, both situations that meet the criteria must be included in the correct answer.

Explanation for Incorrect Answer B :  Choice (B) is incorrect. Since situation III does not have anything to do with government funding, it is irrelevant to the discussions in Passage 1 and Passage 2.

Explanation for Incorrect Answer D :  Choice (D) is incorrect. Whereas situation II supports the position taken in Passage 1 in favor of arts funding and provides an example contrary to the argument in Passage 2 that funding may inhibit artists, situation III has nothing to do with funding.

Explanation for Incorrect Answer E :  Choice (E) is incorrect. Although situations I and II support the position taken in Passage 1 and provide examples contrary to the argument in Passage 2, situation III does not meet these criteria and so it cannot be included in the correct answer.

(A) I only

(B) III only

(C) I and II only

(D) II and III only

(E) I, II, and III

    

Back to Score Report  

Copyright © 2006 The College Board. All rights reserved. Privacy Policy Terms of Use Contact Us

 

页码,16/16The Official SAT Online Course

2006-11-12file://E:\新建文件夹\a8.htm

UnRe

gistered

严禁用于商业用途!

Help | Profile | My Organizer | My Bookmarks | Logout

Answers and Explanations

Test Sections

Section 1

Section 2

Section 3

Section 5

Section 6

Section 7

Section 8

Section 9

Section 10

Back to Score Report  

View Answers and Explanations     Online - Practice Test #1

1 A community college charges an activity fee of per student and has a student

body of students. If every student pays the fee, what is the total amount in activity fees collected from the students?

ANSWERS AND EXPLANATIONS Explanation for Correct Answer D :  Choice (D) is correct. To find the total amount of money paid by the students, you multiply the number of students by the amount each student pays:

Explanation for Incorrect Answer A :  Choice (A) is not correct. See the explanation for the correct response (D).

Explanation for Incorrect Answer B :  Choice (B) is not correct. See the explanation for the correct response (D).

Explanation for Incorrect Answer C :  Choice (C) is not correct. See the explanation for the correct response (D).

Explanation for Incorrect Answer E :  Choice (E) is not correct. See the explanation for the correct response (D).

(A)

(B)

(C)

(D)

(E)

2

In the figure above, and

If what is the value of

(A)

(B)

(C)

(D)

页码,1/12The Official SAT Online Course

2006-11-12file://E:\新建文件夹\a9.htm

UnRe

gistered

严禁用于商业用途!

ANSWERS AND EXPLANATIONS Explanation for Correct Answer C :  Choice (C) is correct. Since and the measure of is it follows that Since and the measure of is

it follows that

Explanation for Incorrect Answer A :  Choice (A) is not correct. This is the value of

Explanation for Incorrect Answer B :  Choice (B) is not correct. See the explanation for the correct response (C).

Explanation for Incorrect Answer D :  Choice (D) is not correct. See the explanation for the correct response (C).

Explanation for Incorrect Answer E :  Choice (E) is not correct. See the explanation for the correct response (C).

(E)

3

Some common nail sizes and their corresponding lengths are shown in the table above. If nail sizes from 2d up to 10d increase by a constant length for each increase of 1d in size, what would be the length, in inches, of a 6d nail?

ANSWERS AND EXPLANATIONS Explanation for Correct Answer C :  Choice (C) is correct. Comparing the first two entries on the chart, you can

see that for a increase in nail size, the length increases by inch. To

get to the length of a nail from the given length of a nail, there is a

increase in size, and so the length must increase by inch.

(A)

(B)

(C)

(D)

(E)

页码,2/12The Official SAT Online Course

2006-11-12file://E:\新建文件夹\a9.htm

UnRe

gistered

严禁用于商业用途!

Therefore, the length of a nail is inches.

Explanation for Incorrect Answer A :  Choice (A) is not correct. See the explanation for the correct response (C).

Explanation for Incorrect Answer B :  Choice (B) is not correct. See the explanation for the correct response (C).

Explanation for Incorrect Answer D :  Choice (D) is not correct. See the explanation for the correct response (C).

Explanation for Incorrect Answer E :  Choice (E) is not correct. See the explanation for the correct response (C).

4 In a sequence of numbers, the first number is and each number after the first is more than times the preceding number. What is the fourth number in the sequence?

ANSWERS AND EXPLANATIONS Explanation for Correct Answer D :  Choice (D) is correct. If the first number in the sequence is then following the stated rule, the second number is The third number is

and the fourth is

Explanation for Incorrect Answer A :  Choice (A) is not correct. This is the number you get if you think the fourth term is However, you should plug the preceding number of the sequence into this expression, not the number of the term.

Explanation for Incorrect Answer B :  Choice (B) is not correct. See the explanation for the correct response (D).

Explanation for Incorrect Answer C :  Choice (C) is not correct. See the explanation for the correct response (D).

Explanation for Incorrect Answer E :  Choice (E) is not correct. This is the fifth number in the sequence.

(A)

(B)

(C)

(D)

(E)

5If and what is in terms of

(A)

(B)

(C)

(D)

(E)

页码,3/12The Official SAT Online Course

2006-11-12file://E:\新建文件夹\a9.htm

UnRe

gistered

严禁用于商业用途!

ANSWERS AND EXPLANATIONS Explanation for Correct Answer E :  Choice (E) is correct. Since the expression can be substituted for

in the equation This yields

Explanation for Incorrect Answer A :  Choice (A) is not correct. See the explanation for the correct response (E).

Explanation for Incorrect Answer B :  Choice (B) is not correct. See the explanation for the correct response (E).

Explanation for Incorrect Answer C :  Choice (C) is not correct. See the explanation for the correct response (E).

Explanation for Incorrect Answer D :  Choice (D) is not correct. See the explanation for the correct response (E).

6 If the average (arithmetic mean) of 5 and is 7 and the average of and is what is the average of and

ANSWERS AND EXPLANATIONS Explanation for Correct Answer C :  Choice (C) is correct. Since the average of and is it follows that

therefore, Since the average of and is it

follows that therefore, Thus, the average of and

is

Explanation for Incorrect Answer A :  Choice (A) is not correct. is the correct value of but the question asks for the average of and

Explanation for Incorrect Answer B :  Choice (B) is not correct. See the explanation for the correct response (C).

Explanation for Incorrect Answer D :  Choice (D) is not correct. is the correct value of but the question asks for the average of and

Explanation for Incorrect Answer E :  Choice (E) is not correct. The sum of and is but the question asks for the average of and

(A)

(B)

(C)

(D)

(E)

7If then

页码,4/12The Official SAT Online Course

2006-11-12file://E:\新建文件夹\a9.htm

UnRe

gistered

严禁用于商业用途!

ANSWERS AND EXPLANATIONS Explanation for Correct Answer A : 

Choice (A) is correct. Because you are subtracting from itself, the

expression in parentheses is equal to zero. When is multiplied by the result is

Explanation for Incorrect Answer B :  Choice (B) is not correct. See the explanation for the correct response (A).

Explanation for Incorrect Answer C :  Choice (C) is not correct. See the explanation for the correct response (A).

Explanation for Incorrect Answer D :  Choice (D) is not correct. See the explanation for the correct response (A).

Explanation for Incorrect Answer E :  Choice (E) is not correct. See the explanation for the correct response (A).

(A)

(B)

(C)

(D)

(E)

The figure on the left is called an ell. The lengths of some of its sides are given, and all the angles are right angles. For any positive integer n, an n-ell is the figure formed by positioning n ells adjacent to each other as shown in the 3 -ell on the right.

8

What is the perimeter of the -ell?

ANSWERS AND EXPLANATIONS Explanation for Correct Answer A :  Choice (A) is correct. The 3-ell has one side of length sides of length and sides of length Therefore, its perimeter is

(A)

(B)

(C)

(D)

(E)

页码,5/12The Official SAT Online Course

2006-11-12file://E:\新建文件夹\a9.htm

UnRe

gistered

严禁用于商业用途!

Explanation for Incorrect Answer B :  Choice (B) is not correct. See the explanation for the correct response (A).

Explanation for Incorrect Answer C :  Choice (C) is not correct. is the perimeter of the 3-ell plus the lengths of the dashed line segments inside the 3-ell. The dashed lines are not part of the perimeter and should not be counted.

Explanation for Incorrect Answer D :  Choice (D) is not correct. See the explanation for the correct response (A).

Explanation for Incorrect Answer E :  Choice (E) is not correct. is times the perimeter of an ell, but that is not equal to the perimeter of a 3-ell. In calculating you count twice the line segments shown as dashed on the 3-ell, but these are not part of the perimeter of the 3-ell and should not be counted.

9 The perimeter of an -ell is and the perimeter of a -ell is What is the perimeter of a -ell?

ANSWERS AND EXPLANATIONS Explanation for Correct Answer A :  Choice (A) is correct. When the 80-ell and the 20-ell are put together to form the 100-ell, they have a common border of length but this border is not part of the perimeter of the 100-ell. Because the length of this border is counted twice in adding the perimeters of the 80-ell and the 20-ell, the perimeter of the 100-ell is less than the sum of the two perimeters; that is, the perimeter is

Explanation for Incorrect Answer B :  Choice (B) is not correct. See the explanation for the correct response (A).

Explanation for Incorrect Answer C :  Choice (C) is not correct. Simply adding the perimeter of the 80-ell and the 20-ell will not give the perimeter of the 100-ell because of the excess length of the common border.

Explanation for Incorrect Answer D :  Choice (D) is not correct. See the explanation for the correct response (A).

Explanation for Incorrect Answer E :  Choice (E) is not correct. See the explanation for the correct response (A).

(A)

(B)

(C)

(D)

(E)

10

Which of the following graphs best represents the information in the table above?

页码,6/12The Official SAT Online Course

2006-11-12file://E:\新建文件夹\a9.htm

UnRe

gistered

严禁用于商业用途!

ANSWERS AND EXPLANATIONS Explanation for Correct Answer B :  Choice (B) is correct. From the age of months to the age of months, the growth of the snake is very rapid, so the graph should have a large positive slope. This growth is not constant, however; from months to months, the growth, while still positive, has tapered off greatly. Therefore, the graph should have a positive slope close to from months to months. The only graph shown with both of these characteristics is graph (B).

Explanation for Incorrect Answer A :  Choice (A) is not correct. This graph shows a constant positive growth rate, but from the data in the table, you can see that the growth rate is not constant.

Explanation for Incorrect Answer C :  Choice (C) is not correct. This graph shows a constant negative growth rate (analogous to a shrinking snake). According to the data in the table, not only

(A)

(B)

(C)

(D)

(E)

页码,7/12The Official SAT Online Course

2006-11-12file://E:\新建文件夹\a9.htm

UnRe

gistered

严禁用于商业用途!

is the growth rate not constant, it is also positive.

Explanation for Incorrect Answer D :  Choice (D) is not correct. This graph shows a growth rate that starts out slowly and then speeds up. According to the data in the table, just the opposite happens: the growth rate begins very quickly and then slows down.

Explanation for Incorrect Answer E :  Choice (E) is not correct. This graph shows a growth rate that begins very quickly and then reverses and turns past to a negative value. This is analogous to the case where a snake grows quickly, then grows slowly, and eventually begins to shrink. From the data in the table, you can see that this is not accurate.

11 1. Add to 2. Multiply the sum by 3. Subtract from the product. If the steps above are followed in order, which of the following is a simplified expression for the result?

ANSWERS AND EXPLANATIONS Explanation for Correct Answer E :  Choice (E) is correct. After step one, the expression is Multiplying

by two yields the expression and subtracting

gives the expression This simplifies to

Explanation for Incorrect Answer A :  Choice (A) is not correct. See the explanation for the correct response (E).

Explanation for Incorrect Answer B :  Choice (B) is not correct. See the explanation for the correct response (E).

Explanation for Incorrect Answer C :  Choice (C) is not correct. See the explanation for the correct response (E).

Explanation for Incorrect Answer D :  Choice (D) is not correct. See the explanation for the correct response (E).

(A)

(B)

(C)

(D)

(E)

12 If is a positive integer, which of the following is equivalent to

ANSWERS AND EXPLANATIONS Explanation for Correct Answer A :  Choice (A) is correct. Adding two identical terms is equivalent to multiplying the term by

(A)

(B)

(C)

(D)

(E)

页码,8/12The Official SAT Online Course

2006-11-12file://E:\新建文件夹\a9.htm

UnRe

gistered

严禁用于商业用途!

Explanation for Incorrect Answer B :  Choice (B) is not correct. This would be the answer if the expression given were

Explanation for Incorrect Answer C :  Choice (C) is not correct. This would be the answer if the expression given

were

Explanation for Incorrect Answer D :  Choice (D) is not correct. This would be the answer if the expression given

were

Explanation for Incorrect Answer E :  Choice (E) is not correct. This would be the answer if the expression given

were

13 During a one-cent sale, a shopper pays the regular price for a bottle of vegetable oil and pays for a second bottle. If the regular price of the vegetable oil is how much per bottle does the shopper save by buying two bottles at this sale?

ANSWERS AND EXPLANATIONS Explanation for Correct Answer B :  Choice (B) is correct. Normally, the shopper pays per bottle of

vegetable oil. During the sale, the shopper pays a total of for two

bottles of oil, which is per bottle of oil. The shopper saves

the difference between the two prices, or per bottle of oil.

Explanation for Incorrect Answer A :  Choice (A) is not correct. is the price of the second bottle of vegetable oil during the sale.

Explanation for Incorrect Answer C :  Choice (C) is not correct. is the average price of a bottle of vegetable oil during the sale, but the question asks for the amount saved per bottle during the sale.

Explanation for Incorrect Answer D :  Choice (D) is not correct. See the explanation for the correct response (B).

Explanation for Incorrect Answer E :  Choice (E) is not correct. is the regular price of a bottle of vegetable oil.

(A)

(B)

(C)

(D)

(E)

14If what is the value of

页码,9/12The Official SAT Online Course

2006-11-12file://E:\新建文件夹\a9.htm

UnRe

gistered

严禁用于商业用途!

ANSWERS AND EXPLANATIONS Explanation for Correct Answer E :  Choice (E) is correct. Cross multiply to get which simplifies

to Dividing both sides of this equation by gives

Explanation for Incorrect Answer A :  Choice (A) is not correct. See the explanation for the correct response (E).

Explanation for Incorrect Answer B :  Choice (B) is not correct. See the explanation for the correct response (E).

Explanation for Incorrect Answer C :  Choice (C) is not correct. See the explanation for the correct response (E).

Explanation for Incorrect Answer D :  Choice (D) is not correct. See the explanation for the correct response (E).

(A)

(B)

(C)

(D)

(E)

15A right circular cylinder has a base of circumference If the volume of the cylinder is what is the height?

ANSWERS AND EXPLANATIONS Explanation for Correct Answer B :  Choice (B) is correct. The volume of a right circular cylinder is given by the formula where is the radius of the base and is the height of the cylinder. If the base has a circumference of it must have a diameter of and thus a radius of You can then write the volume equation as

which simplifies to

Explanation for Incorrect Answer A :  Choice (A) is not correct. is the radius of the cylinder, not the height.

Explanation for Incorrect Answer C :  Choice (C) is not correct. See the explanation for the correct response (B).

Explanation for Incorrect Answer D :  Choice (D) is not correct. If the area of the base of the cylinder were the height of the cylinder would be but the question states that the circumference of the base is not the area.

(A)

(B)

(C)

(D)

(E)

页码,10/12The Official SAT Online Course

2006-11-12file://E:\新建文件夹\a9.htm

UnRe

gistered

严禁用于商业用途!

Explanation for Incorrect Answer E :  Choice (E) is not correct. See the explanation for the correct response (B).

16 In the -coordinate plane, the graph of intersects line at and

What is the least possible value of the slope of

ANSWERS AND EXPLANATIONS Explanation for Correct Answer D :  Choice (D) is correct. The slope of line equals

or To find values for and

you can use the equation of the parabola. Notice that since the points

and lie on the graph of their coordinates can be

substituted for and in the equation Substituting for

and for in the equation gives This simplifies to

and further to so Substituting for

and for in the equation gives This simplifies to

and further to so

Since there are two values for each variable, there are four possible values for the expression and

To find the least possible value of the slope of line you must compare all four possible values of the slope, which is equal to

and The least possible value of the

slope is

Explanation for Incorrect Answer A :  Choice (A) is not correct. While the slope of line could be this is not the least possible value of the slope, and the question asks for the least possible value.

Explanation for Incorrect Answer B :  Choice (B) is not correct. While the slope of line could be this is not the least possible value of the slope, and the question asks for the least possible value.

Explanation for Incorrect Answer C :  Choice (C) is not correct. While the slope of line could be this is not the least possible value of the slope, and the question asks for the least possible value.

Explanation for Incorrect Answer E :  Choice (E) is not correct. See the explanation for the correct response (D).

(A)

(B)

(C)

(D)

(E)

    

Back to Score Report  

Copyright © 2006 The College Board. All rights reserved. Privacy Policy Terms of Use Contact Us

页码,11/12The Official SAT Online Course

2006-11-12file://E:\新建文件夹\a9.htm

UnRe

gistered

严禁用于商业用途!

 

页码,12/12The Official SAT Online Course

2006-11-12file://E:\新建文件夹\a9.htm

UnRe

gistered

严禁用于商业用途!

Help | Profile | My Organizer | My Bookmarks | Logout

Answers and Explanations

Test Sections

Section 1

Section 2

Section 3

Section 5

Section 6

Section 7

Section 8

Section 9

Section 10

Back to Score Report  

View Answers and Explanations     Online - Practice Test #1

1 In their zeal to make beachfront living widely available, developers have overbuilt, thereby they endanger fragile coastlines.

ANSWERS AND EXPLANATIONS Explanation for Correct Answer C :  Choice (C) is correct. It avoids the error of the original by changing the second independent clause to a verb phrase ("have endangered") that is parallel to the earlier verb phrase ("have overbuilt") in the new compound predicate.

Explanation for Incorrect Answer A :  Choice (A) uses improper coordination. It joins two complete thoughts ("In their zeal . . . developers have overbuilt" and "thereby they endanger fragile coastlines") with only a comma.

Explanation for Incorrect Answer B :  Choice (B) displays improper coordination. It places one complete thought ("they endanger fragile coastlines as a result") immediately after another ("In their zeal . . . developers have overbuilt") with no conjunction or punctuation between them.

Explanation for Incorrect Answer D :  Choice (D) exhibits improper coordination. It uses a semicolon to join unequal sentence parts (a complete thought before the semicolon and a phrase after it).

Explanation for Incorrect Answer E :  Choice (E) involves improper coordination. It joins two complete thoughts ("In their zeal . . . developers have overbuilt" and "the fragile coastlines are endangered by this") with only a comma.

(A) overbuilt, thereby they endanger fragile coastlines

(B) overbuilt they endanger fragile coastlines as a result

(C) overbuilt and thereby have endangered fragile coastlines

(D) overbuilt; fragile coastlines endangered thereby

(E) overbuilt, the fragile coastlines are endangered by this

2Hawaii’s Haleakala, being more than 10,000 feet high, and the world’s largest dormant volcano.

ANSWERS AND EXPLANATIONS Explanation for Correct Answer D :  Choice (D) is correct. It avoids the error of the original by providing the verb "is" to complete the predicate of the sentence.

(A) Haleakala, being more than 10,000 feet high and

(B) Haleakala, more than 10,000 feet high, it is

(C) Haleakala which is more than 10,000 feet high, being

(D) Haleakala, more than 10,000 feet high, is

(E) Haleakala, more than 10,000 feet high; it is

页码,1/9The Official SAT Online Course

2006-11-12file://E:\新建文件夹\a10.htm

UnRe

gistered

严禁用于商业用途!

Explanation for Incorrect Answer A :  Choice (A) is a sentence fragment. The sentence is grammatically incomplete because it has no main verb.

Explanation for Incorrect Answer B :  Choice (B) is unsatisfactory because the word "it" introduces an independent clause ("it is... volcano"). The opening phrase ("Haleakala... high") now has no verb and is left unattached to the sentence.

Explanation for Incorrect Answer C :  Choice (C) results in a sentence fragment. The use of an improper verb form ("being" instead of "is") leaves the sentence grammatically incomplete.

Explanation for Incorrect Answer E :  Choice (E) involves improper coordination. The clause preceding the semicolon is incomplete because it has no main verb.

3 I do not blame Leslie for her anger yesterday, being it was her plan and she should have credit for it.

ANSWERS AND EXPLANATIONS Explanation for Correct Answer E :  Choice (E) is correct. It avoids the error of the original by using a colon between the main clauses as is appropriate when the second clause explains the idea in the first clause.

Explanation for Incorrect Answer A :  Choice (A) involves the use of an improper idiom. The word "being" is used to introduce the clause, "it was her plan," where it would be more idiomatic to introduce it with "because" or to separate the clauses with a colon.

Explanation for Incorrect Answer B :  Choice (B) is unsatisfactory because it involves the use of an improper idiom. The clause, "she should have credit for it," is introduced by the phrase, "being that," where it would be more idiomatic to introduce it with "since" or "because."

Explanation for Incorrect Answer C :  Choice (C) involves the use of a vague pronoun. It is not clear to whom the pronoun "they" refers.

Explanation for Incorrect Answer D :  Choice (D) involves a lack of clarity. The confusing word order makes the meaning of the sentence unclear.

(A) yesterday, being it was her plan and she should have credit for it

(B) yesterday, being that she should have credit for it when it was her plan

(C) yesterday when it was her plan, for which they should give her credit for it

(D) yesterday; since she should receive credit, it being her plan

(E) yesterday: it was her plan, and she should have received credit for it

4 At the time at which temperatures approach absolute zero, or -459.7° F, metals become highly conductive, and their volume shrinks dramatically.

(A) At the time at which temperatures approach

(B) When temperatures approach

(C) Since temperatures approached

(D) At the point temperatures had approached

页码,2/9The Official SAT Online Course

2006-11-12file://E:\新建文件夹\a10.htm

UnRe

gistered

严禁用于商业用途!

ANSWERS AND EXPLANATIONS Explanation for Correct Answer B :  Choice (B) is correct. It avoids the error of the original by providing the adverb "When" instead of the repetitive adverbial phrase, "At the time at which."

Explanation for Incorrect Answer A :  Choice (A) involves unnecessary repetition. The phrase "At the time at which" is used to introduce the subordinate clause, where the adverb "When" would be simpler and clearer.

Explanation for Incorrect Answer C :  Choice (C) involves an improper sequence of verb tenses. It provides a past-tense verb ("approached") instead of the present-tense verb that is needed to match the present-tense verbs in the main clause of the sentence.

Explanation for Incorrect Answer D :  Choice (D) is unsatisfactory because it involves an improper sequence of verb tenses. The pluperfect verb "had approached" does not match the present tense of the other verbs in the sentence.

Explanation for Incorrect Answer E :  Choice (E) involves the use of an improper idiom. It provides the phrase "While temperatures approaching absolute zero," where it would be more idiomatic to use a clause introduced by the adverb "When."

(E) While temperatures approaching

5 Participants in the executive leadership workshop expect a program of outstanding speakers and gaining information about new approaches to management.

ANSWERS AND EXPLANATIONS Explanation for Correct Answer B :  Choice (B) is correct. It avoids the error of the original by using two parallel nouns, "information" and "program," to name the two things participants expect.

Explanation for Incorrect Answer A :  Choice (A) fails to maintain parallelism. The verbal phrase "gaining information" is not parallel with the earlier noun "program."

Explanation for Incorrect Answer C :  Choice (C) does not maintain parallelism. The verbal phrase "being informed" is not parallel with the earlier noun "program."

Explanation for Incorrect Answer D :  Choice (D) has a flaw in parallelism. The verbal phrase "being informed" is not parallel with the earlier noun "program."

Explanation for Incorrect Answer E :  Choice (E) exhibits improper coordination. It places one complete thought ("in addition . . . gain information") immediately after another ("Participants . . . of outstanding speakers") with no conjunction or punctuation to connect them.

(A) and gaining information

(B) as well as information

(C) as well as being informed

(D) and also being informed

(E) in addition, they expect to gain information

页码,3/9The Official SAT Online Course

2006-11-12file://E:\新建文件夹\a10.htm

UnRe

gistered

严禁用于商业用途!

6 One of the unforeseen consequences of the editor’s management style is that it leaves so little room for innovation.

ANSWERS AND EXPLANATIONS Explanation for Correct Answer A :  Choice (A) is correct. It appropriately uses a subordinate noun clause, introduced by the relative pronoun "that," to describe a result of the editor's management style.

Explanation for Incorrect Answer B :  Choice (B) involves noun-pronoun disagreement. It improperly uses the plural pronoun "they" to refer to the singular phrase "the editor's management style."

Explanation for Incorrect Answer C :  Choice (C) involves the use of a vague pronoun. There is nothing in the sentence to which the pronoun "their" can logically refer.

Explanation for Incorrect Answer D :  Choice (D) results in an illogical sentence. The phrase "innovation has so little room left from it" makes no sense.

Explanation for Incorrect Answer E :  Choice (E) involves the use of an improper idiom. It provides an infinitive phrase ("to leave... innovation") where it would be more idiomatic to use a subordinate clause introduced by the relative pronoun "that."

(A) that it leaves so little room for innovation

(B) that they leave so little room for innovation

(C) that sufficient room is not left for their innovation

(D) that innovation has so little room left from it

(E) to leave so little room for innovation

7For decades, African American music has inspired musicians throughout the world, including in Russia.

ANSWERS AND EXPLANATIONS Explanation for Correct Answer B :  Choice (B) is correct. It avoids the error of the original by changing the vague phrase "in Russia" to "those of Russia," where the pronoun "those" clearly refers to musicians.

Explanation for Incorrect Answer A :  Choice (A) involves the use of a vague modifying phrase. It is not clear whether the phrase "in Russia" refers to inspiration or to musicians.

Explanation for Incorrect Answer C :  Choice (C) involves the use of a vague pronoun. It is not clear what the pronoun "this" refers to.

Explanation for Incorrect Answer D : 

(A) including in Russia

(B) including those of Russia

(C) this includes Russia

(D) one of which is Russia

(E) one example being Russia

页码,4/9The Official SAT Online Course

2006-11-12file://E:\新建文件夹\a10.htm

UnRe

gistered

严禁用于商业用途!

Choice (D) results in an illogical sentence. There is nothing in the sentence to which the pronoun "which" can logically refer.

Explanation for Incorrect Answer E :  Choice (E) is unsatisfactory because it results in an illogical sentence. There is nothing in the sentence of which Russia could logically be an example.

8 By the end of the eighteenth century, watchmaking technology had greatly improved, and they were standard equipment for military personnel.

ANSWERS AND EXPLANATIONS Explanation for Correct Answer D :  Choice (D) is correct. It avoids the error of the original by providing the noun "watches" in place of the vague pronoun "they."

Explanation for Incorrect Answer A :  Choice (A) involves the use of a vague pronoun. There is no plural noun in the sentence to which the pronoun "they" can logically refer.

Explanation for Incorrect Answer B :  Choice (B) involves the use of an improper idiom. It uses the phrase "it was standard equipment for... watches" where it would be more idiomatic to say "watches were standard equipment."

Explanation for Incorrect Answer C :  Choice (C) results in an illogical sentence. The linking word "with" illogically indicates that the issuing of watches to military personnel was a part of the improvement in watchmaking technology rather than a result of it.

Explanation for Incorrect Answer E :  Choice (E) involves the use of a vague pronoun. It is not clear what the pronoun "it" refers to.

(A) and they were standard equipment for military personnel

(B) so it was standard equipment for military personnel to have watches

(C) with watches included in the standard equipment for military personnel

(D) and watches had become standard equipment for military personnel

(E) and for military personnel it was standard equipment

9 Nancy and Carlos will represent Central High in the swimming competition, their work in this having been excellent this year.

ANSWERS AND EXPLANATIONS Explanation for Correct Answer D :  Choice (D) is correct. It avoids the error of the original by using the specific noun "swimming" to replace a wordy phrase ("work in this") containing a vague pronoun, "this."

Explanation for Incorrect Answer A :  Choice (A) involves wordiness and use of a vague pronoun. In the phrase "work in

(A) competition, their work in this having been excellent this year

(B) competition, they have done excellent work this year in this

(C) competition, for this year they have done excellent work in this

(D) competition, for their swimming has been excellent this year

(E) competition, their work as swimmers having been excellent this year

页码,5/9The Official SAT Online Course

2006-11-12file://E:\新建文件夹\a10.htm

UnRe

gistered

严禁用于商业用途!

this," the meaning of the pronoun "this" is unclear.

Explanation for Incorrect Answer B :  Choice (B) displays improper coordination. It joins two complete thoughts ("Nancy and Carlos will represent . . . swimming competition" and "they have done excellent work this year in this") with only a comma.

Explanation for Incorrect Answer C :  Choice (C) uses an ambiguous pronoun. The pronoun "this" could refer either to swimming or to swimming competition.

Explanation for Incorrect Answer E :  Choice (E) exhibits wordiness. The phrase "work as swimmers" can be reduced to one word, "swimming."

10 After 1907, residents of the Omaha Reservation could use the hospital in Walthill, Nebraska, it was established by Dr. Susan LaFlesche Picotte, an Omaha Indian.

ANSWERS AND EXPLANATIONS Explanation for Correct Answer E :  Choice (E) is correct. It avoids the error of the original by replacing the clause introduced by "it was" with an appropriate modifying phrase, "established... Omaha Indian," to describe the hospital.

Explanation for Incorrect Answer A :  Choice (A) involves improper coordination. Two complete thoughts are linked by only a comma.

Explanation for Incorrect Answer B :  Choice (B) involves awkward and illogical phrasing. The sentence does not make sense because the word "and" makes the hospital appear to have existed before it was established.

Explanation for Incorrect Answer C :  Choice (C) involves an improper sequence of tenses. The tense of the verb phrase “has been established” does not combine logically with the tense of the verb phrase “could use” earlier in the sentence.

Explanation for Incorrect Answer D :  Choice (D) involves the use of a vague pronoun. It is not clear what the pronoun "it" refers to.

(A) hospital in Walthill, Nebraska, it was established by Dr. Susan LaFlesche Picotte, an Omaha Indian

(B) hospital; it was in Walthill, Nebraska and established by Dr. Susan LaFlesche Picotte, an Omaha Indian

(C) hospital that has been established by Dr. Susan LaFlesche Picotte, an Omaha Indian, in Walthill, Nebraska

(D) Walthill, Nebraska, hospital where an Omaha Indian, Dr. Susan LaFlesche Picotte, established it

(E) hospital established in Walthill, Nebraska, by Dr. Susan LaFlesche Picotte, an Omaha Indian

11

Eating food that has a high concentration of fat causes essentially the same reaction in the stomach than if you eat too fast.

(A) than if you eat

(B) than to eat

(C) as if one eats

(D) as eating

(E) as it does when eating

页码,6/9The Official SAT Online Course

2006-11-12file://E:\新建文件夹\a10.htm

UnRe

gistered

严禁用于商业用途!

ANSWERS AND EXPLANATIONS Explanation for Correct Answer D :  Choice (D) is correct. It avoids the error of the original by using the word "as," which pairs with the earlier word "same" to create an appropriate idiom indicating similarity.

Explanation for Incorrect Answer A :  Choice (A) uses an improper idiom. After the word “same,” the conjunction “than” is inappropriate because it indicates difference rather than similarity.

Explanation for Incorrect Answer B :  Choice (B) contains an improper idiom. After the word "same," the conjunction "than" is inappropriate because it indicates difference rather than similarity.

Explanation for Incorrect Answer C :  Choice (C) fails to maintain parallelism. The clause "if one eats" is not parallel with the earlier verbal "eating."

Explanation for Incorrect Answer E :  Choice (E) exhibits wordiness. The words "it does when" are not needed.

12 Not one of the students in the advanced chemistry class have passed a single test with a grade better than a C, but the second half of the course will be easier.

ANSWERS AND EXPLANATIONS Explanation for Correct Answer E :  Choice (E) is correct. It avoids the error of the original by making the singular verb "has" agree with its singular subject, "one."

Explanation for Incorrect Answer A :  Choice (A) has an error in subject-verb agreement. Even though the plural verb "have" agrees with the interrupting noun "students," it does not agree with its singular subject, "one."

Explanation for Incorrect Answer B :  Choice (B) contains an error in subject-verb agreement. The plural verb "have" agrees with the interrupting noun "students," but not with with its singular subject, "one."

Explanation for Incorrect Answer C :  Choice (C) exhibits an error in subject-verb agreement. While the plural verb "have" does agree with the interrupting noun "students," it does not agree with its singular subject, "one."

Explanation for Incorrect Answer D :  Choice (D) displays loose modification. This word order fails to make clear exactly what the verbal phrase "having better than a C grade on a single test" modifies.

(A) have passed a single test with a grade better than a C

(B) have managed to pass a single test with better than a C grade

(C) have passed a single test any better than a grade of C

(D) has passed having better than a C grade on a single test

(E) has passed a single test with better than a C grade

13 In neighborhoods throughout the United States, one can encounter hundreds of different rope-jumping games, each with its own rules.

页码,7/9The Official SAT Online Course

2006-11-12file://E:\新建文件夹\a10.htm

UnRe

gistered

严禁用于商业用途!

ANSWERS AND EXPLANATIONS Explanation for Correct Answer A :  Choice (A) is correct. The singular pronoun "its" agrees with the earlier singular pronoun "each."

Explanation for Incorrect Answer B :  Choice (B) involves a shift in pronoun number. The plural pronoun "their" does not agree with the earlier singular pronoun "each."

Explanation for Incorrect Answer C :  Choice (C) uses ineffective subordination. The conjunction "when" incorrectly suggests that the main relationship between the two clauses is temporal.

Explanation for Incorrect Answer D :  Choice (D) displays disagreement of subject and verb and also of noun and pronoun. Although the pronoun "which" refers to the plural noun "games," neither the singular verb "has" nor the singular pronoun "its" agrees with that noun.

Explanation for Incorrect Answer E :  Choice (E) uses improper coordination. It joins two complete thoughts ("In neighborhoods . . . rope-jumping games" and " they each . . . of their own") with only a comma.

(A) each with its own rules

(B) each having their own rules

(C) when they each have their own rules

(D) which has its own rules

(E) they each have rules of their own

14

A flurry of do-it-yourself books on the market today are inspiring homeowners to do their own repairs.

ANSWERS AND EXPLANATIONS Explanation for Correct Answer D :  Choice (D) is correct. It avoids the error of the original by making the singular verb "is" agree with its singular subject, "flurry."

Explanation for Incorrect Answer A :  Choice (A) has an error in subject-verb agreement. Even though the plural verb "are" agrees with the interrupting noun "books," it does not agree with its singular subject, "flurry."

Explanation for Incorrect Answer B :  Choice (B) contains an error in subject-verb agreement. The plural verb "are" agrees with the interrupting noun "books," but not with with its singular subject, "flurry."

Explanation for Incorrect Answer C :  Choice (C) uses inappropriate idioms. After the verb "is inspiring," the phrase "into doing" is less effective than "to do," and "repairing" is less effective than "repairs."

(A) are inspiring homeowners to do their own repairs

(B) are inspiring to homeowners about their own repairs

(C) is inspiring homeowners into doing their own repairing

(D) is inspiring homeowners to do their own repairs

(E) inspiring homeowners to repair their own homes

页码,8/9The Official SAT Online Course

2006-11-12file://E:\新建文件夹\a10.htm

UnRe

gistered

严禁用于商业用途!

Explanation for Incorrect Answer E :  Choice (E) creates a sentence fragment. Since the phrase has no verb (only the verbal forms "inspiring" and "to repair"), it does not state a complete thought.

  

Back to Score Report  

Copyright © 2006 The College Board. All rights reserved. Privacy Policy Terms of Use Contact Us

 

页码,9/9The Official SAT Online Course

2006-11-12file://E:\新建文件夹\a10.htm

UnRe

gistered

00
打字机
更多SAT考试资料,请登录http://sat.tiandaoedu.com获取